Карта и территория

«Карта и территория» («Map and Territory») — первая часть электронной книги Элиезера Юдковского «Рациональность: от ИИ до зомби» (Rationality: From AI to Zombies). Это отредактированная и расширенная версия серии блог-постов из числа «Цепочек» («The Sequences»), и в ней рассматривается природа рациональности, убеждений и объяснений.

«Карта и территория» содержит четыре «цепочки» эссе, завершаясь самодостаточным эссе «Простая истина». Все они собраны в книге «Рациональность: от ИИ до зомби», но здесь собраны ссылки на их переводы.

Следующая часть: Как успешно менять своё мнение.

Предисловие

Элиезер Юдковский

Перед вами — собрание записей в блог за два года. Сейчас я оглядываюсь на этот проект и вижу, что многое сделал совершенно неправильно. Я считаю, это хорошо. Если бы я оглянулся назад и НЕ увидел ошибок — это бы значило, что с 2009 года я не вырос как писатель и не улучшил своё понимание материала. «Ой!» — говорим мы, когда пересматриваем свои убеждения и стратегии. Если, оглядываясь назад, ты не видишь, что именно ты сделал неправильно — значит, ты с тех пор ничему не научился и не изменил свое мнение.

В течение двух лет я писал посты в блог и не думал о том, чтобы помогать людям в их повседневной жизни, и я ошибался. Я хотел помогать людям с большими, сложными и важными проблемами, и подбирал впечатляющие абстрактные примеры.

По прошествии времени я вижу, что это было второй по важности ошибкой в моём подходе. Она была связана с первой: я не понимал, что огромная проблема в изучении описанного мною способа рассуждений — применять его на практике, а не просто знать теорию. Я не осознавал, что важна именно практическая часть; оглядываясь назад, я могу сказать разве что «Ой!» и «Эх».

Да, порой те самые большие проблемы и вправду важны, но это не отменяет простой правды: чтобы мастерски овладеть этими навыками, нужно практиковаться, а это трудно делать на отвлеченных задачах. (Сегодня CFAR, Центр прикладной рациональности, постоянно работает над тем, чтобы устранить последствия этой серьёзной ошибки).

И третья огромная ошибка в том, что я слишком сильно концентрировался на рациональных убеждениях — и слишком мало на рациональном действии.

Четвёртая по важности ошибка — организация содержания цепочек. В частности, нужно было раньше создать вики и предоставить возможность читать посты как части цепочек.

Эту ошибку по крайней мере можно исправить. В нашей книге «Rationality: From AI to Zombies» Роб Бенсинджер отредактировал мои записи и организовал их по-другому, стараясь не переписывать содержание материалов (хотя кое-где всё же пришлось это сделать).

Моей пятой ошибкой было то, что я — как мне казалось — пытался прямо говорить о глупости того, что казалось мне глупым. Я хотел избежать искажения под названием бульверизм — это когда вы начинаете обсуждение с того, что люди, которые верят в то-то и то-то, глупые. Сначала я рассматривал саму тему, а потом уже говорил: «Вот почему это глупо». Но в 2009 году я ещё не определился, важно ли, чтобы меня окружали люди, в открытую презирающие гомеопатию. Я считал (и до сих пор считаю), что если вежливо обходиться с какой-то идеей, то некоторые люди понимают это как «ничего страшного, если я скажу, что верю в гомеопатию; это не повредит моему статусу». Что ирония и насмешки могут развеять их иллюзии.

Думаю, сейчас я был бы вежливее. Беспардонность послужила определенной цели и даже кому-то помогла. Однако теперь я более серьёзно отношусь к риску построить сообщество, в котором нормальной и ожидаемой реакцией на новичка будет открытое презрение и насмешки.

Тем не менее я очень рад, что моя читательская аудитория не стала использовать мою риторику для унижения или травли. (Я хотел бы отдельно упомянуть Скотта Александера: он более приятный человек, чем я, и отлично пишет на эти темы, а потому заслуживает благодарности за построение здоровой атмосферы сообщества Less Wrong.)

Если я оглядываюсь назад и сообщаю, что провалился — значит, у меня были цели, которых я хотел достичь. Что же это за цели?

Есть один ценный способ рассуждений, которому пока не учат в школах. Ему систематически вообще нигде не учат. Он просто проявляется сам у тех, кто вырос на книгах вроде «Вы, конечно, шутите, мистер Фейнман» — или у тех, кому в университете повезло с хорошим преподавателем.

Чаще всего этот способ мышления практикуется в науке и проведении экспериментов. Просто идёте и смотрите на Вселенную, а не выдумываете. И говорите «Ой!» и отказываетесь от плохой теории, если экспериментом она не подтверждается.

Но такой склад ума применяется и шире. Он универсальнее тех очков, которые вы надеваете в лаборатории. Его можно использовать в реальной жизни, хотя тут есть свои нюансы. Если вы не можете сказать «Ой!» и отказаться от нерабочей теории — вы будете постоянно стрелять себе в ногу. Вы обречены перезаряжать оружие и спускать курок. Вы знаете таких людей. И иногда вы, даже не задумываясь об этом, действуете в точности как они. Хорошо бы существовал способ мышления, благодаря которому можно перестать так делать.

Несмотря на серьёзность моих ошибок, статьи, выложенные за два года ведения моего блога помогли удивительному количеству людей, чего я не ожидал. Это работает не слишком надёжно, но иногда всё-таки срабатывает.

В современном обществе так мало учат навыкам рациональных убеждений и принятия решений, так мало учат математике и другим наукам, на которых всё это основано! В результате даже просто прочитать мои излияния о проблемах науки и философии может быть полезным. Посмотрите на всё это с десятков разных точек зрения — это иногда помогает уловить единый ритм.

На самом деле всё сводится к одному и тому же. Я описывал огромные отвлеченные задачи и пренебрегал повседневной жизнью, но законы в обоих случаях одинаковы. Я концентрировался на одном и упускал многое другое; но всё сводится к одному. Я горжусь тем, что оглядываюсь назад — и даже при учёте всех моих ошибок и всех моих «Ой!», даже спустя пять лет я всё ещё считаю, что это лучше, чем ничего.

— февраль 2015

Перевод: 
Remlin, Muyyd, Staenrey
Оцените качество перевода: 
Средняя оценка: 5 (53 votes)

Искажения: введение

Роб Бенсинджер

Это не тайна. Но почему-то это редко всплывает в разговорах и очень немногие спрашивают, что же нам с этим делать. Это шаблон, спрятанный за всеми нашими победами и поражениями, невидимый нашему глазу. Что же это?

Представьте себе урну с 70 белыми и 30 красными шарами; вы вытаскиваете 10 наугад. Возможно, 3 из них будут красными, и вы верно угадаете, сколько всего красных шаров в урне. Или, возможно, у вас будет 4 красных шара, а может быть, другое число. И тогда вы получите неверное общее число.

Эта случайная ошибка — цена за неполное знание, и это в целом неплохо, как и любая ошибка. В среднем наши оценки будут верны, и чем больше мы узнаём — тем меньше мы делаем ошибок.

Но предположим, что белые шары тяжелее и опускаются на дно урны. Тогда ваша выборка может быть нерепрезентативной постоянно.

Такие ошибки называются систематическими. Когда ваш метод познания мира искажён, изучение мира может вводить вас в заблуждение. Накопленные данные могут даже искажать предсказания. Для человека, привыкшего высоко ценить знания и способы их получения, это пугающая перспектива. Если мы хотим быть уверены, что познание помогает, а не делает всё только хуже, нужно узнавать об искажениях в наших данных и исправлять их.

Идея когнитивного искажения в психологии работает похожим образом. Когнитивное искажение — систематическая ошибка нашего мышления (в противоположность случайной ошибке, вызванной простым невежеством). Статистическая погрешность искажает выборку так, что она менее точно отражает положение дел. Когнитивные искажения же портят не выборку, а наши убеждения, и убеждения менее точно отражают факты; такие искажения вмешиваются в процесс принятия решений, и мы достигаем наших целей с меньшей надёжностью.

Возможно, вы оптимист, а красные шары можно использовать как лекарство от редкой тропической болезни, которую подхватил ваш брат. Тогда вы переоцените количество красных шаров в урне, потому что вам хочется, чтобы их было больше. И здесь искажена не выборка, а вы сами.

Тем не менее нужно быть осторожным, говоря про искажения в людях. Обычно мы имеем в виду, что люди нечестны или предвзяты. Но когнитивные искажения — это совершенно другое. Они — неотъемлемая часть человеческого мышления, а не дефект, который можно списать на воспитание или гнилой характер1.

Когнитивное искажение — это проторенная дорожка для ваших внутренних шаблонов мысли, по которой нельзя прийти к истине (или другой потенциально достижимой цели — например, счастью). Как и статистические погрешности, когнитивные искажения могут искривлять наше видение реальности, в большинстве случаев их нельзя исправить, просто собрав побольше данных, и их эффекты со временем суммируются. Но когда неточный инструмент, который нужно откалибровать, это вы сами — избавиться от искажений будет уникальным вызовом.

И тут возникает вполне очевидный вопрос. Если нельзя доверять своему мозгу, то как вообще чему-то можно доверять?

Хорошо бы иметь конкретное название для преодоления когнитивных искажений и вообще ошибок, которыми наши сознания могут себе навредить.

Мы могли бы назвать этот проект как угодно. Я считаю, что на данный момент название «рациональность» прекрасно подходит.

Рациональные чувства

Голливуд считает, что быть «рациональным» — значит суровым и гиперинтеллектуальным. Вспомните Спока из «Звёздного пути» — он «рационально» подавляет эмоции, «рационально» отказывается полагаться на интуицию, но его может легко сбить с толку «иррациональный» оппонент2.

Есть и другое определение «рациональности», которое используют математики, психологи и социологи. Грубо говоря, идея в том, чтобы действовать наилучшим возможным образом. Рациональный человек, даже если он не знаком с темой, формирует максимально близкие к реальности убеждения на основе имеющихся у него свидетельств. Неважно, насколько плохо идут дела у рационального человека — он делает наилучший возможный выбор и таким образом повышает свои шансы на успех.

Реальная рациональность состоит не в том, чтобы игнорировать интуицию и эмоции. Для рационального человека быть рациональным зачастую означает лучше осознать свои чувства и учитывать их при принятии решений.

Рациональность — это и знание о том, когда не нужно слишком долго размышлять. В экспериментах, где нужно было выбрать постер на стену или предсказать исход баскетбольного матча, испытуемые показывали худший результат, если старательно обдумывали своё решение. 3, 4 Есть задачи, решать которые лучше вдумчиво, и есть другие задачи, где лучше принять решение быстро. Психологи, работающие над теорией двух процессов, выделяют процесс «Система 1» (быстрое, неявное, ассоциативное, автоматическое суждение) и процесс «Система 2» (медленное, явное, интеллектуальное, подконтрольное суждение)5. Стереотипный рационалист целиком полагается на Систему 2, игнорируя чувства и импульсы. Если посмотреть не на стереотип, а на того, кто действительно рационален, кто достигает своих целей, кто действительно смягчает урон от когнитивных искажений, можно обнаружить, что он сильно полагается на привычки и интуицию Системы 1, в ситуациях, где на них можно положиться.

К сожалению, Система 1 является плохим помощником, когда надо определить «стоит ли сейчас доверять Системе 1?». Наша нетренированная интуиция не подскажет нам когда мы должны перестать полагаться на нее. Предвзятость и непредвзятость ощущаются одинаково6. С другой стороны, поведенческий экономист Дэн Ариели отмечает: мы предсказуемо иррациональны. Мы запарываемся одними и теми же способами, снова и снова, систематически.

Если мы не можем использовать инстинкт, чтобы понять, когда поддаемся когнитивному искажению, то можно использовать науки о разуме.

Многоликие Ошибки

В процессе эволюции наши мозги для решения задач научились применять когнитивные эвристики — грубые методы, которые позволяют получить ответ быстро. В большинстве случаев этот ответ оказывается правильным, но иногда — нет. Когнитивные искажения возникают в тех случаях, когда эти эвристики достаточно последовательно порождают ярко-выраженные ошибки.

Эвристика репрезентативности, например, является нашей склонностью оценивать феномен, основываясь на том, насколько он характерен по отношению к разным категориям. Это может вести к таким ошибкам, как конъюнктивное заблуждение. Тверский и Канеман обнаружили, что субъекты эксперимента считали менее вероятным, что сильный игрок в теннис «проиграет в первом сете», чем «проиграет в первом сете, но выиграет матч»7. То, что сильный игрок сможет отыграться, кажется более характерным, так что мы переоцениваем вероятность такого сложного-но-так-разумно-звучащего рассказа, по сравнению с вероятностью действительно простого сценария.

Также эвристика репрезентативности может быть частью нечувствительности к априорной вероятности, когда мы основываем наше суждение на том, насколько «нормальна» комбинация атрибутов, пренебрегая тем, насколько распространены атрибуты в популяции в целом8. Стив более вероятно является застенчивым библиотекарем, или застенчивым продавцом? Большинство людей будет думать над этим, анализируя насколько «застенчивость» сочетается с стереотипами этих профессий. Они терпят неудачу в том, чтобы учесть насколько больше продавцов в сравнении с библиотекарями в Соединенных Штатах9.

Среди других примеров искажений есть: игнорирование срока действия (оценка опыта без учета того, как долго он будет переживаться), заблуждение невозвратных затрат (чувство необходимости продолжать заниматься тем, на что уже потрачены силы и время, тогда как следует сжечь мосты и двигаться дальше), и ошибка подтверждения (придавать больше значения подтверждающим наши убеждения свидетельствам)10. Игнорирование вероятности — еще один пример когнитивного искажения. В течение месяцев и лет после атаки 11 сентября, множество людей предпочитали вождение на дальние расстояния полетам. Захват был маловероятен, но теперь чувствовалось, что он был одним из вариантов; всего лишь возможность захвата самолета сильно влияла на решения. Полагаясь на черно-белое мышление (машины и самолеты либо «безопасны» либо «опасны», и точка), люди подвергали себя еще большей опасности. В то время как им следовало взвешивать вероятности умереть в автомобильной аварии против смерти во время перелета через страну — первое гораздо более вероятно; они, вместо этого, полагались на общее чувство беспокойства и тревоги (аффективная эвристика).
Такой же поведенческий шаблон можно наблюдать и у детей, которые, слушая аргументы за и против ремней безопасности, мечутся между мыслями «ремень это хорошо» и «ремень это плохо», вместо взвешивания за и против11. Еще несколько примеров искажений: правило кульминации\завершения (оценка воспоминаний на основе самых интенсивных моментов и того, как они завершились); якорение (принятие решения, основываясь на недавно полученной информации, даже если она не относится к делу)12 и само-якорение (использование самого себя в качестве модели определения вероятных черт, не оценивая внимательно насколько вы атипичны)13; искажение «статус кво» (чрезмерная переоценка пользы нормального и ожидаемого, по сравнению с новым и другим)14.

Знание об искажениях редко может защитить от них. В ходе исследований слепоты по отношению к искажениям, субъекты эксперимента предсказали, что если они будут знать, что оцениваемая картина принадлежит кисти известного художника, им будет сложней нейтрально оценить качества этой картины. И, действительно, субъекты, которым сообщили имя автора картины и попросили оценить ее качество продемонстрировали действие искажения, которого они предсказали, относительно контрольной группы. Когда их опрашивали после, эти же субъекты заявляли, что их оценка картины была объективной и нетронутой искажением — и так во всех группах! 15. По аналогии, Пронин пишет на тему искажения гендерной слепоты:
В одном из исследований, участники оценивали мужские и женские кандидатуры на пост шефа полиции, и потом сравнивали пользу «знания жизни на улице» с «формальным обучением». В результате, участники предпочитали то образование, которое получили мужские кандидаты (т.е. если им было сказано, что он «знает жизнь на улице», то это считали более важным). Участники были слепы по отношению к искажению гендерного восприятия; действительно, чем объективней они себя считали, тем больше признаков искажения демонстрировали16. Даже если мы знаем об искажениях, замечает Пронин, мы остаемся «наивными реалистами», когда речь идет о наших собственных убеждениях. Мы быстро откатываемся на позицию оценки наших убеждений, как свободных от искажений представлений о реальности, такой, какая она на самом деле есть17.

Особенно отвратительна мысль о том, что именно наши убеждения ошибочны. Даже если мы корректно определим ошибки других, для своих у нас есть особое слепое пятно. Как показал опрос, проводившийся в аэропорту среди 76 ожидающих, люди оценивали себя, в среднем, как менее подверженных когнитивным искажениям, чем типичная персона в аэропорту. В частности, люди думали о себе, как о необычайно объективных, когда речь заходила об общественно порицаемых искажениях или искажениях с мало заметными в ближайшем времени последствиями 18. Другое исследование показало, что сильные чувства по отношению к проблеме воспринимались людьми как усиливающие понимание и объективность, если эти чувства испытывали опрашиваемые, если же речь заходила о других, то их, по мнению опрашиваемых, такие чувства делали менее объективными и более предвзятыми. Нам не удается заметить «ошибочно-чувствующиеся мысли», когда мы занимаемся самоанализом, так что мы полагаем себя более объективными, чем все остальные19.

Изучение искажений может сделать вас более уязвимыми для сверхуверенности и ошибки подтверждения, по мере того, как вы будете видеть влияние искажений у окружающих, но не у вас. И слепое пятно, в отличии от остальных искажений, особенно серьезно проявляется среди интеллектуальных, вдумчивых, непредубежденных20, 21.

Это причина для беспокойства.

И все же… Кажется, мы способны на большее. Известно, что мы можем снизить влияние нечувствительности к априорной вероятности, представляя вероятности, как частоты событий. Мы можем минимизировать игнорирование срока действия, уделяя внимание сроку действия и изображая его графически22. Сила проявления различных искажений отличается у разных людей, так что должны существовать способы повлиять на наши заблуждения.

Однако, если мы хотим стать лучше, недостаточно просто создать список когнитивных искажений. Подход к устранению искажений в «Рациональности: от ИИ до Зомби» предполагает системное понимание того, как работает правильное мышление и как мозг отстает от него. Этот подход можно сравнить с описанием Серфаса, который заметил, что «годы работы в финансовом секторе» не влияют на уязвимость к заблуждению невозвратных затрат, тогда как «посещение некоторого количества курсов по бухгалтерскому учету» влияет.

Следовательно, нужно отличать опыт от компетентности, где «компетентность» означает «развитие схематических принципов, включающих концептуальное понимание проблем», и, в свою очередь, позволяет лицу, принимающему решения, распознать конкретное искажение. Однако, использование компетентности в качестве ответной меры требует больше, чем просто быть знакомым с контекстом проблемы или быть экспертом в предметной области. Оно требует полного понимания скрытой подоплеки конкретного искажения, способности заметить его и использовать для его нейтрализации соответствующие инструменты23.

Цель этой книги — заложить фундамент для создания рациональной компетентности. То есть научить глубже понимать структуры очень общих проблем: человеческих ошибок, самообмана, и тысячи способов того, как утонченная мысль может победить сама себя.

Пара слов об этом тексте

«Рациональность: от ИИ до Зомби» родилась в качестве серии эссе Элиезера Юдковского, опубликованных между 2006 и 2009 в экономическом блоге «Overcoming Bias» и его ответвлении, блоге «Less Wrong». Я работал с Юдковским вот уже год в Machine Intelligence Research Institute(MIRI), некоммерческой организации, основанной им в 2000 для исследования теоретических требований к созданию умней-чем-человеческого интеллекта (ИИ).

Читая его посты, я заинтересовался его работой. Он впечатлил меня способностью кратко передать идеи, выработка которых у меня занимала годы изучения аналитической философии. В поиске путей объединения духа науки и скептицизма со строгим и системным подходом к исследованиям, Юдковский старается не только опровергнуть, но и понять ложные шаги и слепые пятна плохой философии (плохой недо-философии). Помогая собрать его эссе в книгу, я надеюсь облегчить погружение в них и понимание их как целого.

Получившийся в результате рациональный букварь полон личных деталей — Юдковский использует свой опыт общения с матерью, ортодоксальной иудейкой (психиатром) и отцом (физиком), разговоров в чате и рассылках. Читатели, знакомые с Юдковским по «Гарри Поттеру и Методам Рационального Мышления», его научно-ориентированной интерпретацией Гарри Поттера Дж. К. Роулинг, узнают элементы непочтительного иконоборчества, и основные концепты.

С точки зрения стилистики, эссе в этой книге представляют гамму от «жизнерадостного учебника» до «сборника вдумчивых виньеток» и «буйного манифеста», контент, соответственно, тоже отличается от эссе к эссе. В книге сотни постов Юдковского из блога собраны в двадцать шесть «цепочек», как бы глав, связанных по смыслу постов. Цепочки же сгруппированы в шесть книг, раскрывающих темы:

Книга 1 - Карта и территория. Что такое убеждение и почему одни убеждения работают лучше других? Эти четыре цепочки объясняют Байесовские понятия рациональности, убеждений и свидетельств. Основная тема: штуки, которые мы называем «объяснениями» и «теориями», не всегда функционируют как карты для навигации по миру. В результате есть риск перепутать наши ментальные карты с другими инструментами.

Книга 2 - Как Действительно Изменить Свое Мнение. Штука, называемая «правдой», похоже полезная. Так почему же мы спешим с выводами, наступаем на те же грабли, повторяя те же ошибки? Почему мы так плохо умеем вырабатывать точные убеждения и как делать это лучше? В этих семи цепочках обсуждаются предвзятые рассуждения и ошибка подтверждения, особый фокус на образцах самообмана, которые трудно заметить, и ловушке «использования аргументов как солдат».

Книга 3 - Машина В Призраке. Почему мы не эволюционировали более рациональными? Даже при условии ограниченных ресурсов, похоже, мы могли бы получать больше эпистемической отдачи от наших свидетельств. Чтобы получить более реалистичную картину того, почему и как наш разум исполняет свои биологические функции, нам нужно залезть под капот и внимательно посмотреть как работает эволюция и наш мозг. Эти три цепочки иллюстрируют, как даже философы и ученые могут заблуждаться, если полагаются на интуитивное, не-техническое понимание эволюционных и психологический идей. Обозначив наши разумы внутри огромной целеполагающей системы, мы можем идентифицировать некоторые основы человеческого рассуждения и понять как эта система может «потерять цель».

Книга 4 - Всего Лишь Реальность. В каком мире мы живем? Каково наше место в нем? Основываясь на примерах работы эволюционных и когнитивных моделей из предыдущих цепочек, в этих шести цепочках мы исследуем природу ума и физического закона. В дополнение к применению и обобщению прошлых уроков о научных тайнах и скупости, эссе этой части поднимают новые вопросы о роли науки в рациональности отдельного индивида.

Книга 5 - Всего Лишь Доброта. Как что-то становится ценным — морально, эстетически или пруденциально? Эти три цепочки задаются вопросами о том, как мы можем оправдать, пересмотреть и принять наши ценности и желания. Цель — понять наши цели, не повредив их достижению. Самая сложная проблема: как понять, когда стоит доверять запутанному, сложному, работающему от случая к случаю импульсивному пониманию «хорошо» и «плохо», а когда заменить его на простые, не ведающие исключений, принципы.

Книга 6 - Становясь Сильней. Как люди и сообщества могут применить все это на практике? Эти три цепочки начинаются с автобиографической истории о самых серьезных философских промахах Юдковского, сочетающейся с советами о том, как другие люди могут избежать подобных. Книга заканчивается рекомендацией по разработке курса обучения прикладной рациональности, основанной (основана рациональность, а не рекомендация (прим. перев.)) на фактических данных. И по формированию групп и институтов для поддержки заинтересованных студентов, педагогов, исследователей и друзей.

Еще цепочки дополнены «интерлюдиями», эссе с личного сайта Юдковского (http://www.yudkowsky.net). Они связаны с разными цепочками, например, «Двенадцать добродетелей рациональности» лирически суммируют множество уроков «Рациональности: от ИИ до Зомби», и часто цитируются в эссе.

Кликнув на звездочку в конце каждого эссе, вы попадете на оригинальную версию с сайта Less Wrong (так можно оставить комментарий) или с сайта Юдковского. Также вы можете обнаружить словарь терминов на http://wiki.lesswrong.com/wiki/RAZ_Glossary.

Карта и территория

Первая книга начинается с цепочки о когнитивных искажениях: «Предсказуемо неправы». Книга не ограничена этой темой: плохие привычки и плохие идеи тоже имеют значение, даже если они становятся следствием содержания нашего разума, а не его структуры. Таким образом, эволюционировавшие и изобретенные будут показаны в цепочках, начиная с дискуссии в «Ложных Убеждениях» о том, как ожидания расходятся с декларированными убеждениями.

Оценка иррациональности была бы неполной без теории, описывающей рациональность, или если «теория» будет состоять из смутных трюизмов, без точного объяснительного механизма. Цепочка «Замечая замешательство» раскрывает вопрос: почему полезно основывать поведение на «рациональных» ожиданиях и как это ощущается изнутри.

Дальше, «Загадочные ответы» ставит вопрос: способна ли наука решить эти проблемы для нас. Ученые основывают свои модели на повторяемых экспериментах, а не на спекуляциях и слухах. Наука также имеет великолепную репутацию в сравнении с историями, религией и… Да в сравнении с чем угодно имеет. Стоит ли волноваться по поводу «ложных» убеждений, ошибки подтверждения, ошибки знания задним числом и им подобных, если мы работаем с сообществом людей, желающих объяснять феномены, а не рассказывать трогательные истории?

За этим следует «Простая Истина», отдельная аллегория о природе знания и убеждений.

Однако именно когнитивные искажения предоставляют наиболее ясное и прямое представление о нашей психологии, виды эвристик и логику наших ограничений. Именно с искажений мы и начнем. Строка в Чжуан-цзы, пре-Даосского философского трактата, гласит: «Ловушка для рыбы существует из-за рыбы, заполучив рыбу, ты можешь позабыть о ловушке» 24.

Приглашаю исследовать эту книгу именно в таком духе. Используйте ее так, как использовали бы ловушку для рыбы, всегда внимательно следя за целью, для которой вы ее используете. Берите то, что можете использовать, то, что будет полезным, остальное — отбросьте. И пусть ваша цель послужит вам верно.

Благодарности

Я чрезвычайно признателен Nate Soares, Elizabeth Tarleton, Paul Crowley, Brienne Strohl, Adam Freese, Helen Toner и десяткам волонтеров, вычитывавших эту книгу.

Отдельно от всей души хочу сказать спасибо Alex Vermeer, который помог довести эту книгу до конца, и Tsvi Benson-Tilsen, который прочел ее от корки до корки, чтобы убедиться в удобочитаемости и связности изложения.

  • 1. Идея персональной ошибки, медиа ошибки, и т.д. похожи на статистическую в том смысле, что это ошибки. По-другому можно обобщить идею искажений, сфокусировавшись на их ассоциации с неслучайностью. В машинном обучении, например, индуктивное искажение— совокупность предположений, которые использует обучающийся для выведения предсказаний на основе имеющихся данных. Т.е. обучающийся предубежден в специфическом направлении, но так как на этом пути может быть правда, не так уж и плохо для агента иметь индуктивное искажение. Оно ценное и необходимое. Это различает индуктивную ошибку от остальных довольно четко.
  • 2. Печальное совпадение: Леонард Нимой, актер, сыгравший Спока, ушел из жизни за несколько дней до выхода этой книги. Несмотря на то, что мы использовали его персонажа как классический пример «Голливудской рациональности», это не значит, что мы не уважаем память о нем.
  • 3. Timothy D. Wilson et al., “Introspecting About Reasons Can Reduce Post-choice Satisfaction,” Personality and Social Psychology Bulletin 19 (1993): 331–331.
  • 4. Jamin Brett Halberstadt and Gary M. Levine, “Effects of Reasons Analysis on the Accuracy of Predicting Basketball Games,” Journal of Applied Social Psychology 29, no. 3 (1999): 517–530.
  • 5. Keith E. Stanovich and Richard F. West, “Individual Differences in Reasoning: Implications for the Rationality Debate?,” Behavioral and Brain Sciences 23, no. 5 (2000): 645–665, http://journals.cambridge.org/abstract_S0140525X00003435.
  • 6. Timothy D. Wilson, David B. Centerbar, and Nancy Brekke, “Mental Contamination and the Debiasing Problem,” in Heuristics and Biases: The Psychology of Intuitive Judgment, ed. Thomas Gilovich, Dale Griffin, and Daniel Kahneman (Cambridge University Press, 2002).
  • 7. Amos Tversky and Daniel Kahneman, “Extensional Versus Intuitive Reasoning: The Conjunction Fallacy in Probability Judgment,” Psychological Review 90, no. 4 (1983): 293–315, doi:10.1037/0033-295X.90.4.293.
  • 8. Richards J. Heuer, Psychology of Intelligence Analysis (Center for the Study of Intelligence, Central Intelligence Agency, 1999).
  • 9. Wayne Weiten, Psychology: Themes and Variations, Briefer Version, Eighth Edition (Cengage Learning, 2010).
  • 10. Raymond S. Nickerson, “Confirmation Bias: A Ubiquitous Phenomenon in Many Guises,” Review of General Psychology 2, no. 2 (1998): 175.
  • 11. Cass R. Sunstein, “Probability Neglect: Emotions, Worst Cases, and Law,” Yale Law Journal (2002): 61–107.
  • 12. Dan Ariely, Predictably Irrational: The Hidden Forces That Shape Our Decisions (HarperCollins, 2008).
  • 13. Boaz Keysar and Dale J. Barr, “Self-Anchoring in Conversation: Why Language Users Do Not Do What They ‘Should,”’ in Heuristics and Biases: The Psychology of Intuitive Judgment: The Psychology of Intuitive Judgment, ed. Griffin Gilovich and Daniel Kahneman (New York: Cambridge University Press, 2002), 150–166, doi:10.2277/0521796792.
  • 14. Scott Eidelman and Christian S. Crandall, “Bias in Favor of the Status Quo,” Social and Personality Psychology Compass 6, no. 3 (2012): 270–281.
  • 15. Katherine Hansen et al., “People Claim Objectivity After Knowingly Using Biased Strategies,” Personality and Social Psychology Bulletin 40, no. 6 (2014): 691–699.
  • 16. Eric Luis Uhlmann and Geoffrey L. Cohen, “‘I think it, therefore it’s true’: Effects of Self-perceived Objectivity on Hiring Discrimination,” Organizational Behavior and Human Decision Processes 104, no. 2 (2007): 207–223.
  • 17. Emily Pronin, “How We See Ourselves and How We See Others,” Science 320 (2008): 1177–1180, http://psych.princeton.edu/psychology/research/pronin/pubs/2008%20Self%2….
  • 18. Emily Pronin, Daniel Y. Lin, and Lee Ross, “The Bias Blind Spot: Perceptions of Bias in Self versus Others,” Personality and Social Psychology Bulletin 28, no. 3 (2002): 369–381.
  • 19. Joyce Ehrlinger, Thomas Gilovich, and Lee Ross, “Peering Into the Bias Blind Spot: People’s Assessments of Bias in Themselves and Others,” Personality and Social Psychology Bulletin 31, no. 5 (2005): 680–692.
  • 20. Richard F. West, Russell J. Meserve, and Keith E. Stanovich, “Cognitive Sophistication Does Not Attenuate the Bias Blind Spot,” Journal of Personality and Social Psychology 103, no. 3 (2012): 506.
  • 21. Не путать с людьми, думающими о себе как о интеллектуальных, из-за эффекта мнимого превосходства.
  • 22. Michael J. Liersch and Craig R. M. McKenzie, “Duration Neglect by Numbers and Its Elimination by Graphs,” Organizational Behavior and Human Decision Processes 108, no. 2 (2009): 303–314.
  • 23. Sebastian Serfas, Cognitive Biases in the Capital Investment Context: Theoretical Considerations and Empirical Experiments on Violations of Normative Rationality (Springer, 2010).
  • 24. Zhuangzi and Burton Watson, The Complete Works of Zhuangzi (Columbia University Press, 1968).
Перевод: 
Muyyd, Remlin
Оцените качество перевода: 
Средняя оценка: 4.7 (23 votes)

Предсказуемо неправы

Материалы цепочки распространяются по лицензии CC BY-NC-SA 3.0

Автор: 
Элиезер Юдковский

Что такое рациональность

Элиезер Юдковский

Под рациональностью я подразумеваю:

  1. Эпистемическую рациональность, то есть систематическое улучшение точности своих убеждений.
  2. Инструментальную рациональность, то есть систематическое достижение желаемых результатов.

Когда вы открываете глаза и осматриваете комнату, вы замечаете ноутбук на столе и книжный шкаф около стены. Если с вашими глазами или вашим мозгом случится что-нибудь плохое, возможно, у вас в голове окажется мысленная модель, которая скажет, что шкаф стоит там, где его на самом деле нет. В этом случае, когда вы пойдёте за книгой, вам будет обидно.

Примерно так и выглядят ложные убеждения — карта мира, не соответствующая территории1. Эпистемическая рациональность посвящена построению карт, точно отображающих реальность. Точную связь между убеждением и реальностью обычно называют «истиной», и я тоже с радостью пользуюсь этим словом.

С другой стороны, инструментальная рациональность посвящена управлению реальностью. Вы делаете будущее таким, каким вам хочется. Инструментальная рациональность — это искусство выбрать действия, которые приведут к последствиям, стоящим выше в вашей шкале предпочтений. Иногда я называю это «выигрыванием».

Так что рациональность — это искусство формировать истинные убеждения и принимать выигрышные решения.

Поиск «истины» не означает, что нужно отвергать нечёткие или косвенные свидетельства. Вы можете оглядеть комнату и построить её мысленную модель. Убеждения о комнате принципиально ничем не отличаются от убеждений, что у Земли есть расплавленное ядро, а Юлий Цезарь был лысым. Возможно, эти вопросы вам кажутся более сложными и абстрактными, чем вопрос о вашем шкафе, ведь вас и предметы этих вопросов разделяет значительное пространство и время. И тем не менее состояние ядра Земли в 2015 году нашей эры и состояние головы Цезаря в 50 году до нашей эры — это вопросы о фактах. Эти факты могут влиять на реальность, даже если у вас нет никакой возможности лично встретить Цезаря или посмотреть на земное ядро.

А когда я пишу о «выигрывании», это не значит, что речь идёт о достижении успеха за счёт других. Не обязательно превращать жизнь в соревнование, её можно строить на базе сотрудничества и самопожертвования. Под «вашими ценностями» подразумевается абсолютно всё, о чём вы заботитесь — включая других людей. Они не ограничиваются исключительно эгоистическими ценностями.

Зачастую, когда люди говорят «Х — рационально!», это просто более звучный способ сказать: «Я думаю, что Х — истинно» или «Я считаю, что Х — хорошо!». Так зачем же использовать дополнительное слово «рационально», если можно говорить «истинно» и «хорошо»?

Тот же самый аргумент применим и к слову «истинно». Незачем говорить «истинно, что снег белый», когда можно сказать просто «снег белый». Идея истины полезна, потому что она позволяет нам говорить о соответствии карты и территории в целом. Фраза «истинные модели позволяют лучше делать проверяемые предсказания, чем ложные модели» — это полезное обобщение, и его нельзя сделать, не пользуясь понятиями вроде «истины» и «точности».

Аналогично, фраза «Рациональные агенты принимают решения, которые максимизируют вероятностное ожидание согласованной функции полезности» относится к понятию (инструментальной) рациональности, а фразу «Рационально есть овощи» наверняка можно заменить на «Полезно есть овощи» или «Есть овощи — в ваших интересах». Нам нужно понятие, вроде «рациональности», чтобы описывать общие факты о том, какие способы мышления систематически приводят нас к истине или к выигрышу, а также какие способы систематически приводят нас к обратному.

Иногда экспериментальные психологи обнаруживают, что в некоторых ситуациях люди, размышляют, судя по всему, очень странно. Например, человек присваивает утверждению «Билл играет джаз» вероятность меньшую, чем утверждению «Билл — бухгалтер, который играет джаз». Поскольку, очевидно, любой играющий джаз бухгалтер — играет джаз, такое суждение выглядит странно. Однако какими принципами мы пользуемся, когда говорим, что это суждение неверно?

Экспериментальные психологи применяют два золотых стандарта: теория вероятностей и теория принятия решений.

Теория вероятностей — это набор законов, лежащих в основе рациональных убеждений. Математика теории вероятностей абсолютно одинаково описывает и а) как выяснить, где находится ваш книжный шкаф, и б) как узнать температуру земного ядра, и в) как вычислить, сколько волос было на голове Юлия Цезаря. С точки зрения того, как нужно обрабатывать свидетельства и наблюдения, чтобы пересматривать и обновлять свои убеждения, это одинаковые задачи. Аналогично, теория принятия решений — это набор законов, лежащих в основе рациональных действий, и её можно применять независимо от того, о каких целях и каких вариантах действий идёт речь.

Обозначим выражением «Р(что-нибудь)» вероятность того, что это что-нибудь случится, а выражением «Р(А, В)» — вероятность, что А и В случатся вместе. Поскольку в теории вероятностей есть универсальный закон, что P(A)P(A, B), суждение, что Р(Билл играет джаз) меньше, чем Р(Билл играет джаз, Билл — бухгалтер) отмечается как неверное.

Говоря формально, такое рассуждение о вероятностях — небайесианское. Убеждения и действия, рациональные в строго определённом математическом смысле, называются байесианскими.

Обратите внимание, современное понятие рациональности не привязано к словесным рассуждениям. Я привёл пример, как вы открываете глаза, осматриваетесь и строите мысленную модель комнаты со шкафом около стены. Современное понятие рациональности — достаточно общее и включает ваши глаза и зрительную кору вашего мозга, как объекты на карте. Оно включает и вашу интуицию, которую сложно выразить словами. Математика не обращает внимание на то, что мы используем одинаковое слово «рациональный», и описывая Спока2, и описывая байесианский способ рассуждений. Математические модели — это хороший способ достигать своих целей или строить карту мира, независимо от того, какие у нас сложились предубеждения и стереотипы относительно слова «рациональность».

Однако это не решает проблему о том, что именно подразумевается под занятием «рациональностью», по двум основным причинам.

Во-первых, невозможно строго вычислить байесовские вероятности, необходимые для решения подавляющего большинства реальных задач. Никто не в состоянии посчитать правильное решение и ему следовать. Это всё равно, что попытаться предсказать поведение рынка акций, исходя из движения кварков.

Именно поэтому существует целый сайт под названием «Less Wrong», а не одна страница, которая ограничивается простым перечислением строгих аксиом. Для человеческого разума находить истину и достигать своих целей — это целое искусство. Чтобы сталкиваться с правдой и делать то, что должно, нам надо изучать собственные недостатки, справляться со своими предубеждениями, удерживаться от самообмана, поддерживать себя в хорошей эмоциональной форме, и так далее, и так далее.

Во-вторых, иногда математика сама по себе приводит к новым вопросам. Например, точные правила теории вероятностей приводят к вопросу антропного принципа, где неизвестно число наблюдателей. Точные правила теории принятия решений приводят к задачам вроде парадокса Ньюкома, в которых другие агенты могут предсказать ваше решение до того, как вы его примете.

В таких случаях совершенно бессмысленно пытаться решить проблему, введя новое определение слова «рациональность» и заявив: «Таким образом под «рациональностью», по определению, будет подразумеваться предпочитаемый мной ответ». В этом случае, естественно, встанет вопрос, почему ваше определение должно кого-то интересовать. Теория вероятностей важна для меня не потому, что это священные слова, вручённые нам Лапласом. Байесианское обновление убеждений (вместе с оккамовскими априорными вероятностями) интересно для меня, потому что я ожидаю, что именно такой способ думать приведёт нас к карте, которая систематически более точно отображает территорию.

А ещё некоторые вопросы о том, как следует думать, кажется, не решаются целиком ни с помощью теории вероятностей, ни с помощью теории принятия решений. Например, вопрос о том, какие чувства испытывать по отношению к правде. В этих случаях какое-то собственное определение «рациональности» тоже лишь будет включать в себя уже существующий ответ, но не позволит найти что-то новое.

Я не собираюсь здесь спорить о значении слов, даже если речь идёт о слове «рациональность». Привязка неких последовательностей букв к определённым понятиям нужна, чтобы люди могли разговаривать между собой — передавать мысли от одного человека к другому. Невозможно изменить реальность или доказать какую-то мысль, манипулируя этими привязками.

Поэтому если вы поняли, какое понятие я связываю со словом «рациональность» и со словосочетаниями «эпистемическая рациональность» и «инструментальная рациональность», передача информации уже случилась. В рамках разговора о том, как определять «рациональность», мы сделали всё, что нужно. Дальше мы не будем разговаривать, какой смысл привязан к слогам «ра-ци-о-наль-ность». Дальше мы будем разговаривать, как именно лучше рассуждать.

Если вы говорите «Для меня (эпистемически) рационально считать, что Х, однако истина в том, что Y», вы наверняка используете слово «рационально» не в том же смысле, что и я. (Например, «рациональность» должна «выдерживать рефлексию» — «рациональный» взгляд на свидетельства и «рациональные» размышления о том, как ваш разум обрабатывает эти свидетельства, не должны приводить к разным выводам.)

Аналогично, если вы вдруг говорите «Для меня (инструментально) рационально сделать Х, но я считаю, что правильно делать Y», то почти наверняка вы используете для слова «рационально» или для слова «правильно» какие-то другие значения. Я использую термин «рациональность» нормативно: чтобы выбирать предпочитаемый способ рассуждений.

В этих случаях — а также во всех других, когда люди не соглашаются относительно смысла слов — вам стоит заменять слово «рационально» на более точные описания. Например: «Ради собственного благополучия следовало бы убежать, но я надеюсь, что хотя бы постараюсь вытащить ребёнка с рельсов». Или: «Причинная теория приятия решений в привычной формулировке говорит, что в задаче Ньюкома нужно брать оба ящика, но я предпочту получить миллион долларов».

Я вообще рекомендую перечитать это эссе, заменяя все вхождения слова «рационально»(и однокоренные) на «фузально». Проверьте, изменится ли смысл текста. Если изменится, стремитесь не к рациональности, а к фузальности.

В слове «рационально» таятся многочисленные ловушки. Однако во многих случаях с его помощью можно прекрасно передавать тот смысл, который я в него вкладываю. В этих случаях я не боюсь его использовать.

И всё же будьте с этим словом осторожны и не используйте его напрасно. Вы ничего не заработаете, просто произнося его вслух. Слишком много рассуждающий о пути не достигнет просветления.

  • 1. Фраза «Карта — это не территория» является одним из ключевых положений дисциплины под названием «общая семантика». В эссе Rationalist Fiction Юдковский рассказывает, что ещё в детстве прочитал эту фразу в научно-фантастическом романе «Мир Нуль-А» Альфреда ван Вогта. Примечательно, что в данной книге Юдковский ни разу не упоминает ни общую семантику, ни её главного разработчика – Альфреда Коржибски — Прим. перев.
  • 2. Известный персонаж сериала «Звёздный путь», . — Прим.перев.
Перевод: 
Alaric
Номер в книге "Рациональность: от ИИ до зомби": 
1
Оцените качество перевода: 
Средняя оценка: 3.6 (34 votes)

Мне сегодня рационально

Элиезер Юдковский

Есть распространённое мнение, что рациональность противостоит эмоциям: все оттенки грусти и все оттенки радости автоматически нелогичны лишь потому, что это всего лишь чувства. Что удивительно, я не нашел ни одной теоремы в теории вероятностей, которая бы доказывала, что я должен быть холодным и бесчувственным.

Выходит, рациональность вообще не имеет отношения к чувствам? Это не так, ведь наши эмоции порождаются нашими моделями реальности. Если я решу, что мой мёртвый брат воскрес, я обрадуюсь; если я проснусь и пойму, что это был сон, я буду расстроен. Пэт Ходжилл однажды сказала: «То, что может быть разрушено правдой, должно быть разрушено». Во сне я был счастлив, но правда этому счастью противоречила. Проснувшись, я ощутил грусть, но эта грусть рациональна: нет такой правды, которая могла бы её разрушить.

Рациональность начинается с вопроса «каков мир на самом деле и как он работает», но словно вирус, распространяется на любую другую мысль, зависящую от того, каков он, по нашим представлениям. Убеждения о том, каков мир на самом деле, могут включать все, что ты думаешь о реальности, что существует, а что нет, и что угодно из класса «вещи, которые могут заставить случаться другие вещи». Если ты считаешь, что в твоем чулане живёт гоблин, который привязывает друг к другу шнурки твоих ботинок, то это — убеждение о том, как работает мир. Шнурки реальны: их можно потрогать. Сущность, которая может их связать, тоже должна быть реальной: она — часть гигантской сети причин и следствий, которую мы называем «вселенной».

Чувство злости на гоблина, за то что он связал шнурки - это состояние разума, которое касается не только того, как работает мир. Если ты, скажем, буддист, или тебе сделали лоботомию, или ты просто родился очень флегматичным человеком, то действия гоблина тебя не разозлят. Это не влияет на твои ожидания: ты все еще уверен, что открыв чулан, увидишь пару ботинок, привязанных друг к другу шнурками. Зол ты или спокоен - это не должно влиять на твои прогнозы, поскольку твои эмоции не влияют на происходящее в чулане (хотя, конечно, может потребоваться некоторое усилие, чтобы рассуждать трезво).

Однако, злость сцеплена с состоянием разума, имеющим отношение к тому, как работает мир: ты злишься потому, что думаешь, что гоблин связал шнурки ботинок. Критерий рациональности рапространяется как вирус: от исходного вопроса, завязал ли гоблин шнурки, и до возникшей в итоге злости.

По мере улучшения представлений о том, каков мир на самом деле, эмоции могут как ослабнуть, так и усилиться. Иногда мы избегаем сильных эмоций, отрицая факты, прячась от мира, порождающего столь сильные эмоции. В таком случае, изучая искусство рациональности и тренируясь не отрицать факты, ты можешь заметить усиление эмоций.

Когда я начинал учиться рациональности, я постоянно сомневался, нормально ли испытывать сильные эмоции, допустимо ли это вообще, и правильно ли. Не думаю, что замешательство было вызвано только лишь моим неверным представлением о рациональности. Я встречал немало людей с подобными проблемами, и эти люди даже и не думали становиться рационалистами. Когда они были счастливы, они сомневались в своём праве быть счастливыми, а когда расстроены - не могли понять, нужно ли бороться с этой эмоцией. Ещё со времён Сократа (а возможно, и гораздо раньше) в кругу людей, считавших себя культурными и утончёнными, было не принято показывать, что вам что-то небезразлично. Испытывать чувства постыдно — в приличном обществе так просто не делают. Надо видеть взгляды, которые на меня бросают люди, узнав, насколько мне важна рациональность. И я подозреваю, что причина кроется не в необычности предмета интереса, а в том, что они не привыкли встречать здравомыслящих взрослых людей, которые демонстрируют, что им важно хоть что-то.

Но сейчас я знаю, что нет ничего постыдного в сильных эмоциях. Усвоив правило «То, что может быть разрушено правдой, должно быть разрушено», я также понял, что «То, что питается правдой, должно расцветать». Когда случается что-то хорошее, я счастлив, и не мучаюсь сомнениями, рационально ли быть счастливым. Когда случается что-то ужасное(English), я не бегу от печали, ища ложные утешения и фальшивые плюсы. Я представляю прошлое и будущее человечества, десятки миллиардов смертей на протяжении всей истории, горе и страх, поиск ответов, дрожащие руки, тянущиеся вверх из рек крови, то, чем мы можем стать в тот день, когда звёзды станут нашими городами, вся эта тьма и весь этот свет; я знаю, что я никогда не смогу полностью это понять, и я не знаю слов, которыми можно было бы передать эти мысли. Несмотря на всю мою философию, я всё ещё смущаюсь показывать сильные эмоции, а тебе, возможно, некомфортно видеть их проявления. Но теперь я знаю, что в чувствах нет ничего иррационального.

Перевод: 
BT
Номер в книге "Рациональность: от ИИ до зомби": 
2
Оцените качество перевода: 
Средняя оценка: 4.2 (53 votes)

Зачем нужна истина?

Элиезер Юдковский

Некоторые комментаторы спрашивают меня, зачем необходимо искать истину (к счастью, почти никому не нужно объяснять, что есть истина). Желание сделать своё мировоззрение рациональным вырастает именно из стремления к истине, и уже благодаря этому желанию все мировоззрения можно разделить на «хорошие» и «плохие».

В «Двенадцати добродетелях рациональности» я писал: «Первая добродетель — любопытство». Любопытство — первая причина искать истину, и, несмотря на то, что эта причина не единственна, в ней есть особая восхитительная чистота. В глазах человека, движимого любопытством, приоритет вопроса зависит от его эстетической ценности. Сложный вопрос, где вероятность неудачи необычно высока, стоит больших усилий, чем простой, где ответ и без того ясен — ведь узнавать новое интересно.

Кто-то может возразить: «Любопытство — эмоция, а эмоции иррациональны». Я называю эмоцию «иррациональной», если она основана на ложных убеждениях или, точнее, поведении, неправильном в свете известной информации: «К твоему лицу подносят железо, и ты веришь, что оно раскалено, но можно видеть, что оно холодно — тогда Учение осуждает твой страх. К твоему лицу подносят железо, и ты веришь, что оно холодно, но можно видеть, что оно раскалено — тогда Учение осуждает твоё спокойствие». И наоборот: эмоцию, вызванную истинными убеждениями либо рациональным с точки зрения желания узнать истину мышлением можно назвать «рациональной эмоцией» (Поэтому удобно считать, что спокойствие — не абсолютный ноль шкалы, а тоже эмоция, не лучше и не хуже всех остальных).

Мне кажется, что люди, противопоставляющие «эмоции» и «рациональность», на самом деле говорят о Системе 1 — системе быстрых, основанных на восприятии суждений — и Системе 2 — системе медленных обоснованных суждений. Обоснованные суждения не всегда правдивы и интуитивные суждения не всегда ложны, поэтому важно не путать эту дихотомию с вопросом о рациональности и иррациональности. Обе системы могут служить как истине, так и самообману.

Что ещё заставляет искать истину, не считая любопытства? Желание достигнуть какой-то цели в реальном мире: например, братья Райт хотят построить самолёт и для этого им необходимо знать правду о законах аэродинамики. Или, более повседневно: я хочу шоколадного молока, и поэтому меня интересует, можно ли купить его в ближайшем магазине: тогда я смогу решить, пойти ли туда, или куда-нибудь ещё. В глазах человека, движимого прагматизмом, приоритет вопроса определяется ожидаемой полезностью ответа на него: степенью влияния на решения, важностью этих решений, вероятностью того, что ответ сместит итоговое решение в сторону от первоначального решения.

Поиск истины в прагматичных целях выглядит неблагородным — разве истина не ценна сама по себе? — но такие поиски очень важны, поскольку они создают внешний критерий проверки. Упавший на землю самолёт или отсутствие молока в магазине говорят о том, что ты сделал что-то неправильно. Ты получаешь обратную связь и можешь понять, какие методы мышления работают, а какие нет. Чистое любопытство прекрасно, но стоит найти ответ — оно исчезает вместе с поразительной загадкой, и ничто уже не заставляет проверять ответы. Любопытство — древняя эмоция, появившаяся задолго до древних греков, руководившая ещё предками их предков. Но легенды о богах и героях удовлетворяют любопытство ничуть не хуже результатов научных экспериментов, и очень долго никто не видел в этом ничего плохого. Лишь наблюдение «некоторые методы мышления отыскивают суждения, позволяющие управлять миром» уверенно направило человечество на путь науки.

Итак, есть любопытство, есть прагматизм, что ещё? Третья причина поиска истины, которая приходит мне в голову — честь. Вера в то, что отыскание истины благородно, нравственно и важно. Такой идеал приписывает истине внутреннюю ценность, но он не похож на любопытство. Мысль «Интересно, что за занавесом» ощущается не так, как мысль «Мой долг — заглянуть за занавес». Паладину истины легче верить в то, что за занавес должен заглянуть кто-то ещё, и легче осуждать кого-то за добровольное закрывание глаз. Из этих соображений я называю «честью» убеждение в том, что истина имеет практическую ценность для общества и поэтому её следует искать всем. Приоритеты паладина истины в отношении белых пятен карты определяются не полезностью и не интересностью, но важностью; кроме того, в одних ситуациях долг искать истину зовёт сильнее, чем в других.

Я с подозрением отношусь к долгу как мотивации для поиска истины: не потому, что идеал плох сам по себе, а потому, что из такого мировоззрения могут вытекать некоторые проблемы. Слишком легко приобрести в корне ошибочные методы мышления. Например, посмотрим на наивный архетип рациональности — мистера Спока из «Звёздного пути». Эмоциональное состояние Спока всегда зафиксировано на отметке «спокойствие», даже когда это совершенно неадекватно ситуации. Он часто сообщает чудовищно неоткалиброванные вероятности, называя при этом слишком много значащих цифр («Капитан! Если ты отправишь Энтерпрайз в эту чёрную дыру, то вероятность нашего выживания всего лишь 2,234%!» — и при этом в девяти случаях из десяти Энтерпрайз отделывается мелкими царапинами. Оценка отличается от реального значения на два порядка; каким идиотом надо быть, чтобы раз за разом называть четыре значащие цифры?). Но при этом многие люди, думая о «долге быть рациональным» представляют себе в качестве примера именно Спока — неудивительно, что они не принимают искренне такой идеал.

Если сделать рациональность моральным долгом, то она теряет все степени свободы и превращается в деспотичный первобытный обычай. Получившие неверный ответ люди возмущённо утверждают, что они действовали в точности по правилам, вместо того, чтобы учиться на ошибках.

Но всё же, если мы желаем стать более рациональными, чем наши предки охотники и собиратели, то нам необходимы обоснованные убеждения о том, как правильно мыслить. Написанные нами ментальные программы рождаются в Системе 2, системе медленных обдуманных решений, и очень медленно переселяются — если вообще переселяются — в цепи и сети нейронов, образующих Систему 1. Поэтому, если мы желаем избежать некоторых определённых типов рассуждений, — например, когнитивных искажений — то это желание остаётся внутри Системы 2 в качестве предписания сторониться нежелательных мыслей, превращается в своеобразный профессиональный долг.

Некоторые методы мышления помогают найти истину лучше, чем другие — это приёмы рациональности. Часть приёмов рациональности говорят о преодолении определённого класса препятствий, когнитивных искажений.

Перевод: 
BT
Номер в книге "Рациональность: от ИИ до зомби": 
3
Оцените качество перевода: 
Средняя оценка: 4.4 (23 votes)

Что же такое искажение?

Элиезер Юдковский

Искажения — определённый вид препятствий на пути к истине (то есть их характер как препятствие мешает найти истину, но при этом существует множество других препятствий, искажениями не являющихся).

Если спросить «что такое искажение?» прямо сейчас, то вопрос будет задан слишком рано. Как говорится в английской поговорке, «существует сорок видов безумия, но лишь один вид здравого смысла». Истина — небольшая цель, компактная область в пространстве возможных конфигураций. «Любит она меня или нет?» — простой общий вопрос, однако $E=mc^2$ — всего лишь маленькая точка в пространстве всех возможных уравнений, словно выигрышный лотерейный билет в пространстве всех лотерейных билетов. В ошибке нет ничего исключительного, объяснения требует успех — настолько он невероятен априори.

Моральный долг «бороться с искажениями» (потому что искажения — плохие, злобные и Просто Неправильные штуки) — неверный подход к проблеме. Так может думать кто-то, кто приобрёл деонтологический долг «быть рациональным» в результате социального осмоса. Это порождает людей, пытающихся применять техники, не понимая стоящих за техниками причин (а это плохая, злобная и Просто Неправильная штука, если верить замечательной книге «Вы, конечно, шутите, мистер Фейнман»).

Подойдём к проблеме правильно.

По какой-то причине мы желаем попасть к истине, и по пути нам встречаются различные препятствия. Эти препятствия не сильно отличаются друг от друга: например, есть препятствия, имеющие отношение к недостатку вычислительных мощностей или к дороговизне информации. Большая группа препятствий разделяет общие черты — образует кластер в пространстве препятствий-на-пути-к-истине — и члены этого кластера называются искажениями.

Что такое искажение? Можно ли найти простой тест на принадлежность к этому эмпирическому кластеру? Может быть, мы так и не сможем найти объяснения, лучшего чем «показать пальцем на несколько ярких примеров и надеяться, что слушатель поймёт». Порой для учёного, только начавшего изучать огонь, мудрее сказать «огонь — это вон та яркая оранжевая штука», чем «я определяю огонь как алхимическую трансмутацию материи, выделяющую флогистон». Как я сказал в «Простой истине», нельзя игнорировать что-то лишь потому, что ты не знаешь, как это определить. Я не помню уравнения Общей Теории Относительности наизусть, но тем не менее, если я шагну с обрыва, то я упаду. То же самое можно сказать и про искажения — они не перестанут больно кусаться, если выяснится, что никто не может внятно объяснить, что такое «искажение». Поэтому вполне законно рассказать про ошибки в ситуации с логическим «И», эффект знания задним числом, сверхуверенность, эвристику доступности, ошибочную оценку поведения и гиперболическое обесценивание, а после сказать «Что-то типа этого».

Можно заметить, что мы называем «искажениями» те препятствия на пути к истине, которые созданы ни дороговизной информации, ни недостатком вычислительных мощностей, но формой ментального оборудования. Например, эволюция оптимизировала оборудование для целей, противостоящих эпистемологической точности (оборудование выигрывать политические споры). Или давление естественного отбора извратило желание добиться эпистемологической точности (вместо этого люди верят в то же, что и окружающие, приобретая благодаря этому союзников и друзей). Или классическая проблема эвристики, когда оборудование работает по понятному алгоритму, который обычно делает всё правильно, но временами совершает систематические ошибки. Сама по себе, эвристика доступности не является искажением, но она порождает характерные, компактно описываемые искажения. Человеческий мозг делает что-то неправильно, и проведя кучу экспериментов и/или хорошо подумав, кто-нибудь формулирует проблему на понятном Системе 2 языке — тогда мы называем эту ошибку «искажением». Даже если мы так и не узнаем большего, всё же ясно, что эта ошибка возникает — известным образом — из-за определённой формы когнитивного оборудования. Не из-за того, что оборудования слишком мало, а из-за того, что оно особой формы.

«Искажения» — это не ошибки, порождённые информацией внутри мозга (например, перенятые у кого-то убеждения или некого рода моральный долг). Они не называются «искажениями», и их намного проще исправить после того, как они замечены (тем не менее, источником ошибки, или источником источника ошибки, может быть и какое-либо искажение).

«Искажения» — это не ошибки, вызванные каким-то индивидуальным повреждением мозга или впитанными культурными нравами. «Искажения» — следствие работы механизмов, общих для всех людей.

Нельзя сказать «Платон страдал от когнитивных искажений, ведь он не знал теории относительности» — он не мог о ней узнать, его незнание было порождено не формой его ментального оборудования. Но если Платон считал, что лучшими правителями будут философы, из-за того что он сам был философом — а это убеждение, в свою очередь, возникло благодаря универсальному политическому инстинкту рекламировать себя, а не из-за того, что отец Платона говорил «долг каждого — говорить, что его профессия достойна занимать трон правителя», и не из-за того, что в детстве Платон нюхал слишком много клея — то тогда это значит, что Платон страдал от когнитивных искажений, и не имеет значения, знал ли об этом сам Платон.

Возможно, искажения трудно исправить. Возможно, они не исправляются в принципе. Но дело не в этом. Если, взглянув на своё ментальное оборудование, мы видим заурядный пример известного класса ошибок, и эта ошибка возникает из-за созданной эволюцией формы оборудования, а не из-за недостатка оборудования или плохого его содержимого, то мы называем её искажением.

Лично я вижу цель оттачивания навыков рационалиста в усилении способностей находить истину. Задание состоит в том, чтобы добраться до позитивной отметки (истины), а не в том, чтобы избежать негативной отметки (неудачи). Пространство неудач велико, там найдётся бесконечное множество ошибок в бесконечном их разнообразии. Трудно описать столь обширное пространство: «Свойство этого яблока может не быть свойством того яблока. Поэтому про одно яблоко можно рассказать большее, чем про все яблоки в мире». Пространство успеха меньше, и поэтому про него можно рассказать больше.

Я не питаю отвращения к разговорам об определениях, но нельзя забывать, что это не главная наша цель. Мы преследуем великую цель нахождения истины, нам отчаянно нужны знания, и кроме того, мы любопытны. Поэтому мы стремимся преодолеть все лежащие на пути препятствия, и без разницы, как мы их при этом называем.

Перевод: 
BT
Номер в книге "Рациональность: от ИИ до зомби": 
4
Оцените качество перевода: 
Средняя оценка: 4.2 (32 votes)

Доступность

Элиезер Юдковский

Эвристика доступности - суждение о частоте или вероятности события по тому, насколько легко приходят на ум примеры данного события.

В известном исследовании 1978 года «Оценка частоты смертельных исходов»1 изучались ошибки при количественной оценке серьёзности рисков. Испытуемым называли два бедствия и спрашивали, какое из них происходит чаще. Испытуемые считали, что несчастные случаи уносят столько же жизней, сколько и болезни, думали, что убийство является более частой причиной смерти, нежели самоубийство. Хотя на самом деле, число умерших от болезней в шестнадцать раз превосходит число погибших от несчастного случая, а самоубийства происходят в два раза чаще убийств.

Очевидная гипотеза, объясняющая эти искажённые убеждения - что об убийствах чаще говорят, нежели о самоубийствах, и, таким образом, людям проще вспомнить разговор об убийце, нежели о суициднике. Несчастные случаи производят большее впечатление, нежели болезни - возможно, поэтому людям проще запомнить или вспомнить несчастный случай. В 1979 году, следующее исследование Комбса и Словика показало, что суждения с искажённой вероятностью сильно коррелируют (0.85 и 0.89) с искажёнными цифрами, которые были размещены в двух газетах2. Хотя это не проясняет, легче ли вспомнить убийства, потому что о них больше пишут, или же репортёры больше пишут об убийствах, потому что убийства производят большее впечатление (и поэтому легко запоминаются).

Но так или иначе, эвристика доступности тут присутствует. Избирательная отчётность - это один из основных источников искажений доступности. В родоплеменном окружении большая часть ваших знаний была основана на вашем личном опыте или же услышана напрямую от члена племени, который видел это. То есть между вами и фактом был максимум один слой избирательного сообщения. Сегодня, при помощи интернета, вы можете увидеть сообщения, которые проходят через шесть и более рук по пути к вам - шесть последовательных фильтров. По сравнению с нашими предками, мы живём в большем мире, в котором происходит больше событий, информации о которых к нам доходит меньше, поэтому эффект избирательности куда сильнее, что создает большие искажения доступности.

В реальной жизни, едва ли вы встречались с Биллом Гейтсом. Однако благодаря избирательным сообщениям от СМИ, у вас может появиться искушение сравнивать ваш жизненный успех с его - и страдать, соответственно, от полученного результата. Объективная частота встречи таких людей как Билл Гейтс - 0.00000000015, но слышите вы о нем куда чаще. И наоборот, 19% планеты живёт менее чем на один доллар в день, но я сомневаюсь, что хотя бы одна пятая тех постов, что вы сегодня читали, пишут о них.

Использование доступности, похоже, отвечает и за искажение абсурдности; события, которые никогда не происходили, нельзя вспомнить, вследствие чего их вероятность наступления начинает считаться нулевой. Если давно не было наводнений (а вероятности все еще считаются правильно), люди отказываются покупать страховку от наводнения, даже когда стоит она не больше, чем должна, а выплаты по ней немалые. Кунрейтер и соавторы в своём исследовании3 предполагают, что недооценка угрозы наводнения может происходить от «неспособности людей осмыслить концепцию наводнений, которых никогда не было…люди на затапливаемых равнинах, похоже, являются узниками своего опыта…недавно пережившие наводнение имеют тенденцию привязываться к верхней границе потерь, которую потом считают величиной, о которой и следует задумываться».

В исследовании Бёртона и соавторов4 говорится, что когда строятся дамбы и плотины, они уменьшают частоту наводнений, и, видимо, создают ложное ощущение безопасности, приводя к снижению мер безопасности, в то время как постройка дамб уменьшает частоту наводнений, но увеличивает ущерб от тех, что все же могут произойти (4). Мудрый человек экстраполировал бы из памяти о небольших угрозах возможность больши́х. Но вместо этого, прошлый опыт небольших угроз, похоже, устанавливает верхнюю «границу» риска. Общество, хорошо защищённое от малых угроз, не предпринимает действий против больши́х, расселяясь на затапливаемых равнинах как только риск небольших наводнений уходит. Общество рассматривает регулярные небольшие угрозы так, словно таких угроз большего размера не существует, предпринимая меры безопасности против регулярных небольших наводнений, но не против редких крупных наводнений.

Память не всегда хороший проводник даже для вычисления вероятностей прошедших событий, не говоря уже о будущих.

  • 1. Lichtenstein, S., Slovic, P., Fischhoff, B., Layman, M. and Combs, B. 1978. Judged Frequency of Lethal Events. Journal of Experimental Psychology: Human Learning and Memory, 4(6), November: 551-78.
  • 2. Combs, B. and Slovic, P. 1979. Causes of death: Biased newspaper coverage and biased judgments. Journalism Quarterly, 56: 837-843.
  • 3. Kunreuther, H., Hogarth, R. and Meszaros, J. 1993. Insurer ambiguity and market failure. Journal of Risk and Uncertainty, 7: 71-87.
  • 4. Burton, I., Kates, R. and White, G. 1978. Environment as Hazard. New York: Oxford University Press.
Перевод: 
Remlin
Номер в книге "Рациональность: от ИИ до зомби": 
5
Оцените качество перевода: 
Средняя оценка: 5 (25 votes)

Обременительные детали

Элиезер Юдковский

Простая подтверждающая подробность, чтобы добавить художественной правдоподобности в противном случае сухому и неубедительному рассказу…

— Пу-Ба, из оперы «Микадо»1

Конъюнктивное заблуждение это когда люди оценивают вероятность P(A, B) выше вероятности P(B), несмотря на то, что доказано P(A, B) ≤ P(B). Например, в ходе эксперимента в 1981, 68% испытуемых эксперимента оценили «Рейган увеличит поддержку незамужних матерей и снизит поддержку местных администраций» выше, чем «Рейган увеличит поддержку незамужних матерей».

Длинная серия ловко разработанных экспериментов, отделивших альтернативные гипотезы и прибивших стандартную интерпретацию, показали, что конъюнктивное заблуждение возникает, потому что мы «подменяем суждением репрезентативности суждение о вероятности». Добавляя дополнительные детали, можно сделать итоги более характерными для процесса их производящего. Слова о том, что Рейган увеличит поддержку для незамужних матерей, будут звучать убедительней, если добавить утверждение о том, что Рейган снизит поддержку местных администраций. Неправдоподобность одного утверждения компенсируется правдоподобностью другого; они усредняются.

Поэтому и говорят: добавив деталей, можно сделать сценарий звучащим более правдоподобно, несмотря на то что события, в совокупности, станут менее вероятными.

Если это так, то мы можем легко найти футуристов, бессовестно распространяющих правдоподобные и детализированные сценарии будущего, или людей, легко принимающих огромные порции неподтвержденных утверждений, сдобренных несколькими звучащими сильно добавками. Если вы сталкиваетесь с конъюнктивным заблуждением лоб в лоб, вы, возможно, преуспеете в преодолении этой конкретной проблемы, постоянно поправляя себя. Но это лишь вре́менное решение.

В эксперименте 1982 года профессиональные прогнозисты назначали систематически более высокие вероятности утверждению «Россия вторгнется в Польшу, и дипломатические отношения между СССР и США временно прекратятся», чем «Дипломатические отношения между СССР и США временно прекратятся»2. В этом эксперименте каждую группу прогнозистов просили оценивать только одно утверждение. Какой стратегии эти прогнозисты, как группа, могли следовать, дабы исключить влияние конъюнктивного заблуждения, если никто не знал про альтернативное утверждение? Если никто не знал, что эксперимент и был про конъюнктивное заблуждение? Как они могли улучшить их суждения о вероятностях?

Вре́менные решения не помогут. Лечить нужно не симптомы, а болезнь.

Что прогнозисты могли предпринять во избежание конъюнктивного заблуждения, не видя с чем сравнивать, не зная даже о том, что тестироваться будет именно конъюнктивное заблуждение? Мне кажется, что им следовало заметить «и». С этим надо быть поосторожней, да не просто так, а бежать от него. Даже без знания, что исследователи специально тестировали их на конъюнктивное заблуждение. Им придется замечать соединение деталей, быть шокированными наглостью просьб оценить вероятности таких безумно сложных предсказаний. Им придется штрафовать вероятность существенно - как минимум в четыре раза, в соответствии с деталями эксперимента.

Размышления о возможных причинах приостановки дипломатических отношений между США и Советским Союзом так же помогли бы прогнозистам. «США и Советский Союз приостановили дипломатические отношения без причины» не являлся сценарием, им был «США и Советский Союз приостановили дипломатические отношения по любой причине».

Что насчет субъектов, оценивавших «Рейган увеличит поддержку незамужних матерей и снизит поддержку местных администраций»? Опять же, они должны быть шокированы союзом «и». Более того, им нужно сложить силу утверждений (логарифм вероятности со знаком минус), а не взять среднее. Им следует думать: «Рейган может снизить поддержку местных администраций, а может и не снизить (1 бит), но увеличение поддержки незамужних матерей кажется маловероятным (4 бита). Итоговая сила утверждения: 5 бит»3. Или: «Рейган не поддержит незамужних матерей. Один промах и он вне игры. Другое предположение только ухудшает дело».

По аналогии, давайте рассмотрим шестигранную игральную кость с четырьмя зелеными гранями и двумя красными. Испытуемым давали на выбор три серии исходов — 1 [КЗККК], 2 [ЗКЗККК] или 3 [ЗККККК] — и просили поставить на ту из них, которая с их точки зрения встретится в серии из двадцати бросков4. Шестьдесят пять процентов испытуемых выбрали [ЗКЗККК], вероятность получения которой меньше, чем вероятность получения [КЗККК], так как любая серия, содержащая [ЗКЗККК] включает и [КЗККК]. Как испытуемые могли преуспеть больше? Заметив вкрапление? Возможно; но это лишь лечение симптома, оно не решит фундаментальной проблемы. Вычисляя вероятности? Это определенно решило бы фундаментальную проблему, но мы не всегда можем вычислить точную вероятность.

Испытуемые проиграли эвристически, думая: «Ага! Цепочка 2 имеет наибольшую пропорцию зеленых и красных! Я должен ставить на нее!» Чтобы выиграть эвристически, им следовало думать: «Ага! Цепочка 1 короче. Я должен ставить на нее!».

Бритва Оккама должна вызывать у них сильные эмоции - каждая деталь должна ощущаться как бремя, даже один поворот кости.

Однажды, я беседовал кое с кем, очарованным одним неосмотрительным футуристом (тем, который добавляет множество изящно звучащих деталей). Я пытался объяснить почему я не был так же очарован этими потрясающими, невероятными теориями. Так что я рассказал про конъюнктивное заблуждение, конкретно про эксперимент «приостановленные отношения ± вторжение в Польшу». И он сказал: «Ладно, но как это связано с -», и я ответил: «Более вероятно, что вселенная копируется по любой причине, чем что она копируется посредством черных дыр, которые создаются продвинутыми цивилизациями, созданных в результате эволюции этой вселенной для этой конкретной цели». И он сказал «Ох».

До этого он не ощущал дополнительные детали, как бремя. Вместо этого они были подтверждающими деталями, добавляющими художественной правдоподобности рассказу. Кто-то подает вам группу странных идей, одна из них - вселенная копируется. Далее, он предоставляет поддержку утверждению о копировании. Но это не поддержка группы, несмотря на то, что рассказывается, как одна история.

Вам следует распутывать детали. Следует рассматривать их отдельно и вопрошать: «Откуда мы знаем эту деталь?». Кто-то набрасывает картину падения человечества в нанотехнологическое столкновение, где Китай отказывается подчиняться международному соглашению о контроле, за этим следует гонка вооружений… Подождите-ка… Откуда вы знаете, что это будет Китай? Это хрустальный шар в вашем кармане или вы просто рады быть футуристом? Откуда взялись все эти детали? Откуда взялась эта конкретная деталь?

Ибо сказано:

Если можешь облегчить бремя своё, сделай это.
Нет такой соломинки, что бессильна сломать твой хребет.

  • 1. https://en.wikipedia.org/wiki/The_Mikado
  • 2. Tversky and Kahneman, «Extensional Versus Intuitive Reasoning.»
  • 3. Здесь «сила утверждения» — его двоичный логарифм со знаком минус; чем она больше, тем менее вероятно утверждение. В примере вероятности утверждений должны быть 1/2 и 1/16 соответственно, вероятность всего утверждения — 1/32. — Прим. перев.
  • 4. Amos Tversky and Daniel Kahneman, «Judgments of and by Representativeness,» in Judgment Under Uncertainty: Heuristics and Biases, ed. Daniel Kahneman, Paul Slovic, and Amos Tversky (New York: Cambridge University Press, 1982), 84–98.
Перевод: 
Muyyd
Номер в книге "Рациональность: от ИИ до зомби": 
6
Оцените качество перевода: 
Средняя оценка: 4 (38 votes)

Ошибка планирования

Элиезер Юдковский

Международный Аэропорт Денвера открылся на 16 месяцев позже срока и с превышением бюджета на 2 млрд \$1.

Еврофайтер Тайфун, совместный оборонный проект нескольких европейских стран, был завершён с отставанием в 54 месяца, итоговая стоимость составила 19 млрд \$ вместо изначальных 7 млрд \$.

Сиднейский Оперный Театр, возможно, самый легендарный по превышению начальной стоимости проект. Изначально, оценка предполагала завершение в 1967 за 7 млн \$, но в итоге был завершен в 1973 за 102 млн \$2.

Неужели эти отдельные катастрофы привлекли наше внимание благодаря селективной доступности? Неужели они являются симптомами бюрократического или правительственного провала? Да, очень может быть. Но так же тут играет роль соответствующее когнитивное искажение, повторенное экспериментально с отдельными планирующими индивидуумами.

Бюлер и др. просили студентов оценить сроки сдачи своих академических проектов3. Конкретно они просили сказать, когда проекты будут сданы с 50%, 75%, 99% вероятностью. Попробуйте догадаться сколько студентов закончили до или после оцененного в 50%, 75%, 99% вероятности срока сдачи?

• 13% испытуемых уложились в срок, оцененный с 50% вероятностью успеха.

• 19% уложились в срок, оцененный в 75% вероятности успеха.

• и лишь 45% (менее половины!) уложились в срок, оцененный в 99% вероятности успеха.

Как Бюлер и др. отметили: «Результаты оценки 99% вероятности особенно удивляют: даже если их просили сделать крайне осторожное предсказание, по отношению к которому они чувствовали бы сильную уверенность, самоуверенность студентов намного превзошла их реальные достижения4.

В целом, этот феномен известен как «ошибка планирования». Ошибка планирования - это когда люди думают, что могут планировать, ха, ха.

Ньюби-Кларк и др. нашли ключ к скрытой подоплеке алгоритма планирования. Обнаружили, что

• Просьба спрогнозировать сценарии на лучшей реалистичной догадке

и

• Просьба спрогнозировать сценарии на самом удачном стечении обстоятельств…

… Привели к неразличимым результатам5.

Когда людей просили предоставить «реалистичный» сценарий, они воображали, что все пойдет как запланировано, без неожиданных задержек или непредвиденных катастроф - такой же сценарий, как и при самом удачном стечении обстоятельств.

Как оказалось, реальность преподносит обстоятельства хуже, чем «самое неудачное стечение обстоятельств», какое мы обычно воображаем.

В отличие от большинства когнитивных искажений, для борьбы с ошибкой планирования есть хорошая эвристика. Она не сработает для масштабного бардака вроде Международного Аэропорта Денвера, но пойдет для личного планирования или организации чего-то небольшого масштаба. Просто используйте «наружное наблюдение» вместо «взгляда изнутри».

У людей есть склонность генерировать предсказания, думая о конкретных, уникальных деталях задания, первыми приходящих на ум. И создавать сценарии того, как они будут выполнять задание - это мы, обычно, и называем планированием. Когда вы желаете, чтобы что-то было сделано, вам необходимо спланировать где, когда, и как; подумать сколько времени и ресурсов это потребует; визуализировать все шаги от начала до успешного завершения. Все это - «взгляд изнутри», и он не принимает в расчет неожиданные задержки и непредвиденные катастрофы. Как мы уже увидели, просьба визуализировать «худший сценарий» явно недостаточно сильно гасит оптимизм - они воображают недостаточно Мёрфично.

Взгляд снаружи - когда вы специально избегаете мыслей о конкретных, уникальных деталях проекта, и просто вопрошаете, сколько времени заняло завершение похожего проекта в прошлом. Это кажется контринтуитивным, ведь взгляд изнутри позволяет заметить больше деталей. Мысль об аккуратно сотканном предсказании, в котором была учтена вся доступная информация, выдающем лучшие результаты, искушает.

Но эксперименты показали: чем подробней испытуемые визуализировали, тем оптимистичнее (менее точными) они становились. Бюлер и др. просили испытуемых описать исключительно подробно их планы по шоппингу на Рождество: где, когда и как6. В среднем, эта группа ожидала покончить с покупками менее чем за неделю до Рождества. Другую группу просто попросили сказать, за какой срок до Рождества они планируют купить все необходимые рождественские подарки, и средний ответ был: за четыре дня. Обе группы закончили в реальности за три дня до Рождества в среднем.

Точно так же, Бюлер и др. сообщили об исследовании, в котором японские студенты ожидали закончить их сочинения за десять дней до срока сдачи. Закончили же они за один день до срока. Когда их спрашивали о прошлых сочинениях, выяснилось, что они так же были закончены «за день до срока». Такова сила взгляда снаружи против взгляда изнутри.

Похожее исследование свидетельствует, что опытные посторонние, знающие меньше деталей конкретного задания, но имеющие в памяти выполнение похожих, менее оптимистичны и более точны, чем те, кто проводит планирование.

Похоже, есть относительно надежный способ исправить ошибку планирования, если вам предстоит делать что-то похожее на то, что вы делали ранее. Просто задайтесь вопросом о том, сколько такие проекты длились в прошлом, игнорируя детали. Еще лучше: посоветуйтесь с опытным аутсайдером о том, сколько времени заняло выполнение похожих проектов в прошлом.

Вы получите ответ, который будет звучать отвратительно долгим, который, очевидно, был дан без учета всех особых причин по которым ваше конкретное задание будет выполнено быстрей. Но ответ остается правдой. Смиритесь.

  • 1. Я видел так же предположения о 3,1 млрд \$
  • 2. Roger Buehler, Dale Griffin, and Michael Ross, “Inside the Planning Fallacy: The Causes and Consequences of Optimistic Time Predictions,” in Gilovich, Griffin, and Kahneman, Heuristics and Biases, 250–270.
  • 3. Roger Buehler, Dale Griffin, and Michael Ross, “It’s About Time: Optimistic Predictions in Work and Love,” European Review of Social Psychology 6, no. 1 (1995): 1–32.
  • 4. Roger Buehler, Dale Griffin, and Michael Ross, “Inside the Planning Fallacy: The Causes and Consequences of Optimistic Time Predictions,” in Heuristics and Biases: The Psychology of Intuitive Judgment, ed. Thomas Gilovich, Dale Griffin, and Daniel Kahneman (New York: Cambridge University Press, 2002), 250–270.
  • 5. Ian R. Newby-Clark et al., “People Focus on Optimistic Scenarios and Disregard Pessimistic Scenarios While Predicting Task Completion Times,” Journal of Experimental Psychology: Applied 6, no. 3 (2000): 171–182.
  • 6. Buehler, Griffin, and Ross, “Inside the Planning Fallacy.” https://www.lesswrong.com/posts/CPm5LTwHrvBJCa9h5/planning-fallacy#cite….
Перевод: 
Muyyd
Номер в книге "Рациональность: от ИИ до зомби": 
7
Оцените качество перевода: 
Средняя оценка: 4.8 (32 votes)

Иллюзия прозрачности: почему вас не понимают

Элиезер Юдковский

Существует когнитивное искажение «знание задним числом». Оно проявляется так: люди, знающие, чем закончилась ситуация, верят, что исход можно было легко предсказать заранее. Когда мы знаем результат, мы переосмысливаем ситуацию с учетом этого результата. Даже если мы знаем об этом искажении, мы не можем воспринимать ситуацию так, как её воспринимает человек, который не знает того, что знаем мы.

С этим тесно связана иллюзия прозрачности: мы всегда знаем, что означают наши слова, и ожидаем, что остальные тоже это знают. Мы правильно понимаем смысл, читая написанное нами, обладая знаниями о том, что мы действительно имели в виду. Этим смыслом сложно поделиться с кем-то, кто будет руководствоваться лишь словами.

Джун порекомендовала ресторан Марку; Марк обедает там и обнаруживает, что там (а) посредственные еда и сервис или (б) вкусная еда и безупречный сервис. Затем Марк оставляет сообщение на автоответчик Джун: «Джун, я только что пообедал в ресторане, который ты мне порекомендовала, и я должен сказать, что это было изумительно, просто изумительно». Кейсар рассказал сценарий (а) группе людей, и 59% решило, что сообщение Марка было саркастическим и что Джун распознает сарказм.(1)

Среди тех, кому рассказали сценарий (б), только 3% решили, что Джун должна подумать о сообщении Марка как о саркастическом. Кейсар и Барр, по видимому, указывают, что испытуемые слышали голосовое сообщение. Кейсар показал, что если испытуемым сказали, что ресторан ужасен, но что Марк хотел скрыть свою реакцию, то они верили, что Джун не распознает сарказм в подобном сообщении.

Они предсказывали, что Джун должна распознать сарказм, когда Марк пытался скрыть негативную реакцию, так же легко, как когда он искренне хвалил ресторан. Так что они восприняли сообщение Марка прозрачным. Как будто они ожидали, что Джун поймет всё, что Марк захочет ей сказать.

«The goose hangs high» («Дело на мази») - старая английская идиома, которая не используется в современном языке. Кейсар и Блай сказали группе испытуемых, что «the goose hangs high» означает, что будущее выглядит многообещающим; другой группе сказали, что она означает, что будущее кажется мрачным. Затем испытуемых спросили, какое значение из этих двух кажется более подходящим для идиомы. Каждая группа решила, что именно услышанный ими смысл будет восприниматься как значение идиомы.

(Также тестировались другие идиомы: “come the uncle over someone”, “to go by the board” и “to lay out in lavender”. Ах, английский, прелестный язык).

Кейсар и Хенли тестировали калибровку ораторов. Будут ли они недооценивать, переоценивать, или оценивать верно, как часто слушатели понимают их? (6). Ораторам были предоставлены неоднозначные выражения («Мужчина преследует женщину на велосипеде») и снимающие неоднозначность изображения (мужчина бежит за женщиной, которая едет на велосипеде). Ораторов попросили произнести выражения перед аудиторией и оценить, как много слушателей поняли вложенный смысл. Ораторы думали, что были поняты в 72% случаев, тогда как настоящая цифра - 61%. Когда слушатели не понимали, ораторы оценивали понимание в 46%. Когда слушатели понимали, ораторы оценивали непонимание в 12%.

Другие субъекты, подслушавшие объяснение, показали отсутствие таких ошибок: они оценивали понимание в 56%.

Как заметили Кейсар и Барр, за два дня до атаки Германии на Польшу, Чемберлен послал письмо, вложив туда намерение показать, что Британия вступит в бой, если вторжение состоится (7). Письмо, составленное из дипломатичных выражений, было воспринято Гитлером как примирительное - и танки поехали.

Так что не стоит винить тех, кто не понял твои очевидные слова, в разговоре или письменно. Есть шанс, что твои слова более туманны, чем кажутся.

  1. Boaz Keysar, “The Illusory Transparency of Intention: Linguistic Perspective Taking in Text,” Cognitive Psychology 26 (2 1994): 165–208, doi:10.1006/cogp.1994.1006.
  2. Keysar and Barr, “Self-Anchoring in Conversation.”
  3. Boaz Keysar, “Language Users as Problem Solvers: Just What Ambiguity Problem Do They Solve?,” in Social and Cognitive Approaches to Interpersonal Communication, ed. Susan R. Fussell and Roger J. Kreuz (Mahwah, NJ: Lawrence Erlbaum Associates, 1998), 175–200.
  4. Keysar and Barr, “Self-Anchoring in Conversation.”
  5. Boaz Keysar and Bridget Bly, “Intuitions of the Transparency of Idioms: Can One Keep a Secret by Spilling the Beans?,” Journal of Memory and Language 34 (1 1995): 89–109,
    doi:10.1006/jmla.1995.1005.
  6. Boaz Keysar and Anne S. Henly, “Speakers’ Overestimation of Their Effectiveness,” Psychological Science 13 (3 2002): 207–212, doi:10.1111/1467-9280.00439.
  7. Keysar and Barr, “Self-Anchoring in Conversation.”
Перевод: 
Kroid, Muyyd
Номер в книге "Рациональность: от ИИ до зомби": 
8
Оцените качество перевода: 
Средняя оценка: 4.3 (23 votes)

Ожидая короткие понятийные расстояния

Элиезер Юдковский

Эволюция приспособила homo sapiens к жизни в маленьких (не более 200 человек) племенах охотников и собирателей, не знающих письменности. В этих условиях — в так называемой «среде эволюционной адаптации» — всё накопленное знание сохранялось в памяти и передавалось устно.

В таком мире все фундаментальные знания — всеобщи. Любая информация, которую нельзя назвать строго приватной, является публичной, и из этого правила нет исключений.

В среде эволюционной адаптации было очень маловероятным оказаться от кого-то на расстоянии, большем чем один понятийный шаг, одно логическое умозаключение. Когда ты обнаруживал новый оазис, тебе не надо было рассказывать своим соплеменникам, что такое оазис, или почему стоит пить воду, или как перемещать ноги для того, чтобы ходить. Где находится оазис, знаешь только ты; это знание приватно. Но у всех остальных уже есть предпосылки для того, чтобы понять описание оазиса, все уже владеют понятиями, необходимыми, чтобы думать о воде; это знание универсально. В тех редких случаях, когда какие-то объяснения были необходимы, тебе почти никогда не нужно было разъяснять свои понятия. В самом худшем случае, нужно было рассказать об одном новом понятии, но не о двух (или больше) одновременно.

В среде эволюционной адаптации не было абстрактных дисциплин, сжимавших огромные кучи свидетельств в элегантные теории, описанные в книгах, выводы которых на десятки понятий и сотни умозаключений удалены от всеобщего информационного фундамента.

В среде эволюционной адаптации лишь лгуны или идиоты пытаются рассказывать о вещах, обоснование которых не очевидно. У их слушателей вряд ли возникнет мысль о том, что, возможно, этот парень располагает какой-либо достоверной фундаментальной информацией, неизвестной никому в твоём окружении. Невозможность такого положения дел была надёжной неизменной характеристикой среды эволюционной адаптации.

И наоборот, если ты сказал что-то вопиюще очевидное, а твой собеседник с этим не согласился, то он либо идиот, либо намеренно упирается с целью позлить тебя.

И к тому же, если кто-то рассказывает о какой-то вещи, обоснование которой не очевидно, а затем ждёт от тебя понимания и поддержки, — и возмущается, не найдя их — то он явно безумен.

И я думаю, что этот эффект (вкупе с иллюзией прозрачности и самоякорением(English)) объясняет многие аспекты тех легендарных затруднений, с которыми сталкиваются учёные, пытающиеся общаться с обывателями (или даже просто с учёными из других областей науки). Я часто вижу следующую картину: объяснение не удаётся. Популяризатор науки делает один шаг назад, хотя правильным было бы сделать два, или даже больше шагов назад. Слушатель же, в свою очередь, считает, что всё должно стать понятным через один шаг, хотя на самом деле для этого необходимо больше шагов. Обе стороны ведут себя так, словно понятийное расстояние между ними очень мало; будто бы всеобщие знания очень близки к любому новому знанию.

Биолог, говоря с физиком, может обосновать теорию эволюции, сказав, что она — «простейшее объяснение». Но легендарная история науки не впиталась в души большинства остальных людей; они не видят этих столетий, от Ньютона до Эйнштейна, подаривших фразе «простейшее объяснение» весь её потрясающий смысл, превративших её в Слово Силы, что произносится при рождении теорий и высекается на их надгробных камнях. Для не-учёного фраза «но это — простейшее объяснение!» звучит, как любопытный, но вряд ли решающий аргумент; простота не выглядит могущественным инструментом для постижения офисных интриг или для починки автомобиля. Должно быть, биолог слишком ослеплён любовью к своим идеям для того, чтобы непредвзято взглянуть на альтернативные объяснения, которые звучат настолько же убедительно (они звучат убедительно для меня, поэтому они должны звучать убедительно для любого человека из моего окружения).

Биолог может понять, что впервые теория эволюции звучит странновато. Однако, если кто-то отвергает эволюцию даже после того, как биолог рассказал, что это наиболее простое объяснение и пояснил, почему… Ну, видимо, не-учёные просто глупы, и нет смысла с ними разговаривать.

Хорошее выступление должно проложить понятийный путь, начинающийся от позиций, которые аудитория уже знает или принимает. Если ты задашь недостаточную глубину рекурсии, то в итоге ты будешь просто разговаривать сам с собой.

Каждое новое заявление должно очевидным образом опираться на аргументы, сказанные тобой ранее и принятые аудиторией. Как только эта цепочка нарушается, аудитория начинает считать тебя жертвой секты.

То же самое произойдёт, если ты позволишь себе опираться на аргумент более сильно, чем готовы разрешить слушатели. Например, если ты посчитаешь фразу «эволюция — простейшее объяснение» решающим аргументом (таким она и является), а не любопытным, чуточку забавным нюансом (так она выглядит в глазах человека, не привыкшего чтить Бритву Оккама).

И не давай слушателям намёков на то, что ты считаешь их отставшими от тебя на дюжину понятийных шагов, или то, что ты думаешь, что располагаешь какими-то особыми неизвестными им фундаментальными знаниями. Слушатели не знают о том, что эволюционная психология предсказывает существование дефекта мышления, ведущего к недооценке понятийных расстояний. Они не думают, что у пробок на дороге общения есть какая-то особенная причина. Поэтому их просто будет раздражать твоя снисходительность.

И если ты считаешь, что можешь кратко объяснить понятие «систематической недооценки понятийных расстояний», быстренько бросив пару слов, то вынужден тебя огорчить…

Перевод: 
BT
Номер в книге "Рациональность: от ИИ до зомби": 
9
Оцените качество перевода: 
Средняя оценка: 3.7 (85 votes)

Линза, видящая свои изъяны

Элиезер Юдковский

Свет покидает Солнце, падает на шнурки и отражается от них; часть фотонов проходит сквозь зрачок и попадает на сетчатку; энергия фотонов запускает волну нервных импульсов; нервные импульсы доходят до зрительной коры, где на основе оптической информации строится трёхмерная модель, распознанная как развязанные шнурки; и теперь ты убеждён в том, что твои шнурки и в самом деле развязаны.

Вот секрет осознанной рациональности: в этом процессе передачи сцепленности нет ничего магического, и его можно понять. Ты можешь понять, как ты видишь свои шнурки. Ты можешь думать о том, какие когнитивные процессы создают убеждения, отражающие реальность, и какие когнитивные процессы — нет.

Мыши могут видеть, но они не могут понять зрения. Ты можешь понять зрение, и поэтому ты способен на вещи, которые мышам и не снились. Подивись этому несколько секунд, ведь это действительно чудо.

Мыши видят, но они не знают, что у них есть зрительная кора, и поэтому они не могут систематически бороться с оптическими иллюзиями. Мышь живёт в ментальном мире, где есть кошки, дыры, сыр и мышеловки, — но нет мышиных мозгов. Их камеры не могут сфотографировать линзы собственного объектива. Но люди могут посмотреть на причудливую картину и осознать, что часть того, что они видят, является линзами их объектива. Ты не обязан всегда верить своим глазам, но ты обязан понимать, что у тебя есть глаза — у тебя должны быть отдельные участки памяти для карты и для местности, для чувств и реальности. Если ты считаешь этот навык тривиальным, вспомни, насколько он редок в царстве животных.

Вся идея Науки — это, в сущности, поиск наиболее надёжного способа отразить в зеркале разума содержимое целого мира. Мыши никогда не смогли бы изобрести такую идею. Размышляя о всех вопросах вроде «мы проводим повторяемые эксперименты, чтобы фальсифицировать теорию», мы можем разобраться, почему именно наука работает. Наука — это не отдельный магистерий, далёкий от реальной жизни и непонятный для простых смертных. Нельзя сказать, что науку можно применять только в лабораториях. Наука — это постижимый и объективно существующий процесс, который связывает содержимое мозга с реальностью.

Наука довольно логична, если как следует о ней подумать. Но мыши не могут думать о мышлении, и поэтому у них нет науки. Не проглядите заключённого в этом чуда, не упустите потенциальной мощи, которую нам дарит этот факт. Нам — в смысле «личностям», а не «научным сообществам».

Нужно признать, что задача понять мыслительный механизм может быть сложнее задачи понять часовой механизм — но между этими задачами нет фундаментальных различий.

Однажды я спросил посетителей канала #philosophy: «Верите ли вы в то, что ядерная война случится в течение ближайших 20 лет? Если нет, то почему?». Один человек сказал: «Я не думаю, что в ближайшие 100 лет начнётся ядерная война: все игроки, участвующие в принятии подобных решений, сейчас в ней не заинтересованны». Я спросил: «Но почему ты считаешь, что ситуация сохранится в течение 100 лет?». «Просто надежда», ответил он.

Если поразмыслить об этом мыслительном процессе, то можно увидеть, что перспектива ядерной войны пугает этого человека, и поэтому его мозг отвергает соответствующее убеждение. Но если представить себе миллиард миров — ответвления Эверетта или дупликаты Тегмарка(English) — то станет ясно, что такие размышления не создают корреляции между оптимистами и мирами без ядерной войны, как должно было бы быть, если бы они были бы рациональными.

(В этот момент у кого-то может появится соблазн сказать «Но если у меня есть надежда, я буду работать лучше, заработаю больше денег, тем самым помогу мировой экономике, и в результате страны будет не так просто столкнуть в пучину злобы, бедности и отчаяния, когда возможность ядерной войны всерьёз угрожает будущему. Получается, что надежда имеет отношение к реальности». Раз уж дошло до такого, мне придётся вытащить теорему Байеса и количественно измерить силу этого свидетельства. Оптимизм не может иметь столь огромный эффект на мире; его недостаточно для того, чтобы сместить вероятность ядерной войны на 20%, или насколько там оптимизм смещает степень убеждённости. Сильно изменять свои убеждения из-за события, несущего очень малый заряд сцепленности — практика, не способствующая точному картографированию.)

Задуматься о том, какие убеждения сделают тебя счастливым — посмотреть внутрь, а не наружу. Ответ может сказать что-то новое о твоей психике, но это не свидетельство, сцепленное с окружением. Я не имею ничего против счастья, но счастье должно порождаться картиной мира, а не преступным использованием ментальных карандашей в целях сокрытия правды.

Если ты можешь это увидеть — если ты можешь заметить, что надежда слишком сильно влияет на твои размышления первого уровня; если ты можешь увидеть свой мозг как рисующий карты механизм, в котором есть недочёты — то ты можешь что-нибудь исправить. Мозг — дефектная линза, не совсем точно показывающая действительность. Это верно в отношении мозга как мыши, так и человека. Но мозг человека — это линза, могущая понять свои изъяны, могущая увидеть свои систематические ошибки, свои искажения, а после применить к ним исправления второго уровня. Этот факт делает дефектную линзу намного могущественней на практике. Делает её не совершенной, но намного более эффективной.

Перевод: 
BT
Номер в книге "Рациональность: от ИИ до зомби": 
10
Оцените качество перевода: 
Средняя оценка: 3.5 (100 votes)

Ложные убеждения

В цепочке «Ложные убеждения» показана дискуссия об ожиданиях, которые расходятся с декларированными убеждениями.

Материалы цепочки распространяются по лицензии CC BY-NC-SA 3.0

Автор: 
Элиезер Юдковский

Убеждения должны окупаться

Элиезер Юдковский

Начало одной древней притчи звучит так:

Если дерево падает в лесу, и нет никого рядом, чтобы это услышать — создаёт ли дерево звук? Кто-то говорит «да, оно порождает колебания воздуха». Другой говорит «нет, никакой мозг не производит обработку слуховой информации».

Представим, что после того, как дерево упало, эти двое вместе входят в лес. Будет ли первый ожидать увидеть дерево, упавшее влево, а второй — дерево, упавшее вправо? Представим, что перед падением дерева двое оставили рядом с ним включённый диктофон; а после — воспроизводят его запись. Будет ли кто-либо из них ждать не тех звуков, что другой? Представим, что они присоединили электроэнцефалограф к каждому мозгу на планете — планирует ли кто-нибудь увидеть график, который не рассчитывал увидеть второй? Несмотря на то, что эти люди спорят, один говорит «нет», а другой «да», ожидаемые ими переживания не отличаются. Спорщики считают, что у них разные модели мира, но в этих моделях нет никаких различий по отношению к тому, какие будущие наблюдения им предстоят.

Соблазнительно попытаться уничтожить этот класс ошибок с помощью запрета всех убеждений, не являющихся ожиданиями какого-либо чувственного опыта. Но в мире есть многое, что не ощущается напрямую. Мы не видим атомов, из которых состоит кирпич, но эти атомы действительно существуют. Под твоими ногами пол, но ты не ощущаешь его напрямую, ты видишь отражённый от него свет (или, точнее, ты видишь результат обработки этого света сетчаткой и зрительной корой). Сделать вывод о существовании пола на основе его зрительного наблюдения — значит подумать о незримых причинах, стоящих за ощущениями. Этот шаг выглядит очень незначительным и очевидным, но это всё же шаг.

Ты стоишь на вершине небоскрёба, рядом с тикающими старинными часами, имеющими часовую, минутную и секундную стрелки. В твоей руке шар для боулинга, и ты сбрасываешь его с крыши. На какой по счёту щелчок стрелок ты ожидаешь услышать грохот шара, упавшего на землю?

Чтобы точно ответить на этот вопрос, тебе нужно использовать убеждения вроде «гравитация Земли равна 9,8 м/с^2» и «высота этого здания равна 120 метрам». Эти убеждения нельзя назвать бессловесными ожиданиями чувственного опыта; они довольно словесные, пропозициональные. Можно, не сильно погрешив против истины, описать эти убеждения как предложения, составленные из слов. Но эти убеждения имеют выводимое последствие, которое является прямым чувственным ожиданием — если секундная стрелка часов стоит на числе 12, когда ты бросил шар, то ты ожидаешь увидеть её на числе 1, когда ты услышишь грохот пять секунд спустя. Для того, чтобы ожидать чувственный опыт настолько точно, насколько это возможно, необходимо обрабатывать убеждения, не являющиеся ожиданиями чувственного опыта.

Великая сила Homo Sapiens состоит в том, что мы, лучше чем любой другой вид на планете, можем научиться моделировать невидимое. И в этом же состоит одна из наших величайших слабостей. У людей нередко встречаются убеждения о вещах, которые не просто незримы, но и нереальны.

Тот же самый мозг, что может логически вывести и построить сеть причин, лежащую за чувственным опытом, может построить и сеть причин, не соединённую ни с каким чувственным опытом (или очень плохо соединённую). Алхимики были убеждены в том, что флогистон вызывает огонь — очень упрощённо, это можно представить, как узел с надписью «флогистон», от которого тянется стрелка к чувственному опыту тёплого костра — но это убеждение не производило предсказаний на будущее; связь между флогистоном и наблюдениями всегда корректировалась после наблюдений, вместо того, чтобы как-нибудь ограничить наблюдения заранее. Или, скажем, учитель литературы говорит тебе, что знаменитый писатель Валки Вилкинсен — «пост-утопист». Что изменилось в твоих ожиданиях по поводу его книг в свете этой новой информации? Ничего. Это убеждение — если вообще можно называть это убеждением — вообще никак не связано с чувственным опытом. Но, тем не менее, тебе лучше запомнить о связи между Валки Вилкинсеном и атрибутом «пост-утопист»: тогда ты сможешь извергнуть это обратно на будущем экзамене. Если тебе сообщат, что «пост-утописты» показывают «охлаждение колониальных чувств», то ситуация совершенно аналогична: если автор письменного теста спросит, показывал ли Вилкинсен охлаждение колониальных чувств, то стоит ответить утвердительно. Убеждения связаны друг с другом, хоть и не связаны ни с каким ожидаемым опытом.

Люди могут построить целые сети убеждений, соединённые только друг с другом — будем называть это явление «плавающими» убеждениями. Это уникальный человеческий изъян, не имеющий аналогов у остальных животных, извращение способности Homo Sapiens строить абстрактные и гибкие сети убеждений.

Одна из добродетелей рационализма — эмпиризм — состоит в привычке постоянно задаваться вопросом о том, какой опыт предсказывается этим убеждением — или, ещё лучше, какой запрещается. Ты убеждён, что флогистон — причина огня? Тогда что ты ожидаешь увидеть, исходя из этого? Ты считаешь Валки Вилкинсена пост-утопистом? Тогда что ты рассчитываешь встретить в его книгах? Нет, не «охлаждение колониальных чувств»; какое переживание случится с тобой? Веришь ли ты в то, что если дерево падает в лесу, и нет никого рядом, чтобы это услышать, то оно всё равно создаёт звук? Тогда какой опыт должен выпасть на твою долю?

Ещё лучше спросить о том, какой опыт тогда с тобой точно не случится. Ты веришь в то, что жизненная сила объясняет загадочную разницу между живым и неживым? Тогда какие происшествия это убеждение запрещает, какое событие совершенно точно опровергнет это убеждение? Ответ «никакое» говорит о том, что это убеждение не ограничивает возможные переживания. Оно позволяет случиться с тобой чему угодно. Оно плавает.

Споря по поводу вопроса, вроде бы связанного с фактами, всегда держи в уме различие ожиданий будущего, из-за которого происходит спор. Если найти это различие не удаётся, то, скорее всего, вы спорите о названиях ярлыков в сети убеждений — или, ещё хуже, о плавающих убеждениях: ракушках-прилипалах на сети убеждений. Если ты не представляешь, какой опыт следует из того, что Валки Вилкинсен принадлежит к пост-утопистам, то ты можешь спорить бесконечно (а ещё ты можешь опубликовать бесконечное количество статей в литературных журналах).

И самое главное: не спрашивай, во что верить, — спрашивай, чего ожидать. Каждый вопрос об убеждениях должен порождаться вопросом о предсказаниях, и именно этот вопрос о предсказаниях должен быть в центре внимания. Каждое смутное убеждение должно рождаться как смутное ожидание, а затем оплачивать жилплощадь прогнозами будущего. Если убеждение становится злостным неплательщиком — высели его.

Перевод: 
BT
Номер в книге "Рациональность: от ИИ до зомби": 
11
Аудио: 

Убеждения должны окупаться

Аудиофайл: 
Чтец: 
Даниил Храмцов
Оцените качество перевода: 
Средняя оценка: 4.1 (77 votes)

Сказ о науке и политике

Элиезер Юдковский

Во времена Византийской империи светская жизнь оказалась разделена на два лагеря: Синий и Зелёный. Синие и Зелёные убивали друг друга на дуэлях, в драках «стенка на стенку», в засадах и погромах. Прокопий Кесарийский говорил о них: «Вражда к противникам возникает у них без причины и остаётся навеки; не уважаются ни родство, ни свойство, ни узы дружбы. Даже родные братья, приставшие один к одному из этих цветов, другой к другому, бывают в раздоре между собою». Эдвард Гиббон писал: «Поддержка одной из группировок стала необходимой для любого кандидата, будь он светским или духовным лицом».

Кто же были эти Синие и Зелёные? Всего лишь спортивные болельщики — сторонники синей и зелёной команд в гонках на колесницах.

А теперь представьте общество будущего, которому пришлось сбежать в сеть подземных туннелей и пещер и закрыть все входы. Мы не будем уточнять, бежали ли они от болезни, войны или от радиации. Первым подземным жителям удалось наладить производство еды, возобновление воздуха, найти воду, организовать освещение и выжить. Их потомки благоденствовали и даже стали строить города. О верхнем мире остались лишь легенды, написанные на клочках бумаги, и один из таких клочков описывал небо, бескрайний простор воздуха над полом без стен. Небо было лазурного цвета, и в нем летали странные объекты, напоминающие пучки белого хлопка. Значение слова «лазурный» вызывало противоречия, некоторые говорили, что оно обозначает синий цвет, а другие — зелёный.

В ранние дни подземного сообщества соперничество Синих и Зелёных доходило до открытого насилия, но теперь царит перемирие — мир, рождённый отвращением к бессмысленной вражде. Культурные обычаи изменились, существует многочисленный и преуспевающий средний класс, воспитанный в духе правового государства и непривычный к насилию. В школах преподаётся история: как долго длилась вражда между Синими и Зелёными, как много людей погибло, как мало в итоге изменилось. Граждане подготовлены к странной новой философии, гласящей, что люди — это люди, не важно, за Синих они или за Зелёных.

Сам конфликт не исчез. Общество всё ещё разделено на Синие и Зелёные области, в любом актуальном культурном или политическом вопросе выделяется «Синяя» и «Зелёная» позиции. Синие ратуют за налоги на личные доходы, Зелёные поддерживают налоги на продажи торговцев. Синие придерживаются более строгих законов о браке, в то время как Зелёные хотели бы упростить бракоразводный процесс. Синие пользуются поддержкой центральных городских районов, а периферийные фермеры и продавцы воды обычно оказываются в лагере Зелёных. Синие верят, что Земля — это огромная шарообразная скала в центре вселенной, а Зелёные считают, что Земля — это огромная плоская скала, вращающаяся вокруг другого объекта, называемого Солнцем. Отнюдь не каждый Синий или Зелёный гражданин принимает «Синюю» или «Зелёную» позицию по любому вопросу, но довольно тяжело найти городского торговца, который считает, что небо было синим, и в то же время голосует за налоги на личные доходы и более свободные законы о браке.

Подземелье всё ещё поляризовано. Царит хрупкий мир. Есть некоторое число людей, искренне считающих, что Синие и Зелёные должны быть друзьями. Обычное дело, когда Зелёный покровительствует Синему магазину или Синий любит посещать Зелёную таверну. Из перемирия, изначально рождённого усталостью, медленно растёт дух терпимости и даже дружбы.

Однажды в Подземелье произошло небольшое землетрясение. Группа из шести туристов почувствовала содрогание земли, находясь на прогулке среди руин древнего поселения, где-то в верхних пещерах. Один из туристов упал и поранил колено, и группа решила повернуть назад, опасаясь новых землетрясений. На обратном пути один из них заметил странное дуновение воздуха, какой-то запах, исходящий из давно заброшенного туннеля. Не обращая внимания на предостережения спутников, этот человек одолжил лампу и направился к туннелю. Каменный коридор шёл выше… и выше… и наконец закончился дырой, выходящей из этого мира. Каменные стены закончились. Пространство, бесконечное пространство простиралось в никуда, здесь хватило бы места для тысячи городов. Невообразимо далеко наверху и слишком ярко, чтобы смотреть не щурясь, жгучий огонёк лучился светом, освещающим всё вокруг, как ничем не прикрытая нить накаливания какой-то гигантской лампы. В воздухе, ничем не поддерживаемые, висели непостижимые пучки белого хлопка. И цвет бескрайнего сияющего потолка был…

На этом месте история разветвляется, в зависимости от того, кто именно из туристов решил последовать по коридору к поверхности.

Адитья Синяя стояла под синей бесконечностью и медленно улыбалась. Улыбка не была радостной. В ней была ненависть и раненная гордость. Она припоминала каждый свой аргумент в спорах с Зелёными, каждое соперничество, каждую вырванную победу. «Ты всё время была права», — шепнуло ей небо, — «и теперь ты можешь это доказать». Какое-то мгновенье Адитья стояла, впитывая послание, упиваясь им, а затем она повернулась и ушла в коридор, неся его миру. Шаг, ещё шаг… её пальцы сжались в кулак. «Перемирие закончено», — сказала она.

Бэррон Зелёный бессмысленно глазел на хаос цветов долгие секунды. А потом запоздавшее понимание взорвалось в его животе, как удар молота. Слёзы потекли из его глаз. Бэррон думал о Катэйской Резне, когда армия Синих вырезала всё население городка Зелёных, включая детей. Он думал о древнем Синем генерале — Аннасе Релле, который объявил Зелёных «чумной ямой, язвой, нуждающейся в прижигании». Он думал об огоньках ненависти, которые он замечал в глазах Синих, и что-то внутри него треснуло. «Как ты можешь быть на их стороне?!» – закричал он небу и начал рыдать. Стоя под злобным синим свечением, он знал, что вселенная всегда была обителью зла.

Чарльз Синий ошеломлённо созерцал синий потолок. Как профессор смешанного колледжа он всегда аккуратно подчёркивал, что Синяя и Зелёная точки зрения в равной степени верны и заслуживают терпимого отношения, небо — это метафизическая сущность, а «лазурный» — цвет, который может восприниматься по-разному. На мгновенье Чарльз задумался, не увидит ли какой-нибудь Зелёный, встав на его место, зелёный потолок, или не будет ли потолок зелёным завтра, но он не стал бы делать выживание цивилизации ставкой в этом споре. Это был всего лишь природный феномен, не имеющий никакого отношения к морали или к обществу… Но феномен, который наверняка поймут неправильно, как опасался Чарльз. Он вздохнул и повернулся к коридору. Завтра он придёт сюда один и закроет проход.

Дарья, когда-то Зелёная, пыталась дышать посреди обломков своего мира. «Я не зажмурюсь» — сказала она себе. — «Я не отвернусь». Всю свою жизнь она была Зелёной, а теперь она должна стать Синей. Её друзья, её семья… все они отвернутся от неё. Говори правду, даже если твой голос дрожит, когда-то говорил ей отец. Но сейчас отец был мёртв, а мать никогда не сможет понять. Дарья смотрела в спокойный синий глаз неба, пытаясь принять его, и наконец её дыхание успокоилось. «Я ошибалась», — скорбно сказала она себе. В конце концов, не так уж это и сложно. Она найдёт новых друзей, и, возможно, семья сможет простить её… А может, они даже отважатся сами встать под этим небом и пройти этот экзамен, подумала она с надеждой. «Небо синее», — произнесла Дарья в качестве эксперимента, и ничего ужасного с ней не произошло, правда, у неё не получилось заставить себя улыбнуться. Дарья Синяя печально выдохнула и пошла обратно в свой мир, думая о том, что она скажет.

Эддин Зелёный посмотрел в синее небо и цинично рассмеялся. Наконец-то он понял учебник мировой истории, правда, ему всё равно не верилось, что они были такими дураками. «Глупцы», — произнёс Эддин, — «глупцы, глупцы, всё это время оно было здесь». Ненависть, убийства, войны, и всё это время оно было просто явлением, о котором кто-то когда-то написал на бумаге, как обычно пишут о любом другом явлении. Никакой поэзии, никакой красоты, ничего такого, о чём любой здравомыслящий человек станет беспокоиться. Просто одно бессмысленное слово, влияние которого распространилось за любые разумные границы. Эддин устало прислонился к стене пещеры, пытаясь придумать, как не дать миру взорваться от этого открытия, и задаваясь вопросом, а не заслуживают ли все они именно этого.

Феррис невольно открыл рот, он замер на месте в абсолютном изумлении и восхищении. Его глаза жадно метались туда-сюда, с неохотой покидая одно зрелище, чтобы впиться взглядом в другое. Синее небо, белые облака, бескрайняя неизвестность снаружи, полная мест и предметов, а, возможно, и людей, которых никогда не видели в Подземелье. «О, так вот какой это цвет», — сказал Феррис и отправился исследовать.

Дополнение от ex-Parrot

Лоретта Зелёная посмотрела на небо и сказала: «Оно синее. Следовательно, это не небо. Несмотря на безграничность, несмотря на открытость и несмотря на эти штуки, похожие на клочки белого хлопка. Вообще, после того, как я задумалась, они не кажутся так уж сильно похожими на хлопок» .

Джон Экуменист сказал: «Как я всегда и говорил. Оно лазурное!»

Перевод: 
xeye
Номер в книге "Рациональность: от ИИ до зомби": 
12
Аудио: 

Сказ о науке и политике

Аудиофайл: 
Чтец: 
Даниил Храмцов
Оцените качество перевода: 
Средняя оценка: 4.3 (52 votes)

Вера в убеждения

Элиезер Юдковский

Карл Саган как-то рассказал притчу(English) о человеке (для удобства дадим ему имя Кредерус), который пришёл к нему и заявил: «В моём гараже живёт дракон». Потрясающе! В течение столетий по всему миру гуляли легенды о драконах, но до сих пор ни у кого нет никаких убедительных свидетельств их существования. Конечно же, любой человек ответит: «Я хочу посмотреть на настоящего дракона, где этот гараж?». Но гость вынужден нас огорчить: дракон невидим.

Далее Саган замечает, что невидимость не делает гипотезу о драконе нефальсифицируемой. Можно войти в гараж и услышать громкое дыхание из пустоты, увидеть неожиданно возникающие следы на земле, а потом при помощи инструментов обнаружить, что что-то в гараже поглощает кислород и выделяет углекислый газ.

Поэтому стоит ответить: «Ничего страшного, я попробую услышать его дыхание». Но Кредерус отвечает, что дракон совершенно бесшумен. — А что, если я измерю содержание углекислоты в воздухе? — Нет, дракон не дышит. — Можно распылить в воздухе муку, тогда она налипнет на дракона, и его контур можно будет увидеть. — Дракон проницаем для муки.

Мораль этой басни в том, что плохая гипотеза должна ловко маневрировать, чтобы избежать опровержения. Но я рассказываю эту историю не для того же, что и Карл Саган. Я хочу проиллюстрировать другую идею.

Где-то в недрах разума Кредеруса явно хранится правильная модель ситуации, поскольку он заранее ожидает определённых результатов всех этих проверок (а именно — таких результатов, которые ему придётся оправдывать).

Некоторые философы запутываются в подобных сценариях, не понимая, верит ли Кредерус в дракона, или всё-таки нет. Можно подумать, что в человеческом мозге мало места, и храниться может только одно убеждение за раз! Реальный разум намного сложнее. Как я уже говорил, убеждения бывают разными, не все можно назвать «прямыми ожиданиями чувственного опыта». Кредерус явно не ожидает увидеть в гараже ничего необычного, иначе он не оправдывался бы заранее. Также возможно, что в его фонде словесных убеждений хранится «В моём гараже живёт дракон». Рационалисту может показаться, что эти два убеждения должны конфликтовать друг с другом, даже несмотря на то, что они разных видов. Но, тем не менее, если написать «небо зелёное» на фотографии синего неба, то бумага и не подумает исчезать в языках пламени.

Добродетель эмпиризма должна не позволить нам совершать подобные ошибки. Рационалисты должны постоянно выяснять, какие переживания предсказывают их убеждения — требовать, чтобы убеждения окупались. Но проблема Кредеруса лежит глубже, и её не исцелить этим простым советом. Довольно просто соединить убеждение в драконе с ожидаемым наблюдением гаража: если ты веришь в живущего в гараже дракона, то ты, открыв дверь, рассчитываешь увидеть этого дракона. Если ты не видишь дракона, то это означает, что в гараже дракон не живёт. Действия довольно просты. Ты даже можешь попытаться повторить это со своим гаражом.

Но нет, эта невидимость — симптом чего-то похуже.

Возможно, ты помнишь тот момент из детства, когда ты уже начал сомневаться в существовании деда Мороза, но ещё считал правильным верить в деда Мороза, и поэтому пытался отвергнуть сомнения. Дэниел Деннет заметил: когда трудно верить в X, намного легче верить в то, что ты обязан верить в X. Как можно верить в то, что Первичное Космическое Небо одновременно совершенно синее, и совершенно зелёное? Эта фраза вводит в замешательство; непонятно, что это означает в смысле ожидаемых переживаний, непонятно, во что именно ты бы верил, если бы верил. Намного проще поверить в то, что правильно, хорошо, добродетельно и полезно верить в то, что Первичное Космическое Небо одновременно полностью зелёное и полностью синее. Деннет называет это «верой в убеждение» (того же термина буду придерживаться и я).

Ну и затем, раз мы имеем дело с человеческим разумом, всё как обычно усложняется. Думаю, что даже Деннет слишком сильно упростил то, как эта психологическая уловка работает на практике. К примеру, если человек верит в убеждение, то он не может признаться себе в том, что он просто верит в убеждение: ведь добродетельно верить, а не верить в убеждение, следовательно если ты веришь в убеждение, а не просто веришь, то ты не добродетелен. Никто не скажет (даже про себя): «Я не считаю, что Первичное Космическое Небо одновременно синее и зелёное, но я считаю, что мне следует так считать», разве что этот человек необычайно хорошо умеет признавать свои недостатки. Люди не верят в веру в убеждения, они просто верят в убеждения.

(Если вы находите предыдущий абзац сложным для понимания, попробуйте поизучать математическую логику, которая учит остро различать такие вещи, как утверждение P, доказательство P и доказательство того, что P доказуемо. Такие же коренные различия есть и между P, желанием P, верой в P, желанием верить в P и верой в то, что ты веришь в P.)

Вера в убеждения бывает разной. Она может быть явной: человек осознанно повторяет про себя «Добродетельно верить в то, что Первичное Космическое Небо одновременно совершенно синее, и совершенно зелёное» (и при этом считает, что он в это верит, разве что этот человек необычайно хорошо умеет признавать свои недостатки). Бывают и менее бросающиеся в глаза формы. Возможно, Кредерус боится публичного осмеяния, которое, как он считает, неизбежно последует, если он публично признает, что был неправ (хотя, фактически, любой рационалист искренне порадуется за него, а остальные будут скорее высмеивать Кредеруса, если он, отнюдь, продолжит заявлять о драконе, живущем в гараже). Возможно, Кредеруса передёргивает от перспективы признаться себе в том, что дракона нет (точно также, как его передёргивало бы от физической боли): это противоречит его представлению о себе как о победителе-первооткрывателе, увидевшем в своём гараже то, что упустили все остальные.

Будь все наши мысли теми осознанными предложениями на естественном языке, которыми обычно манипулируют философы, человеческий мозг был бы намного проще для понимания. Быстро утекающие мысленные образы, невысказанные вспышки боли, исполняемые без сознательного ведома желания — всё это составляет такую же часть личности, как и слова.

Несмотря на то, что я не соглашаюсь с Деннетом касательно некоторых деталей и тонкостей, я всё равно считаю, что введённое им понятие вера в убеждение — ключ к понимаю Кредеруса. Однако, необходимо более широко трактовать понятие «убеждения», не ограничивать его вербальными утверждениями. «Убеждение» может включать в себя неявные регуляторы ожиданий. «Вера в убеждение» может содержать неявные ориентиры когнитивного поведения. С точки зрения психологии, утверждение «Кредерус не убеждён в том, что в его гараже живёт дракон; но он убеждён в том, что полезно быть убеждённым в существовании дракона в гараже» в лучшем случае нереалистично. Но вполне допустимо сказать, что Кредерус рассчитывает на отсутствие дракона, и оправдывается, словно он убеждён в наличии дракона.

Человек может каждый день использовать заурядную мысленную картину своего гаража (без драконов), всегда правильно предсказывающую его ощущения после открытия двери, но ни разу в жизни не произнести про себя предложение «В моём гараже драконов нет». Это почти наверняка случалось и с тобой: открывая дверь гаража, или спальни, или чего-нибудь ещё, и ожидая не увидеть драконов, на сознательном уровне ты думаешь о чём угодно, но не о драконах.

И для того, чтобы продолжать стоять на своём и верить в дракона — или чтобы продолжать уклоняться от мысли изменения представления о себе как о верящем в дракона — совсем необязательно думать: «Я хочу верить в дракона, живущего в моём гараже». Достаточно лишь нежелания перспективы признания в ложности заявленных убеждений.

Для того, чтобы правильно готовить оправдания для будущих экспериментальных результатов, Кредерус должен, во-первых, хранить где-то в разуме правильную модель ситуации, контролирующую его ожидания, и, во-вторых, действовать и мыслить таким образом, чтобы защитить либо своё свободно плавающее убеждение о драконе, либо своё представление о себе, как о верящем в дракона.

Если Кредерус верил в убеждение о существовании дракона и вдобавок был убеждён в существовании дракона, то проблема была бы уже не так грозна. Кредерус желал бы рисковать, если речь идёт об экспериментальных предсказаниях, и, возможно, даже согласился бы отказаться от убеждения о существовании дракона, если бы его предсказания бы не сбылись (хотя, если Кредерус был бы не до конца уверен в существовании дракона, то его вера в убеждение могла бы помешать этому признанию). Однако, когда кто-то заранее оправдывается, это обычно требует расхождения убеждения и веры в убеждение.

Перевод: 
BT
Номер в книге "Рациональность: от ИИ до зомби": 
13
Аудио: 

Вера в убеждения

Аудиофайл: 
Чтец: 
Даниил Храмцов
Оцените качество перевода: 
Средняя оценка: 4.4 (33 votes)

Байесианское дзюдо

Элиезер Юдковский

Вы можете получить массу удовольствия, общаясь с людьми, чьи ожидания будущего теряют связь с их же верой в убеждения.

Как-то на одном званом обеде я пытался объяснить человеку, чем я зарабатываю на жизнь; и он сказал: «Я не верю в возможность искусственного интеллекта, ведь лишь Бог может создать душу».

Я, должно быть, движимый божественным наитием, немедленно ответил:

— То есть, если я смогу создать искусственный интеллект, то это докажет ложность вашей религии?

— Что? — сказал он.

— Ну, если ваша религия предсказывает, что я неспособен создать искусственный интеллект, то факт создания мной искусственного интеллекта будет означать неправоту вашей религии. Либо ваша религия допускает возможность создания ИИ, либо его создание опровергнет вашу религию.

Повисла пауза — он осознал, что только что сделал свою гипотезу фальсифицируемой — и затем он ответил:

— Ну, я не имел ввиду, что вы не сможете создать интеллект. Я хотел сказать, что он не может испытывать эмоции таким же образом, что и мы.

— Итак, если я создам искусственный интеллект, в который не будет заложено ничего навроде заранее написанного сценария, и моё творение начнёт говорить о чём-то, похожем на нашу духовную жизнь, то тогда ваша религия неверна.

— Ну, гм, видимо нам придётся остаться при своих мнениях на этот счёт. «Согласиться не соглашаться» — в таких случаях говорят англичане.

— На самом деле, так нельзя. Есть теорема из области рациональности — теорема Ауманна о согласии — которая говорит о том, что два рационалиста не могут согласиться не соглашаться. Если два человека не соглашаются друг с другом, то хотя бы один из них должен быть в чем-то неправ.

Мы коротко прошлись по этой теме. И, наконец, он сказал:

— Ну, кажется, на самом деле я пытался сказать вот что: я считаю, что вы не можете создать что-то вечное.

— Ну, я тоже так считаю!— ответил я. — Я рад, что мы смогли прийти к консенсусу по этому вопросу, как и требует теорема Ауманна о согласии.

Я протянул свою руку и он пожал ее, а потом побрёл дальше.

Женщина, которая стояла рядом и слушала наш разговор, серьёзно посмотрев на меня, сказала: «Это было прекрасно».

— Большое спасибо, — ответил я.

Перевод: 
BT
Номер в книге "Рациональность: от ИИ до зомби": 
14
Аудио: 

Байесианское дзюдо

Аудиофайл: 
Чтец: 
Даниил Храмцов
Оцените качество перевода: 
Средняя оценка: 4.7 (19 votes)

Притворная мудрость

Элиезер Юдковский

Самое жаркое место в аду закреплено за теми, кто во время кризиса оставались нейтральными.
Данте Алигьери, эксперт по аду. Ой, то есть Джон Ф. Кеннеди, который приписывал цитаты кому попало.

Нейтральность и неторопливость с выводами - широко известный прием, который часто применяется в тех случаях, когда люди желают выставить себя взрослыми, мудрыми, непредвзятыми, или просто показать свое превосходство над другими.

Примером этого выступают мои родители, которые отвечали на теологические вопросы вроде «Почему в Древнем Египте, несмотря на обилие хороших записей о множестве разных событий, нет записей о том, что евреи хотя бы были там?» фразами: «Когда я был в твоем возрасте, я тоже задавал такие вопросы, но теперь я это перерос».

Другой пример – директор школы, который, представ перед двумя детьми, пойманными за дракой на игровой площадке, строго говорит: «Неважно кто начал драку, важно лишь кто ее закончил». Ну разумеется важно, кто начал драку. Директор, возможно, не имеет доступа к точной информации на этот счет, но если так, то ему стоит так и сказать, а не отрицать важность того, кто же ударил первым. Представим, что родитель ударил директора – как высоко оценит суд мудрость фразы «не важно кто начал драку» в этом случае? Но для взрослых детская драка – всего лишь неудобство, и для их удобства совершенно не важно, кто ее начал. Для их удобства важно только то, чтобы она закончилась и закончилась так быстро, как только можно.

Похожая динамика, по-моему, управляет международными дипломатическими случаями, когда Великие Силы строго говорят меньшим группам прекратить конфликты прямо сейчас. Великим Силам не важно кто начал: кто спровоцировал, кто непропорционально ответил на провокацию, ведь длящееся неудобство для Великих Сил - это только всего лишь функция длящегося конфликта. Блин, почему бы Израилю и Хамасу не найти наконец общий язык?

Это я называю «притворяться Мудрым». Разумеется, есть множество способов продемонстрировать мудрость. Но демонстрировать ее отказываясь делать догадки, отказываясь анализировать свидетельства, отказываясь выносить суждения, отказываясь принимать сторону, оставаясь в стороне, взирая высокомерно и снисходительно, т.е. демонстрировать мудрость ничего не говоря и не делая - ну, это я нахожу особенно претенциозным.

Паулу Фрейре сказал: «Умывать руки во время конфликта между сильным и бессильным – значит встать на сторону сильного, а не быть нейтральным.1 Игровая площадка на которой учителей не волнует кто начал конфликт представляет собой хорошее место для хулиганов, и отвратительное для их жертв. Это касается и международной политики: мир, где Великие Силы отказываются принимать сторону и лишь требуют немедленных договоренностей – удобный мир для агрессоров и ужасный для тех, на кого эта агрессия направлена. Но, разумеется, это очень удобный мир, если вы в нем Великая Сила или школьный директор.

Так что это поведение по крайней мере отчасти можно объяснить элементарным эгоизмом со стороны Мудрого.

Однако, в то же время, это еще и выражение превосходства. В конце концов, что подумают о директоре другие взрослые, если он встанет на чью-то сторону в обыкновенной детской драке? Ну как же, это ведь понизит статус директора до обычного участника столкновения!

То же самое касается любого уважаемого старшего – исполнительного директора, влиятельного академика, основателя форума – чья репутация непредвзятого человека основывается на том, что он практически никогда не выносит суждений сам, пока другие занимают стороны. Стороны обращаются к ним за поддержкой, но почти всегда впустую. Ведь Мудрых почитают как судей только при условии, что они почти никогда не выносят суждений – ведь в противном случае они были бы рядовыми участниками спора, ничем не лучше других.

(Как ни странно, судьи в судах могут себе позволить регулярно выносить реальные приговоры без автоматической потери своей репутации непредвзятых. Может потому, что все понимают, что они должны судить, ведь это их работа. Или потому, что судьям не приходится раз за разом разрешать вопросы, разделяющие на две части племя, от чьего уважения они зависят).

Существуют случаи, в которых рационально подождать с суждениями. Это, когда люди торопятся с выводами исключительно из-за своих искажений. Как сказал Майкл Руни: «Эту ошибку я неоднократно замечал у студентов, которые начинают изучать философию: столкнувшись с причинами стать скептиками, они вместо этого становятся релятивистами». Выходит, что в случаях, когда рационально не торопиться с выводами о чем-либо, слишком многие вместо этого приходят к решению, что все выводы одинаково приемлемы.

Как мы можем избежать связанной с предыдущей, но отдельной ошибки –псевдорационального поведения, заключающегося в демонстрации вашей неискаженной непредвзятости путем необоснованного утверждения, что текущий баланс свидетельств нейтрален? «Ох, ну, разумеется, есть немало убежденных дарвинистов, но я думаю, что свидетельства не позволяют нам сделать окончательный вывод о превосходстве естественного отбора над разумным творением».

Тут я предлагаю вспомнить, что нейтральность это точное суждение. Это совсем не то же самое, что оставаться вне и над. Это выражение ясной и конкретной позиции о том, что баланс свидетельств в конкретном деле допускает только одну трактовку - нейтральную. Это суждение тоже может оказаться неверным. Вывод о нейтральности может обсуждаться так же, как и вывод о правоте любой из сторон.

Точно так же дело обстоит и с политическими вопросами. Если какие-то люди утверждают, что и у пролайф («движения в защиту жизни») и у прочойс («движения в защиту репродуктивного выбора») позиции есть разумные идеи, и им определенно стоит стремиться к компромиссу и уважению, то они не занимают позицию вне дискуссии. Они выносят конкретное суждение, настолько же конкретное как и суждения «пролайф правы» или «прочойс правы».

Кстати говоря… этот текст не является приглашением к дискуссии об абортах или Палестино-Израильском конфликте в комментариях. Этот сайт - не для этих дискуссий, есть другие места чтобы обсуждать эти полностью заслуживающие обсуждения темы. Может быть потом, когда LessWrong станет достаточно большим… но сейчас не время.

Но дело не в том, что рационалисты слишком зрелые, чтобы говорить о политике. Дело не в том, что рационалисты выше этой глупой потасовки, до участия в которой унижаются только обычные сторонники политических партий и юные энтузиасты.

Робин Хэнсон отмечает, что способность участвовать в горячих спорах - ограниченный ресурс. Если вам удастся найти, где применить те же силы с большим результатом, то с вашей стороны разумно тратить силы на обсуждения, от которых может быть больше пользы, чем на те, в которых множество участников тратит много сил.
Но в таком случае ваши приоритеты – это следствие вашей ограниченности в ресурсах. Это не значит, что вы мудро и спокойно парите над схватками, в которых решаете не участвовать.

Мой ответ Полу Грэму на Хакер Ньюс, похоже, стоит повторить здесь:

Есть разница между:

  • Принятием нейтрального суждения

  • Отказом вложить незначительные ресурсы.

  • Претензией на то, что любой из вариантов выше является признаком большой мудрости, зрелости и демонстрацией превосходства, с подтекстом, что исходные стороны конфликта - это худшие точки зрения, которые не так уж сильно отличаются с высоты вашего полета.

  • 1. Paulo Freire, The Politics of Education: Culture, Power, and Liberation (Greenwood Publishing Group, 1985), 122.
Перевод: 
Muyyd, Remlin, Greenochre, Peter Tanatarov
Номер в книге "Рациональность: от ИИ до зомби": 
15
Аудио: 

Притворная мудрость

Аудиофайл: 
Чтец: 
Даниил Храмцов
Оцените качество перевода: 
Средняя оценка: 5 (19 votes)

Претензия религии на неопровергаемость

Элиезер Юдковский

Самое раннее известное мне упоминание научного эксперимента — это, как ни иронично, история об Илии и жрецах Ваала (English).

Народ Израиля колеблется между Иеговой и Ваалом, поэтому Илия объявляет, что проведёт эксперимент, чтобы решить эту проблему— какое новаторство по тем временам! Жрецы Ваала поместят своего быка на алтарь, а Илия поместит на алтарь быка Иеговы, но никому из них не будет позволено зажечь огонь; чей Бог истинный, тот и заставит огонь сойти на Его жертву. Жрецы Ваала служат для Илии контрольной группой — такое же древесное топливо, такой же бык и такие же жрецы, возносящие молитвы, но ложному богу. Затем Илия льёт воду на свой алтарь, разрушая симметрию эксперимента, но ведь это было так давно — тем самым он обозначает добровольное принятие бремени доказательства — эквивалент современного уровня значимости в 0,05. Огонь сходит на алтарь Илии, что является экспериментальным наблюдением. Народ Израиля, кричащий: «Господь есть Бог!» — экспертная оценка.

А потом они оттащили 450 жрецов Ваала к реке Кишон и перерезали им глотки. Это сурово, но необходимо. Надо жёстко отсечь опровергнутую гипотезу — и сделать это быстро, прежде чем она сможет найти предлог для самозащиты. Если бы жрецам Ваала сохранили жизнь, они бы начали болтать, что религия на самом деле — отдельный магистерий, недоступный ни подтверждению, ни опровержению.

В былые дни люди действительно верили в то, что говорила им их религия, а не просто считали религию важной. Библейские археологи, отправившиеся искать Ноев Ковчег, не считали, что впустую тратят своё время; они предполагали стать знаменитыми. И лишь после того, как не смогли найти подтверждающих свидетельств — а нашли опровергающие — только тогда эти святоши совершили то, что Уильям Бартли назвал возвращением к убеждению: «Я верю, ибо я верую».

В былые дни не существовало концепции религии как отдельного магистерия. Ветхий Завет — это свалка культурного потока сознания: история, право, притчи о морали, и, да, модели того, как работает Вселенная. Вы не найдёте ни в одной строчке Ветхого Завета трансцендентного восхищения сложностью Вселенной. Но вы найдёте множество вполне научных заявлений (English), вроде Вселенной, созданной за шесть дней (что является метафорой Большого Взрыва), или кроликов, жующих жвачку (что является метафорой…).

В былые дни заявление о том, что местную религию «невозможно подтвердить», привело бы вас на костёр. Одно из главных верований ортодоксального иудаизма заключается в том, что Бог появился на горе Синай и произнёс громовым голосом: «Ага, это всё правда». В байесианской перспективе это, чёрт возьми, весьма однозначное доказательство существования нечеловечески могущественной сущности. (Хотя и не того, что эта сущность, собственно, Бог, или что эта сущность добродетельна — это могли быть и подростки-инопланетяне.) Абсолютное большинство религий в истории человечества — не считая придуманных совсем недавно, — рассказывают истории о событиях, которые представляли бы собой совершенно безошибочное доказательство, если бы действительно случились. Независимость религии от фактических реалий — весьма недавняя и исключительно западная концепция. Люди, создававшие оригинальные писания, даже не знали о разнице между одним и другим.

Римская империя унаследовала древнегреческую философию, установила закон и порядок в своих провинциях, вела бюрократические записи и насаждала религиозную терпимость. Новый Завет, созданный уже во времена Римской империи, благодаря этому несёт на себе своеобразную печать модерна. Вы не можете придумать историю о том, как Бог полностью уничтожает город Рим (à la Содом и Гоморра), потому что римские историки ткнули бы вас в это носом, и вы не смогли бы просто закидать их камнями.

Но в былые дни люди, придумавшие истории Ветхого Завета, могли сочинять всё, чего бы им ни захотелось. Ранние египтологи были неподдельно шокированы тем, что не нашли абсолютно никаких следов еврейских племён, когда-либо бывавших в Египте — найти записи о Десяти казнях они и не мечтали, но они хоть что-нибудь обнаружить надеялись. Как оказалось, кое-что всё-таки нашли. Они обнаружили, что в предположительное время Исхода Египет правил большей частью Ханаана. Это гигантская историческая ошибка, но, поскольку библиотек не существует, вас некому ткнуть в неё носом.

А вот в Римской империи были библиотеки. Поэтому Новый Завет не заявлял о больших красочных широкомасштабных геополитических чудесах, что было привычным для Ветхого Завета. Вместо этого Новый Завет заявляет о меньших чудесах, которые, тем не менее, помещаются в те же самые доказательные рамки. Мальчик падает на землю, у него изо рта идёт пена; причина тому — нечистый дух; резонно ожидать, что нечистый дух убежит от истинного пророка, но не убежит от шарлатана; Иисус изгоняет нечистый дух; таким образом, Иисус — истинный пророк, а не шарлатан. Это совершенно обычное байесовское рассуждение, если принять в качестве базовой предпосылку, что эпилепсия вызывается демонами (и что прекращение эпилептического припадка доказывает, что демон сбежал).

Религия высказывала своё мнение не только в фактических и научных вопросах, религия высказывала своё мнение обо всём. Религия создала кодекс законов до возникновения законодательных органов; религия записывала историю до историков и археологов; религия устанавливала сексуальную мораль до женской эмансипации; религия расписывала формы государства до конституций; и религия отвечала на все научные вопросы: от биологической таксономии до формирования звёзд. Ветхий Завет не говорит о чувстве восхищения сложностью Вселенной — он слишком занят назначением смертной казни женщинам, носящим мужскую одежду, что, по тем временам, было вполне удовлетворительным содержанием религиозного текста. Современная концепция религии как чего-то строго морального порождена тем, что все остальные сферы были отняты у неё более компетентными институтами. Мораль — это всё, что осталось.

Или, скорее, люди думают, что мораль — это всё, что осталось. Возьмите свалку культуры, какой она была 2500 лет назад. Со временем человечество безмерно продвинется вперёд и части древней культурной свалки станут уже вопиюще устаревшими. Мораль не защищена от человеческого прогресса — например, сейчас мы весьма неодобрительно смотрим на такую одобренную Библией практику, как рабство. Почему люди думают, что такая мораль всё ещё допустима?

В действительности, нет ничего несущественного в этической проблеме убийства тысяч невинных перворождённых младенцев мужского пола, совершённого с целью убедить неизбранного фараона выпустить рабов, которые, если рассуждать логически, могли быть просто телепортированы из страны. Это должно быть более вопиющим, чем сравнительно тривиальная научная ошибка в заявлении, что у кузнечиков четыре ноги. Однако если вы заявите, что Земля плоская, на вас посмотрят, как на идиота. А вот если вы скажете, что Библия — источник вашей морали, ни одна женщина не даст вам пощёчины. Для большинства людей концепция рациональности определяется тем, что, по их мнению, может сойти им с рук; они думают, что одобрение библейской морали сойдёт им с рук; так что для того, чтобы закрыть глаза на моральные проблемы Библии, нужен лишь весьма терпимый уровень самообмана. Все согласились не замечать слона в посудной лавке, и такое состояние дел какое-то время может сохраняться.

Может быть, однажды человечество продвинется дальше, и каждый, кто предложит Библию в качестве источника морали, столкнётся с тем же отношением, с каким столкнулся Трент Лотт (Trent Lott), поддержавший президентскую кампанию Строма Термонда (Strom Thurmond). И тогда скажут, что истинной сутью религии всегда была генеалогия или ещё что-нибудь.

Идея, что религия — это отдельный магистерий, который нельзя ни доказать, ни опровергнуть, — это Большая Ложь, повторяющаяся снова и снова, так что люди говорят её не вдумываясь; но, критически рассмотренная, она оказывается попросту неверной.
Это — невероятное искажение того, как религия исторически зарождалась, как писания выражают свои верования, что говорят детям для того, чтобы убедить их, и того, во что до сих пор верит большинство религиозных людей на Земле. Нельзя не восхищаться беспредельной дерзостью этой лжи, стоящей на уровне с «Океания всегда воевала с Остазией». Прокурор показывает всем окровавленный топор, а обвиняемый, шокированный на мгновение, заявляет, поразмыслив: «Но вы не можете опровергнуть мою невиновность какими-то там доказательствами — это отдельный магистерий!»

А если это не сработает, возьмите листок бумаги и намалюйте себе карточку «Бесплатный выход из тюрьмы».

Перевод: 
Максим Расчупкин
Номер в книге "Рациональность: от ИИ до зомби": 
16
Оцените качество перевода: 
Средняя оценка: 4.9 (35 votes)

Провозглашения и крики одобрения

Элиезер Юдковский

Однажды я посетил конференцию по теме «совместимы ли религия и наука?». Одна женщина-язычник с жаром рассказывала о своих представлениях о сотворении Земли: гигантская первичная корова была рождена в первичной бездне, а затем создала первичного бога при помощи слюны и языка; потомки первичного бога убили корову и сотворили Землю из её плоти, и так далее. История была длинной, подробной и более абсурдной, чем модель мира, в которой Земля покоится на спине огромной черепахи. И эта женщина явно разбиралась в науке достаточно, чтобы это понимать.

Я до сих пор не могу подобрать слов, чтобы описать, как именно говорила эта женщина. Она говорила… гордо? С самодовольством? Осознанно щеголяя собой?

Казалось, что женщина рассказывала этот миф о сотворении целую вечность (на самом деле, вероятно, прошло не более пяти минут). Странное нечто, гордость/удовлетворение/выставление себя напоказ, явно имело какое-то отношение к её знанию того, что эти убеждения были возмутительны с научной точки зрения. И она не презирала науку: она выступала за то, что наука и религия совместимы. Она даже рассказала о том, что, если взглянуть на землю, в которой жили викинги, то нетрудно понять, почему они верили в первичную бездну (этим объяснением она свела свои верования к чему-то заурядному!), но при этом всё равно настаивала на своей вере в этот миф, говоря об этом с исключительным удовлетворением.

Я не думаю, что понятие «вера в убеждение» можно растянуть настолько, чтобы покрыть это событие. Слишком странной была эта речь. Она не повторяла легенду с фанатичной верой кого-то, кому нужно подбодрять себя. Она не надеялась убедить в чём-то аудиторию, и ей не нужна была наша поддержка для того, чтобы чувствовать себя полноценной.

Деннет, автор понятия «веры в убеждение», считает, что большую часть того, что мы называем «религиозными верованиями» (или «религиозными убеждениями») стоит изучать как «религиозные провозглашения». Представим, что пришелец-антрополог изучает группу современных студентов-филологов, все из которых, кажется, считают, что Валки Вилкинсен является пост-утопистом. В этом случае правильный вопрос звучит не как «почему все они разделяют это странное убеждение?», а как «почему все они пишут это странное предложение на письменном экзамене?». Даже если предложение совершенно бессмысленно, ты всё равно знаешь, когда следует его громко пропеть.

Я думаю, что всё же несколько чересчур считать, что религиозные верования заключаются лишь в громком повторении определённых фраз: большинство людей довольно честны, и после произнесения религиозных предложений вслух чувствуют себя обязанными повторить их мысленно, чтобы эта мысль прозвучала и в сознании.

Но даже понятие «религиозных утверждений» вряд ли покрывает рассказ язычницы о своей вере в первичную корову. Если кому-то нужно произнести религиозное убеждение вслух, чтобы понравиться священнику или собрату по вере — да что там, просто, чтобы подтвердить своё представление о себе как о верующем — ему стоит притвориться верящим намного убедительнее, чем притворялась эта женщина. Пересказывая легенду с нарочито подчёркнутой гордостью, она даже не пыталась быть убедительной, даже не пыталась заставить аудиторию поверить в то, что она воспринимала свою религию всерьёз. Кажется, именно это меня и ошеломило. Несколько известных мне людей верят в свою веру касательно совершенно абсурдных вещей; но когда они страстно рассказывают о предмете своей веры в убеждения, они намного сильнее стараются убедить себя в том, что воспринимают всё это всерьёз.

Наконец, я понял, что язычница не пыталась убедить в чём-то нас и не пыталась убедить в чём-то себя. Её пересказ легенды о сотворении вообще не имел отношения к сотворению мира. Пятиминутная обличительная речь была одобрительным возгласом, что-то вроде транспаранта на футбольном стадионе. Транспарант с надписью «ВПЕРЁД СИНИЕ» не утверждает ничего о фактах и не пытается быть убедительным. Это просто кричалка.

Та странная подчёркнутая гордость… язычница словно участвовала в гей-параде обнажённой (Замечу мимоходом: не имею ничего против участия в гей-параде в обнажённом виде. Лесбиянство не относится к числу вещей, которые могут быть уничтожены правдой). Это было не просто выражением одобрения, как участие в гей-параде, это было оскорбительно эпатажным выражением одобрения, как участие в гей-параде голышом. В этом проглядывало убеждение в том, что её не смогут раскритиковать или арестовать, ведь всё это сделано во имя прославления её сообщества.

Именно поэтому для неё столь большое значение имела смехотворная абсурдность её слов. Попытка звучать более разумно эквивалентна надеванию одежды.

Перевод: 
BT
Номер в книге "Рациональность: от ИИ до зомби": 
17
Аудио: 

Провозглашения и крики ободрения

Аудиофайл: 
Чтец: 
Даниил Храмцов
Оцените качество перевода: 
Средняя оценка: 4.1 (17 votes)

Убеждение как одеяние

Элиезер Юдковский

Я уже разделил убеждения на контроллеры ожиданий, веру в убеждения, провозглашения и крики ободрения. Контроллеры ожиданий мы будем называть «полноценными убеждениями», остальные формы «неполноценными убеждениями». Полноценное убеждение может быть неверным или иррациональным (искреннее убеждение в том, что молитва исцелит больного ребёнка), но остальные формы иногда трудно вообще считать за убеждения.

Ещё один подвид неполноценных убеждений — убеждение как групповая идентификация, способ входить в сообщество. Робин Хансон использует великолепную метафору(English): люди, носящие необычную одежду в качестве своей униформы (например, риза священника или еврейская кипа), поэтому я буду называть это «убеждением как одеянием».

Зная человеческую психологию, можно сказать, что мусульмане, атаковавшие Всемирный торговый центр, без сомнения считали себя героями, защищающими истину, правосудие и Путь Ислама от ужасающих инопланетных чудовищ а-ля «День независимости». Нужно быть сильно не от мира сего — не иметь ни малейшего представления о том, как видят мир обычные люди — чтобы сказать это вслух в баре Алабамы. Американцы так не говорят. Американцы говорят, что террористы «ненавидят нашу свободу», а столкновение самолёта со зданием было «актом трусости». Нельзя говорить «героическое самопожертвование» и «террорист-смертник» в одном предложении, даже с целью правдиво показать, как видит мир Враг. Само понятие «отвага и альтруизм террориста» является одеянием Врага — поскольку об этом понятии говорит Враг. Понятие «трусость и социопатия террориста» является американским одеянием. Хочешь описать, как мир видит Враг, — забудь о кавычках; ты же не одеваешься на Хэллоуин фашистом, так?

Убеждение-как-одеяние может объяснить, почему люди могут придавать такой вес неполноценным убеждениям. Подозреваю, что вера в убеждения или религиозные провозглашения, сами по себе, с трудом порождают глубокие и мощные эмоциональные эффекты. Я не эксперт в этой области, но у меня сложилось следующее впечатление: люди, переставшие ожидать предсказанного религией будущего, пойдут на многое ради того, чтобы убедить себя в своей страстной вере, и эту отчаянность легко спутать с настоящим сильным чувством. Но всё же, это уже не тот огонь, который они носили в детстве.

С другой стороны, человеку очень легко искренне, пылко, на инстинктивном уровне принадлежать группе, болеть за любимую команду (Этот факт — фундамент надувательства под названием «Республиканцы против Демократов» и аналогичных лжедилемм в других странах, но это уже другой разговор). Идентификация с племенем — очень мощная эмоциональная сила, люди готовы за неё умереть. И после того, как человек стал членом племени, он начинает вкладывать в убеждения, которые играют роль племенной униформы, всю ту страсть, с которой он принадлежит этому племени.

Перевод: 
BT
Номер в книге "Рациональность: от ИИ до зомби": 
18
Аудио: 

Убеждение как одеяние

Аудиофайл: 
Чтец: 
Даниил Храмцов
Оцените качество перевода: 
Средняя оценка: 4.7 (12 votes)

Табличка "Аплодисменты"

Элиезер Юдковский

Во время Сингулярного Саммита 2007, один из ораторов ратовал за создание демократичного мультинационального проекта по разработке Искусственного Интеллекта. Я подошел к микрофону и задал вопрос:

Предположим, группа демократических республик сформирует консорциум по разработке ИИ; в процессе будет много политизирования: некоторые группы будут иметь необычайно сильное влияние, другие будут отодвинуты на задний план, другими словами, результат будет напоминать другие продукты современной демократии. С другой стороны, группа независимых ботаников разработает ИИ у себя в подвале. Снабдит его инструкциями опросить всё население земного шара: даже раздать мобильники всем, кто их не имеет, и выполнить то, что будет утверждено подавляющим большинством голосов. Какой из вариантов более «демократичный» и с каким вы будете чувствовать себя более спокойно?

Я хотел узнать, верит ли он в прагматическую адекватность демократического политического процесса или он верит в моральную правильность голосования.

Но оратор ответил:

Первый сценарий выглядит как редакторская статья в журнале «Reason», а второй - как сюжет голливудского фильма.

Смутившись, я спросил:

Так какой демократический процесс вы имели в виду?

Оратор ответил:

Что-то вроде Проекта Человеческого Генома (это был интернационально финансируемый проект).

Я спросил:

Как разные группы будут разрешать свои разногласия в структуре вроде Проекта Человеческого Генома?

И оратор ответил:

Я не знаю.

Благодаря этому обмену репликами, я вспомнил цитату одного диктатора или кого-то еще, которого спрашивали о его намерениях двигать его карманное государство к демократии:

«Мы полагаем, что уже находимся в демократической системе. Некоторые факторы пока отсутствуют, вроде выражения воли народа».

Суть демократии в специфическом механизме разрешения политических конфликтов. Если все группы имеют одинаковые предпочтения, то в демократии не будет нужды - мы и так будем сотрудничать. Процесс разрешения может выражаться через голосование, или выбранного представителя, или даже через чувствительного к голосованию ИИ, но выражение должно быть хоть каким-нибудь. Какой смысл призывать к «демократическому» решению, если не подразумевается какой-то механизм разрешения конфликтов?

Думаю, слово «демократия» в данном контексте не относится к какому-то конкретному предложению или убеждению. Это скорее эквивалент светящейся таблички «Аплодисменты» в студии, которая загорается как сигнал к тому, что аудитория должна начать аплодировать.

Этот случай запомнился лишь потому, что я перепутал табличку «Аплодисменты» с политическим предложением, к моему стыду. В большинстве случаев таблички «Аплодисменты» более прямолинейны и могут быть выявлены простым реверсивным тестом. Например, кто-то скажет:

«Мы должны сбалансировать риски и возможности ИИ»

Если перевернуть это высказывание, мы получим:

Мы не должны соблюдать баланс рисков и возможностей ИИ.

Поскольку обратное звучит абсурдно, следовательно, неперевернутое утверждение, возможно, нормально, подразумевая, что это не несет новой информации. Есть множество хороших причин для утверждения, которое не является информативным вне контекста. «Нам нужно соблюдать баланс рисков и возможностей ИИ» может быть темой для обсуждения; может делать ударение на важности определенных планов для баланса; может критиковать планы, не соблюдающие баланс. Связывая другие утверждения, предложение может передать новую информацию ограниченному рационалисту - но сама по себе связь может быть неочевидной. Но без уточняющих положений, утверждение не более чем табличка «Аплодисменты!»

Порой мне хочется толкнуть речь, целиком состоящую из табличек «Аплодисменты», чтобы посмотреть, сколько времени пройдет, прежде чем аудитория начнет хохотать.

Я здесь, чтобы предложить взвесить риски и возможности продвинутого Искусственного Интеллекта. Нам следует избегать рисков и, если получится, реализовывать возможности. Нам не следует беспричинно подвергаться совершенно необязательным рискам. Для достижения этих целей нам следует планировать рационально и мудро.

Нам не следует действовать из страха и паники, или поддаваться технофобии; однако, не следует и действовать в слепом энтузиазме. Нам следует уважать интересы всех партий, принимающих участие в Сингулярности. Мы должны убедиться, что преимущества продвинутых технологий будут доступны для как можно большего числа индивидов, а не для нескольких. Мы должны, по мере сил наших, избегать конфликтов с применением этих технологий; и мы должны предотвратить попадание этих технологий в руки опасных индивидов. Нам следует думать над этими вопросами до, а не после, когда уже будет поздно что-либо предпринимать…

Перевод: 
Muyyd, Remlin
Номер в книге "Рациональность: от ИИ до зомби": 
19
Аудио: 

Требование аплодисментов

Аудиофайл: 
Чтец: 
Даниил Храмцов
Оцените качество перевода: 
Средняя оценка: 5 (18 votes)

Замечая замешательство

Чтобы грамотно описать, что такое иррациональность, необходима теория, описывающая рациональность. Причём эта теория не должна состоять из расплывчатых трюизмов, которые ничего не объясняют точно. Цепочка «Замечая замешательство» раскрывает вопрос: почему полезно основывать поведение на «рациональных» ожиданиях и как это ощущается изнутри.

Автор: 
Элиезер Юдковский

Сфокусируй неуверенность

Элиезер Юдковский

Что случится с процентом по облигациям: он поднимется, опустится, или не изменится? Если ты работаешь экспертом в телепрограмме, где тебе нужно объяснить произошедшее постфактум, то тебе незачем волноваться. Какая бы из этих трёх возможностей ни реализовалась, ты всё равно сможешь объяснить, почему результат отлично вписывается в разработанную тобой теорию рынка. Нет смысла думать о том, что эти три возможности каким-то образом противоречат друг другу или несовместимы между собой, поскольку ты в любом случае на сто процентов выполнишь свою работу в качестве эксперта.

Хотя подожди. Представь, что ты только совсем недавно появился на телевидении и недостаточно опытен для того, чтобы придумывать правдоподобные объяснения на лету. Нужно заранее подготовить заметки для завтрашнего прямого эфира, а времени у тебя не так уж и много. Тогда было бы очень полезно знать, какой именно результат произойдёт на самом деле, — поднимется или опустится ли процент по облигациям — ведь тогда понадобится подготовить лишь один набор оправданий.

Увы, предвидеть будущее невозможно. Что ты собираешься делать? Ты определённо не будешь использовать «вероятности». Школа рассказала нам, что такое «вероятности»: так называются числа от нуля до единицы в тексте некоторых задач; сейчас же никто не сообщил тебе никаких чисел от нуля до единицы. Что ещё хуже, ты чувствуешь себя как-то неуверенно, а — если тебе не изменяет память — во время вычисления ответа на такие задачи у тебя никогда не было таких ощущений. Лекции по математике читаются в чистых и знакомых аудиториях — разве уместно применять математику не в стенах института, а в запутанных и непонятных жизненных ситуациях? Использовать какой-либо стиль мышления в непригодной для него обстановке — всегда плохая идея. Ясно, о «вероятностях» вспоминать не надо.

Тем не менее, у тебя есть всего лишь 100 минут для того, чтобы подготовить оправдания. Нельзя потратить все 100 минут, обдумывая сценарий «повышение», и ещё потратить все 100 минут, набрасывая реплики для сценария «понижение», и ещё потратить 100 минут на размышления о сценарии «неизменность».

Если какая-нибудь проверочная комиссия собирается исследовать твоё расходование времени, то лучше бы потратить равное количество времени на подготовку к каждому возможному исходу. Никто не сообщил тебе чисел от нуля до единицы, и поэтому у тебя на руках нет никакой документации, могущей обосновать неравные временные затраты. Ты легко можешь представить, что именно скажут тебе проверяющие: «И почему же вы работали над оправданием №3 ровно 42 минуты, мистер Финклдинджер? Почему не 41 или 43 минуты? Признайте свою необъективность! Вы отдаёте предпочтение своим субъективным любимчикам!»

Однако ты с облегчением вспоминаешь, что никакая проверочная комиссия и не собирается искать в твоих поступках повода для обвинений. Это замечательно, ведь завтра прозвучит важное объявление от Федерального Резерва Банка США, и кажется маловероятным, что цены на процент по облигациям не изменятся. У тебя нет ни малейшего желания тратить драгоценные 33 минуты на подготовку речи, которую ты не планируешь произносить.

В голове всплывают наброски объяснений: подробные рассказы о том, почему каждое событие правдоподобно вытекает из твоей теории рынка. Но почти сразу становится ясно, что сейчас правдоподобность не поможет: все исходы правдоподобны. Каждый сценарий вписывается в твою теорию рынка, но это не имеет никакого отношения к тому, как следует поделить время на подготовку. Между сотней минут и способностью вписывать события в теорию есть принципиальное отличие: первое — ограниченно, второе — нет.

И всё же… У тебя нет зацепок, но всё же ты, кажется, ожидаешь эти события с разной силой. Какие-то оправдания кажутся тебе более важными, какие-то — менее. И — восхитительная деталь — если представить что-то, делающее повышение процента более вероятным, то объяснения для сценариев понижения и неизменности кажутся уже менее нужными.

Кажется, что существует связь между тем, насколько ты ожидаешь увидеть каждый из исходов, и тем, как ты хочешь разделить время подготовки между их оправданиями. Разумеется, эту связь невозможно измерить. У тебя есть 100 минут на подготовку, но здесь и не пахнет сотней «единиц предвкушения», или чего-нибудь такого. (Хотя ты всё-таки понял, что твоя функция полезности растёт примерно как логарифм от времени, потраченного на подготовку оправдания того события, которое произойдёт на самом деле.)

Но всё же… В мысли о том, что ожидание конечно, — и конечное ожидание подобно конечному времени на подготовку объяснений, а не бесконечной способности объяснять — явно что-то есть. Возможно, имеет смысл думать об ожидании, как о каких-нибудь ресурсах: например, как о деньгах. После такого сравнения сразу же тянет подумать о том, где можно достать ещё ожидания, но это бессмысленно: сколько ожидания бы ты не раздобыл, времени на подготовку от этого не прибавится. Нет, задача решается по-другому: нужно попытаться использовать свои ограниченные запасы ожидания наилучшим образом.

Ни о чём подобном на лекциях по статистике не говорилось. Никто не рассказал, что делать, когда это чёртово ощущение неопределённости так сильно давит на мысли. Никто не рассказал, что делать, когда неизвестно никаких чисел от нуля до единицы. Хотя при чём тут это? Если уж использовать числа, то с равным успехом можно использовать любые числа: нет никаких зацепок, указывающих на то, какой раздел математики следует использовать, если здесь стоит использовать вообще хоть какой-нибудь раздел математики. Может быть, пригодятся пары чисел: число слева, число справа. Такой подход можно будет назвать «теорией Декстера-Синистера», поскольку именно так «правый» и «левый» звучат на латыни. Или что-нибудь ещё, какой-нибудь другой метод и иная аксиоматика. (Во всяком случае, число «100» — 100 минут на подготовку — точно должно где-то участвовать, это понятно)

Почему никто не открыл правил фокусировки неопределённости? Правил, позволяющих распределить ожидание таким образом, чтобы большинство ожидания попало в тот исход, который произойдёт на самом деле?

Но как будет называться это искусство? И как будут выглядеть эти правила?

Перевод: 
BT
Номер в книге "Рациональность: от ИИ до зомби": 
20
Оцените качество перевода: 
Средняя оценка: 4.7 (18 votes)

Что такое свидетельство

Элиезер Юдковский

«Предложение „снег белый“ истинно тогда и только тогда, когда снег белый» — Альфред Тарский.

«Говорить, что сущее есть и не-сущее не есть, — значит говорить истинное» — Аристотель, «Метафизика 4».

Если две эти цитаты не выглядят удовлетворительным определением «истины» — прочтите это. Сегодня я расскажу о «свидетельствах» (причём речь будет идти об убеждениях-о-фактах, говорящих о том, каким является мир, а не об эмоциях или морали. О разнице между этими понятиями сказано здесь).

Человек идёт по улице, неожиданно его шнурки развязываются. Некоторое время спустя, по какой-то непонятной причине, он становится убеждён в том, что его шнурки развязаны. Свет покидает Солнце, ударяется о шнурки и отпрыгивает прочь; некоторые фотоны входят в зрачок и попадают на сетчатку; энергия фотонов запускает волну нервных импульсов; нервные импульсы доходят до зрительной коры, где на основе оптической информации строится трёхмерная модель, распознанная как развязанные шнурки. Происходит последовательность событий — цепочка из причин и следствий — начавшаяся во внешнем мире и закончившаяся внутри мозга, в конечном итоге которой человек приобретает имеющиеся у него убеждения. На выходе этого процесса — состояние разума, которое отражает состояние шнурков.

Что такое «свидетельство»? Это событие, сцепленное с интересующей тебя сущностью последовательностью из причин и следствий. Если, к примеру, объект исследования — твои шнурки, то входящий в зрачки свет есть свидетельство, сцепленное со шнурками. Замечу, что здесь я использую слово «сцепленность» не в том смысле, что принят в квантовой механике (entanglement, в русской литературе иногда встречается как «запутанность»); две вещи «сцеплены», если благодаря соединяющей их цепочке причин и следствий, их состояния становятся скореллированными.

Не каждое воздействие создаёт нужный для свидетельства вид сцепленности. Не очень впечатляюще построить машину, гудящую, если в неё ввести выигрышный лотерейный номер, если эта машина будет точно также гудеть и на все остальные лотерейные номера. Отражённый от ботинков свет не будет полезным свидетельством касательно шнурков, если фотоны были бы одними и теми же вне зависимости от того, завязаны ли шнурки.

Говоря более формально: событие называется свидетельством о сущности А, тогда и только тогда, когда это событие проявляется по-разному в зависимости от различных состояний сущности А (говоря на языке теории информации, взаимная информация события и сущности А должна быть больше нуля).

Сцепленность заразительна, если с ней правильно обращаться, и именно поэтому людям нужны и глаза, и мозг. Если фотоны отразятся от шнурков и затем столкнутся с камнем, то камень не сильно изменится. Камень не будет сцеплен со шнурками никаким полезным с практической точки зрения образом, его состояние будет одним и тем же вне зависимости от того, были ли завязаны шнурки. Именно поэтому камни не стоит приглашать в суд в качестве свидетелей. Фотоплёнка, напротив, будет сцеплена со шнурками через отражённые от обуви фотоны, и поэтому её можно предъявить как улику. Если твои глаза и мозг работают правильно, то ты сам становишься сцеплен со своими шнурками.

Именно поэтому рационалисты так бережно относятся к, на первый взгляд, парадоксальному утверждению: «убеждение стоит того, чтобы в него верить, лишь в том случае, когда тебя, в принципе, можно убедить в него не верить». Сетчатка, чьё состояние не меняется в зависимости от того, какой в неё входит свет, — сетчатка слепого. Некоторые системы убеждений, довольно очевидно пытаясь защитить себя, утверждают, что ряд убеждений ценен лишь в том случае, когда ты веришь в них безоговорочно: что бы ты ни видел, о чём бы ты ни думал — верь! Мозг должен оставаться в том же состоянии независимо от того, какая информация входит в его недра. Отсюда выражение «слепая вера». Если то, во что ты веришь, не зависит от того, что ты видишь, — ты слеп точно так же, как и человек с пустыми глазницами.

Если твои глаза и мозг работают правильно, то твои убеждения становятся сцепленными с фактами. Рациональное мышление порождает убеждения, сами по себе являющиеся свидетельствами.

Если твой язык говорит правду, то твои рациональные убеждения — которые есть свидетельства — могут быть свидетельствами в глазах кого-то ещё. Сцепленность передаётся по цепочке причин и следствий, а слова произнесённые есть причина, и слова услышанные есть следствие. Сказав «У меня развязались шнурки» по телефону, ты делишься сцепленностью с другом.

Поэтому среди честных людей, верящих в честность друг друга, рациональные убеждения будут заразительны. Именно поэтому выглядит столь подозрительным заявление о том, что твои убеждения не заразительны: заявление о том, что ты веришь, исходя из каких-то личных причин, не распространяющихся на остальных. Если твои убеждения сцеплены с реальностью, то они должны быть заразительны среди честных людей.

Если твоя модель реальности говорит о том, что результаты работы твоих когнитивных процессов не должны быть заразительны, то твоя модель реальности говорит о том, что твои убеждения не есть свидетельства, что твои убеждения не сцеплены с реальностью. В этом случае нужно что-то исправить, и отметить свои убеждения как «ложные».

Разумеется, если ты до конца осознаёшьощущаешьсмысл всего этого, то это означает, что ты уже отметил свои убеждения как «ложные». Потому что «убеждение не сцеплено с реальностью» означает «убеждение не истинно». В ту же секунду, когда ты перестал верить в то, что «предложение „снег белый“ истинно», ты автоматически перестал верить и в то, что снег белый, или на очень глубоком уровне сломалось что-то очень важное.

Поэтому объясни, почему используемый тобой тип мышления порождает убеждения, отражающие реальность. Объясни, почему ты считаешь себя рациональным. Объясни, почему, используя подобный твоему тип мышления, люди будут думать «снег белый» если, и только если, снег белый. Если ты не веришь в то, что результаты работы твоих когнитивных процессов сцеплены с реальностью, то почему ты доверяешь результатам работы своих когнитивных процессов? Ведь это должно быть одним и тем же!

Перевод: 
BT
Номер в книге "Рациональность: от ИИ до зомби": 
21
Оцените качество перевода: 
Средняя оценка: 3.7 (Всего оценок: 51)

Свидетельство: рациональное, правовое, научное

Элиезер Юдковский

Предположим, ваш добрый друг, комиссар полиции, говорит вам по строжайшему секрету, что криминальный главарь вашего города - Вилки Вилкинсен. Как рационалист, вы обязаны поверить этому заявлению? Попробуем так: если вы возьмете и оскорбите Вилки, я назову вас безрассудным. Так как благоразумно вести себя так, как если бы Вилки имел существенно большую-чем-по-умолчанию вероятность быть криминальным главарем, следовательно, заявление комиссара полиции должно быть сильным Байесовским свидетельством.

Наша судебная система не поместит Вилки под стражу на основе заявления комиссара полиции. Оно не рассматривается как правовое свидетельство. Возможно, если упекать за решетку всех обвиненных комиссарами полиции в том, что они возглавляют преступную организацию, то вы по-началу поймаете множество боссов, плюс, тех, кто не нравился комиссарам. Власть имеет свойство портить людей с временем, так что со временем вы будете ловить все меньше реальных боссов (которые будут применять более серьезные меры для обеспечения своей анонимности) и все больше невинных (несдержанная власть привлекает коррупцию, как мед привлекает мух).

Но это не значит, что заявление комиссара полиции не является рациональным свидетельством. Оно все еще обладает однозначными отношениями правдоподобия, и ты все еще будешь безрассудным, оскорбляя Вилки. Но на социальном уровне, преследуя социальную цель, мы осознанно определяем «правовое свидетельство», как совокупность лишь конкретных типов свидетельств, например, наблюдения комиссара полиции в ночь на четвертое апреля.
Все правовые свидетельства, в идеале, должны быть рациональными свидетельствами, но не наоборот. Мы внедряем особые, сильные стандарты, прежде чем объявлять, что данное рациональное свидетельство является еще и «правовым».

Набирая эту фразу в 8:33 вечера, Pacific time, 18-го августа 2007, я ношу белые носки. Следует ли тебе, как рационалисту, верить этим словам? Да. Могу я давать показания об этом в суде? Да. Это научное заявление? Нет, ведь нет эксперимента, который бы ты мог провести, дабы верифицировать это. Наука собрана из множества обобщений, применяемых к множеству частных случаев, чтобы ты мог провести новые реальные эксперименты, которые тестируют обобщения, и следовательно, подтвердить для себя, что обобщение является правдой, не полагаясь на чей-то авторитет. Наука - публичное, воспроизводимое знание человечества.

Как и судебная система, наука - социальный процесс, включающий подверженных ошибкам людей. Мы хотим защитить фонд убеждений, которые особенно надежны. Так же мы хотим социальные правила, которые бы способствовали производству нового знания. Так что мы внедряем особые, сильные, дополнительные стандарты прежде чем объявить конкретное рациональное знание «научным», добавив его в фонд убеждений. Должен ли рационалист верить в историческое существование Александра Великого? Да. У нас есть лишь грубая картинка древней Греции, не слишком достоверная, но это лучше чем максимальная энтропия. Но мы зависим от авторитетов, Плутарха, например; мы не можем игнорировать его и проверить все сами. Историческое знание - ненаучное знание.

Должен ли рационалист верить, что Солнце взойдет 18-го сентября 2007 года? Да, но не с абсолютной уверенностью, таков уж принцип ставок. (Для педантов: стоит ли верить, что 18-го сентября 2007 года вращение Земли и её орбита относительно Солнца останутся примерно такими же?) Это заявление, которое я написал в своем эссе 18-го августа 2007-го является научным убеждением?

Отрицание применимости прилагательного «научный» к заявлению «Солнце взойдет 18-го сентября 2007-го» может показаться ошибочным. Если наука не может делать предсказания о будущих событиях, которым еще предстоит произойти, то она бесполезна; она не будет способна предсказывать результаты эксперимента перед его проведением. Предсказание о том, что Солнце взойдет, определенно, является экстраполяцией научного обобщения. Оно основано на моделях Солнечной системы, которые ты можешь подтвердить самостоятельно.

Но представь, что ты создаешь новый эксперимент для верификации предсказания №27 в новом контексте теории Х. У тебя может не быть причин полагать, что предсказание ошибочно; ты можешь лишь хотеть проверить его в новом контексте. Утверждение о «научности» этого убеждения может показаться опасным, до завершения эксперимента. Уже есть «традиционное предсказание» и «предсказание теории Х». Но если ты уже знаешь «научное убеждение» о результате, зачем осложнять себе жизнь экспериментом?

Ты начинаешь, надеюсь, понимать, почему я отождествляю Науку с обобщениями, а не с историей экспериментов. Исторические события случаются лишь раз; обобщения же применяются для множества событий. История не воспроизводима, научные обобщения - да.

Является ли мое определение «научного знания» истинным? Это не слишком хорошо сформулированный вопрос. Особые стандарты, что мы внедряем для науки имеют практическую цель. Нигде на звездах или горах не написано, что p < 0.05 является стандартом научной публикации. Сейчас многие утверждают, что 0.05 - слишком слабый стандарт; что будет полезно понизить его до 0.01 или 0.001.

Может быть будущие поколения, действуя в соответствии с убеждением, что научное знание - публичное и воспроизводимое, будут помечать как «научные» лишь те статьи, что напечатаны в бесплатных журналах. Ведь если ты требуешь плату за знание, является ли оно знанием человечества? Можем ли мы доверять результатам, если людям приходится платить, чтобы критиковать их? Действительно ли это наука?

Вопрос «Действительно ли это наука» плохо сформулирован. Является ли байесовским свидетельством журнал с закрытым доступом и ценой подписки 20000$ в год? Вместе с частными заявлениями комиссара полиции о том, что Вилки — криминальный босс, думаю я, ответ — «Да». Но следует ли канонизировать журналы с ограниченным доступом как «науку»? Должны ли мы допускать их в защищенный фонд убеждений? Как по мне, так науке послужит больше, если научным будет считаться публичное, воспроизводимое знание в фонде человечества.

Перевод: 
Muyyd
Номер в книге "Рациональность: от ИИ до зомби": 
22
Оцените качество перевода: 
Средняя оценка: 4.7 (19 votes)

Сколько свидетельств понадобится

Элиезер Юдковский

Напомню, что свидетельство — это «событие, сцепленное с интересующей тебя сущностью последовательностью из причин и следствий», а сцепленность — «событие проявляется по-разному в зависимости от различных состояний цели». Так какое количество сцепленности — сколько свидетельств — требуется для того, чтобы поддержать убеждение?

Начнём с простого вопроса (достаточно простого для того, чтобы можно было получить ответ математически): насколько нужно сцепиться с лотереей, чтобы выиграть? Скажем, есть 70 шаров, вытаскиваемых в случайном порядке, и, чтобы выиграть, нужно, чтобы совпало шесть чисел. Тогда существует 131 115 985 возможных комбинаций, и вероятность того, что произвольный лотерейный билет выиграет, равна 1/131 115 985 (это 0,0000007%). Чтобы выиграть в лотерею, необходимы свидетельства, достаточно избирательные для того, чтобы благоволить одной комбинации, а не 131 115 984 её альтернативам.

Скажем, существуют вероятностные тесты, различающие выигрышные и проигрышные билеты. Например, можно ввести комбинацию в чёрный ящик, который всегда гудит, если комбинация выигрышна, и не всегда гудит, если комбинация проигрышна. Допустим, вероятность этого лишь 1/4 (или, в байесианской терминологии, отношение правдоподобия чёрного ящика — четыре к одному: если комбинация была выигрышной, то ящик загудит с вероятностью в четыре раза больше, чем для проигрышной).

Но возможных комбинаций очень много. Если ввести в ящик 20 проигрышных комбинаций, 5 из них (в среднем) заставят его загудеть — просто из-за вероятности ошибиться в 25%. Если ввести в ящик все 131 115 985 возможных комбинаций, то ящик загудит не только после выигрышной, но и после 32 778 996 проигрышных (в среднем).

Этот чёрный ящик не позволит выиграть лотерею, но это лучше, чем ничего. Благодаря ему, вероятность выигрыша вырастает от 1/131 115 985 до 1/32 778 997. Наблюдается прогресс в деле отыскания истины внутри обширного пространства возможностей.

Теперь предположим, что можно использовать второй ящик для того, чтобы проверить комбинацию дважды, независимо. Оба ящика точно загудят на правильную комбинацию, а вероятность гудка в ответ на неправильную комбинацию — 1/4 независимо для каждого ящика, и поэтому оба ящика загудят на проигрышную комбинацию с вероятностью лишь в 1/16. Можно сказать, что суммарное свидетельство, полученное в результате двух независимых тестов, имеет отношение правдоподобия 16:1. Число проигрышных лотерейных билетов, прошедших оба теста — 8 194 749 (в среднем).

Раз всего возможно 131 115 985 лотерейных билетов, то соблазнительно сказать, что необходимы свидетельства, чья суммарная сила будет примерно 131 115 985 к 1 — то есть нужно событие (или серия событий), в 131 115 985 раз более вероятное при условии, что комбинация выигрышная, чем при условии, что комбинация проигрышная. Но на самом деле этого свидетельства хватит лишь на то, чтобы дать 50% вероятность выигрыша. Почему? Потому что, если применить фильтр этой силы к 131 миллиону проигрышных билетов, то один (в среднем) проигрышный билет его пройдёт. Выигрышный билет тоже его пройдёт, и в результате получатся два прошедших фильтр билета. Вероятность выиграть 50%, если купить можно лишь один.

Лучше посмотреть на ситуацию следующим образом. Вначале, есть 1 выигрышный билет и 131 115 984 проигрышных, поэтому шансы выиграть 1:131 115 984. Шансы ящика загудеть — 1 (для выигрышного билета) к 0,25 (для проигрышного). Умножив 1:131 115 984 на 1:0,25 , получаем 1:32 778 996. После добавления ещё ящика свидетельств, шансы опять умножаются на 1:0,25 , и теперь они равны 1 к 8 194 749: 1 выигрышный билет и 8 194 749 проигрышных.

Удобно измерять свидетельства в битах — не в тех битах, которые можно найти на жёстком диске, а в математических битах, которые концептуально от них отличаются. Эти биты — просто логарифмы вероятностей по основанию 1/2. Например, если возможны четыре случая — A, B, C и D, чьи вероятности 50%, 25%, 12,5% и 12,5% соответственно, и я говорю, что случилось D, то тем самым я передаю тебе 3 бита информации, так как вероятность сообщённого результата — 1/8.

Удачное совпадение: 131 115 984 чуточку меньше, чем 2 в 27-й степени. Поэтому 14 ящиков, или 28 бит свидетельствующей информации — событие, в 268 435 456 раз более вероятное при условии, что гипотеза-о-билете верна, чем при условии, что она ложна, — увеличит шансы с 1:131 115 984 до 268 435 456:131 115 984, что примерно равно 2:1. Шансы 2:1 означают, что на каждые две победы приходится один проигрыш, то есть, если взять в руки 28 битов свидетельствующей информации, то вероятность выигрыша будет 2/3. Добавим ещё один ящик, 2 бита свидетельствующей информации, и шансы сдвинутся до 8:1. Появление ещё двух ящиков превратит шансы выигрыша в 128:1.

Так что, если ты хочешь получить право на сильное убеждение в том, что ты выиграешь лотерею (то есть, скажем, чтобы вероятность твоей неправоты была меньше 1%), то 34 бит свидетельствующей информации о выигрышной комбинации вполне достаточно.

В общем случае, для ответа на вопрос «сколько свидетельств для этого понадобится?» нужно использовать примерно такие же правила оценки. Чем больше пространство возможностей, или чем сильнее априорная невероятность гипотезы по сравнению с её ближайшими соседями, или чем более уверенным хочется быть, тем больше нужно свидетельств.

Правила нельзя обмануть. Никто не может формировать убеждения, основываясь на неадекватных свидетельствах. Скажем, у тебя есть ряд из 10 ящиков, и ты вбиваешь комбинации в каждый из них. Ты не можешь остановиться на первой комбинации, успешно прошедшей все ящики, и сказать: «Но шанс на то, что это случится для проигрышного билета — один к миллиону! Чёрт с этими полурелигиозными обычаями байесианцев, я закончил!». Этот тест пройдёт не только победитель, но ещё и 131 проигрышный билет (в среднем). Ты пришёл к слишком сильному выводу, основываясь на недостаточном количестве свидетельств, не сумев побороть громадность пространства возможностей и априорную невероятность. Это не надуманное бюрократическое предписание, это математика.

Конечно, можно быть убеждённым в чём-то, основываясь на неадекватных свидетельствах, если сильно хочется; но убеждения при этом не могут быть истинными. Ситуацию можно сравнить с попыткой завести машину без бензина, игнорируя глупое, закостенелое, несправедливое и смехотворное правило «автомобилю нужен бензин для того, чтобы ездить». Было бы намного удобнее и дешевле, если бы люди отменили этот закон, разве это не очевидно вообще всем? Что же, можно попробовать, если сильно хочется. Можно даже закрыть глаза и представить себе, что машина движется. Но для того, чтобы на самом деле прибыть к правдивым убеждениям, необходимы свидетельства-бензин и, чем дальше ехать, тем больше бензина понадобится.

Перевод: 
BT
Номер в книге "Рациональность: от ИИ до зомби": 
23
Оцените качество перевода: 
Средняя оценка: 3.9 (44 votes)

Самоуверенность Эйнштейна

Элиезер Юдковский

В 1919 году сэр Артур Эддингтон возглавил экспедиции в Бразилию и на остров Принсипи, чтобы пронаблюдать солнечные затмения и тем самым опытным путем проверить то, что предсказывает новая теория, созданная Эйнштейном, — общая теория относительности. Некий журналист спросил Эйнштейна, что тот будет делать, если наблюдения Эддингтона разойдутся с предсказаниями теории. Как известно, Эйнштейн ответил: «Тогда мне будет жаль Господа Бога. Теория всё равно верна».

Это заявление звучит чрезмерно дерзко, словно бросая вызов общепринятой в Традиционной Рациональности позиции, которая утверждает, что эксперимент — главный судья. Эйнштейн словно был одержим столь великой гордыней, что отказывался преклонить голову перед тем, что говорит мироздание, как это должен делать всякий ученый. Кто способен узнать, верна ли теория, еще до экспериментальной проверки?

Конечно, Эйнштейн оказался прав. Я стараюсь не подвергать критике людей, когда они правы. Если они по-настоящему ее заслуживают, мне не придется долго ждать случая, который прояснит их ошибку.

И Эйнштейн, возможно, был не столь уж опрометчиво дерзок, как это звучало.

(От переводчика: далее под силой эксперимента или сложностью гипотезы будет иметься в виду их разрешающая способность в битах в соответствии с подходом Шеннона. Для тех, кто слабо знаком с теорией информации, можно, не вдаваясь в детали, сказать, что это мера длины кратчайшего сообщения, описывающего гипотезу. См. также статью о бритве Оккама)

Чтобы назначить вероятность, большую 50%, одной верной гипотезе из набора в 100 млн возможных, вам нужно как минимум 27 бит свидетельств, или около того. Если у вас нет столь информативного способа проверки, нельзя рассчитывать, что вы сможете найти верную гипотезу: недостаточно сильные эксперименты оставят более чем одну идею потенциально истинной. Если вы попробуете произвести проверку, дающую ложноположительный исход в одном случае из миллиона (т. е. силой примерно в 20 бит), то в итоге получите сотни возможных гипотез. Чтобы просто отыскать верный ответ в широком пространстве возможностей, нужно много свидетельств.

Традиционная Рациональность подчеркивает роль подтверждения: «Если вы хотите убедить меня в истинности X, вам потребуется предоставить мне Y свидетельств». Я часто соскальзывал к подобной формулировке, когда на самом деле хотел сказать что-то вроде «Чтобы обосновать убежденность в этом заявлении с вероятностью большей 99%, нужно 34 бита свидетельств». Или «для того, чтобы присвоить вашей гипотезе вероятность больше 50%, вам нужно 27 бит свидетельств». Традиционная формулировка подразумевает, что вы начинаете исследование с догадок или неких только вам понятных рассуждений, которые приводят вас к гипотезе, и только затем накапливаете «свидетельства», подтверждающие ее, чтобы убедить научное сообщество или обосновать свою убежденность.

Однако с байесовской точки зрения вам, чтобы просто задать гипотезу на пространстве возможных теорий, нужны свидетельства в объеме, примерно равном сложности этой гипотезы. (Вопрос пока не в том, чтобы убедить кого-либо или обосновать что-либо.) Если перед вами сто миллионов альтернатив, вам нужно не меньше 27 бит свидетельств, чтобы просто однозначно сосредоточиться на единственной версии.

Это справедливо, даже если вы называете свою идею «догадкой» или «озарением». Работа интуиции — реальный процесс в настоящем мозге. Если ваш разум не обладает хотя бы десятью битами байесовски непротиворечивых, неизбыточных и соответствующих гипотезе данных, то он не в состоянии выделить корректную гипотезу силой в 10 бит — ни сознательно, ни подсознательно, ни как-либо еще. Если вы хотите отыскать одну из миллионов целей с помощью только лишь 19 бит связанной информации, подсознание не сможет сделать это лучше, чем сознание. Подсознательные догадки могут казаться загадочными тому, в чью голову приходят, но не в состоянии нарушить принципы устройства мироздания.

Вы уже видите, к чему я веду: в момент, когда Эйнштейн изначально формулировал гипотезу, когда уравнения начали приходить ему в голову, у него уже должны были быть достаточные экспериментальные данные, чтобы его внимание смогло сосредоточиться единственным образом именно на уравнениях ОТО. Иначе они не получились бы верными.

Теперь подумаем, насколько похоже на правду, что Эйнштейн мог владеть именно такими экспериментальными данными, чтобы ОТО завладела его вниманием, но ее достоверность была бы оценена лишь в 55%? Предположим, что сложность гипотезы ОТО — 29,3 бита. Правдоподобно ли, чтобы в курсе физики, который изучал Эйнштейн, было ровно 29,5 бит свидетельств?

Неправдоподобно. Если у Эйнштейна было достаточно экспериментальных данных, чтобы единственным способом разработать уравнения ОТО в самом начале, то, вероятно, у него уже было достаточно свидетельств, чтобы быть чертовски уверенным в истинности общей теории относительности.

Из-за того, что мозг человека — несовершенный обработчик информации, на деле у Эйнштейна, возможно, было чрезмерно больше свидетельств, чем в принципе требуется идеальному байесовскому агенту, чтобы присвоить ОТО внушительную степень доверия.

Слова учёного «Тогда мне будет жаль Господа Бога, теория всё равно верна» не звучат так уж пугающе, если вы взглянете на них с этой точки зрения и будете помнить, что из всего пространства вариантов именно общая теория относительности оказалась справедливой.

Перевод: 
Quilfe, Ira1985, Роман Биккулов
Номер в книге "Рациональность: от ИИ до зомби": 
24
Оцените качество перевода: 
Средняя оценка: 3.4 (40 votes)

Бритва Оккама

Элиезер Юдковский

Чем сложнее объяснение, тем больше свидетельств необходимо, чтобы просто определить его в пространстве убеждений (в Традиционной Рациональности это формулируется вводящим в заблуждение образом, скажем, «чем сложнее утверждение, тем больше требуется оснований, чтобы его принять»). Как можно измерить сложность объяснения? Как определить, сколько свидетельств потребуется?

Допустим, вы, проведя какие-то эксперименты, получили ряд интересных результатов. Почему эти данные выглядят именно так, а не иначе? На ум приходят несколько объяснений, но какое из них выбрать?

Кажется, пришло время вспомнить принцип бритвы Оккама, точнее, следующую его формулировку: «следует считать верным самое простое объяснение, не противоречащее собранным данным». Но как оценить степень простоты? Роберт Хайнлайн как-то заявил, что самое простое объяснение звучит так: «Женщина, живущая дальше по улице — ведьма, значит это сделала она».

Становится понятно, что длина предложения на естественном языке — не очень хороший способ измерить «сложность». И нельзя утверждать, что теория «вписывается» в факты просто потому что не может опровергнуть их - этого недостаточно.

Но в чём причина того, что длина предложения — плохая мера сложности? Потому что, произнося предложение, ты используешь обозначения для понятий, которые знает слушатель, и именно в них слушатель уже хранит сложность. Скажем, можно превратить предложение Хайнлайна в аббревиатуру «ЖЖНВТСО!», тогда всё объяснение можно сообщить одним словом. Или, ещё лучше, можно дать предложению короткий произвольный код навроде «фнорд!». Уменьшают ли эти действия сложность? Нет, потому что тогда собеседнику нужно заранее сказать, что «ЖЖНВТСО!» означает «Женщина, живущая напротив — ведьма, так сделала она». «Ведьма», в свою очередь, тоже обозначение для ряда очень необычных утверждений, и то, что все знают, каких именно, не означает, что «ведьма» — это просто.

Гигантский электрический искровой разряд падает с неба, сжигая дерево, и древние скандинавы говорят: «Наверное, какая-то могущественная личность разгневалась и бросила в дерево молнию». Человеческий мозг — самый сложный артефакт во всей известной вселенной. Гнев выглядит простым лишь потому, что мы не видим всей паутины нейронов, отвечающей за эту эмоцию (Представь, как трудно было бы объяснить пришельцам без чувства юмора, почему мы смеёмся над «Летающим цирком Монти Пайтона». Но это не говорит, что люди лучше пришельцев — у людей нет ощущения фнордотоватости). Сложность гнева, и, конечно, сложность разума, не бросилась в глаза авторам гипотезы о Торе, агенте-швыряющим-молнии.

Чтобы человек понял гипотезу Тора, нужно всего лишь бросить пару фраз. Чтобы человек понял уравнения Максвелла, нужно пересказать ему несколько книг. У людей есть встроенное понятие «гнев», но нет встроенного понятия «дифференциальное исчисление». Придётся объяснять язык, и язык, лежащий за языком, и основы математики, и лишь потом можно начинать лекцию об электричестве.

И всё же кажется, что в каком-то смысле уравнения Максвелла проще, чем человеческий мозг, или чем швыряющий-молнии-агент.

Вот разгадка: как выяснилось, намного проще написать компьютерную программу, симулирующую уравнения Максвелла, чем компьютерную программу, симулирующую пронизанный эмоциями разумный мозг Тора.

В алгоритмической теории информации «сложность описания» измеряется длиной кратчайшей компьютерной программы, выводящей это описание. Прежде чем говорить о «кратчайшей компьютерной программе», нужно задать пространство компьютерных программ, для чего нужен язык и интерпретатор. Индукция Соломонова использует машины Тьюринга (точнее, последовательности битов, задающие машины Тьюринга). Что делать, если тебе не нравятся машины Тьюринга? Можешь заплатить некоторый фиксированный штраф за сложность и спроектировать универсальную машину Тьюринга, которая будет интерпретировать любой код на том языке, который тебе нравится. Штраф за сложность зависит лишь от размера универсального интерпретатора для выбранного языка программирования, и поэтому различные формулировки в некотором смысле совершенно равносильны.

На мой взгляд, лучшая формулировка индукции Соломонова — требующая, чтобы компьютерная программа делала не детерминистическое предсказание, а приписывала строкам вероятности. Например, программа, объясняющая поведение симметричной монеты, будет просто приписывать одинаковую вероятность всем $2^N$ строкам длины $N$. Как понимать «объясняющая поведение» или «не противоречащая данным»? Чем больше вероятность, которую программа приписывает полученным данным, тем лучше программа их «объясняет». И сумма всех вероятностей должна равняться единице, и поэтому, чтобы лучше «объяснить» одну возможность, программа должна забрать сколько-то вероятностной меры у другой возможности, и теперь она будет «объяснять» её хуже. Монета не может в 100% случаев выпадать орлом, и в 100% случаев выпадать решкой.

Что можно сказать про оптимальный компромисс между сложностью программы и её способностью объяснять данные? Если забыть о сложности и думать только об объяснении, то лучшими будут программы, предсказывающие данные детерминистически, то есть приписывающие им 100% вероятность. Если монета выпала «ОРРООР», то программа, заявляющая, что монета фиксирована и изначально должна была показать «ОРРООР», объясняет данные в 64 раза лучше, чем программа, считающая монету симметричной. С другой стороны, если рассматривать только сложность, то гипотеза о симметричной монете всегда проще любой другой гипотезы. Даже если монета выпадает «ОРООРОООРООООРОООООР…». Гипотеза «монета симметрична» действительно проще и объясняет эту последовательность точно также хорошо, как и любую другую последовательность из 20 бросков — не лучше и не хуже — но легко увидеть другую гипотезу, выглядящую не слишком уж сложной, и объясняющую эти наблюдение намного лучше.

Программа, которой позволили хранить дополнительный бит информации, способна в два раза урезать пространство возможностей, и, следовательно, приписать в два раза больше вероятности точкам в оставшемся пространстве. Отсюда выходит, что один бит сложности должен стоить как минимум двукратного улучшения способности объяснять. Поэтому программа, в явном виде хранящая инструкцию «приписать ОРРООР 100% и 0% всем остальным», не сможет выиграть у всех остальных программ. Шесть бит, отведённые на хранение «ОРРООР» сводят на нет всю достоверность, полученную 64-кратным улучшением способности объяснять. Иначе, рано или поздно, придётся решить, что все симметричные монеты фиксированы.

Если, конечно, эта программа не написана умно, и не пытается сжать строки данных. Во всех остальных случаях перемещение информации из данных в код не помогает укрепить достоверность программы.

Как именно работает индукция Соломонова? Нужно расcмотреть все допустимые программы (если допустима любая программа, то индукция становится невычислимой), причём каждая программа имеет априорную вероятность, равную $(1/2)^N$, где $N$ — её длина в битах, а затем вероятность корректируется, исходя из того, насколько хорошо программа объясняет данные на текущий момент. В результате получается группа «экспертов» различной степени достоверности, могущая предсказывать следующие биты: просто просуммируй мнения, умножив их на весовой коэффициент авторитета.

Принцип минимальной длины сообщения почти эквивалентен индукции Соломонова. Сначала ты посылаешь строку, описывающую код, а затем строку, описывающую данные, используя этот код. Объяснение, создающее кратчайшее суммарное сообщение, считается лучшим. Если приравнять набор возможных кодов к пространству всех компьютерных программ и считать сообщение-с-определениями универсальным интерпретатором, то принцип минимальной длины сообщения почти эквивалентен индукции Соломонова (почти — потому, что он выбирает кратчайшую программу, а не суммирует все возможные программы).

Это позволяет яснее увидеть проблему с использованием фразы «женщина, живущая напротив — ведьма, так сделала она» для объяснения закономерности в последовательности «0101010101». Если ты отправляешь другу письмо, пытаясь описать последовательность, которую ты наблюдал, тебе придётся сказать: «женщина, живущая напротив — ведьма, она сделала так, что последовательность вышла 0101010101». Обвинения в колдовстве не позволили сократить вторую половину сообщения. Тебе по-прежнему нужно описать, во всех подробностях, порождённые её запретным искусством данные.

Колдовство объясняет известные данные в том смысле, что оно качественно их разрешает. Но это лишь потому, что колдовство позволяет вообще всё, как и флогистон. Поэтому, после того, как слово «ведьма» сказано, тебе всё равно предстоит описать все наблюдения, не упуская даже мельчайшей детали. Посылая сообщение о колдовстве, ты не сжимаешь сообщение с данными. Первое сообщение — бесполезный пролог, мёртвый груз, увеличивающий суммарную длину.

Подвох фразы «так сделала ведьма» спрятан в слове «так». Как именно сделала ведьма?

Конечно, благодаря эффекту знания задним числом, якорению, лжеобъяснениям, лжепричинности, предвзятости подтверждения и целенаправленным размышлениям, может казаться совсем очевидным то, что, если женщина ведьма, то, конечно же, она заставит монету выпасть 0101010101. Но это уже отдельный разговор.

Перевод: 
BT
Номер в книге "Рациональность: от ИИ до зомби": 
25
Оцените качество перевода: 
Средняя оценка: 3.4 (94 votes)

Сила рационалиста

Элиезер Юдковский

(Этот случай произошёл ещё в те давние седые времена, когда я посещал IRC-чаты. Время затуманило память и мой рассказ может быть неточным)

Итак, дело происходило в IRC-чате. Один из посетителей спрашивает совета у сведущих в медицине людей. Его друг обратился к нему со следующей историей: у него начались внезапные боли в грудной клетке, поэтому бедняга вызвал скорую, и скорая приехала, но медработники сказали «ничего страшного» и уехали, боль в груди же становится всё сильнее и сильнее. «Что мне делать?» — спрашивает он у людей в чате, пересказав эту историю.

Эта история сбила меня с толку. Я помню, как я читал о бездомных Нью-Йорка, вызывающих скорую только для того, чтобы оказаться где-нибудь в тёплом месте, и о медиках, вынужденных отвозить их в пункт неотложной медицинской помощи. Даже на 27-й итерации, ведь, в противном случае, медкомпания может быть засужена на очень серьёзную сумму денег. Аналогично, пункты неотложной помощи юридически обязаны лечить всех, вне зависимости от их платежеспособности (Эти серьёзные расходы ложатся на плечи госпиталя, поэтому госпитали закрывают свои пункты неотложной помощи… В связи с этим мне очень интересно узнать, в чём смысл обучать экономистов, если мы всё равно собираемся их игнорировать?).

Так что я не совсем понимал, как могли произойти описанные события. Любого жалующегося на боль в груди человека, должны были бы немедленно увезти на скорой.

А затем я потерпел неудачу как рационалист. Я вспомнил несколько случаев, когда мой доктор совершенно отказывался паниковать в ответ на сообщения о симптомах, которые, на мой взгляд, были крайне тревожными. И медицинское учреждение всегда оказывалось правым. Каждый раз. Боли в грудной клетке как-то были и у самого меня, но доктор терпеливо разъяснил мне, что я описываю мышечную боль, а не инфаркт.

Поэтому я написал в чате: «Что же, если врачи сказали «ничего страшного», то это действительно так и есть — они бы увезли больного в госпиталь, если бы его состояние грозило бы хоть чем-нибудь серьёзным».

Таким вот способом я всё же впихнул историю в уже существующую модель, хотя в глубине души ощущал, что объяснение немного натянуто…

Некоторое время спустя этот товарищ возвращается в чат и сообщает, что его друг целиком всё выдумал: от болей в груди до отказа врачей помочь. Очевидно, это был не самый честный его друг.

И лишь в эту секунду я осознал то, что должен был понять сразу же: слова неизвестного знакомого знакомого по IRC-каналу могут быть не так достоверны(English), как опубликованная в журнале статья. Увы, вера легче неверия; мы верим инстинктивно, но неверие требует сознательного усилия(English).

Но вместо того, чтобы заподозрить розыгрыш, я, сильно постаравшись, заставил свою модель реальности объяснить аномалию, которая никогда не происходила. И я знал, насколько постыдны подобные поступки. Я знал, что полезность модели измеряется не тем, что она может объяснить, а тем, что она объяснить не может. Ничего не запрещающая гипотеза позволяет всё, тем самым терпя неудачу в попытке упорядочить ожидания будущего.

Cила рационалиста состоит в способности быть озадаченным вымыслом больше, чем реальностью. Если ты одинаково хорошо объясняешь любой исход, то знаний у тебя — ноль.

Временами все мы слабы. Тогда я был способен быть сильнее, но, увы, совершил ошибку. У меня была вся информация, необходимая для правильного ответа, я даже заметил проблему — а затем я её проигнорировал. Замешательство было Подсказкой, а я выбросил свою Подсказку.

Ощущение натянутости заслуживает львиной доли внимания.

Замешательство — важная подсказка на пути к истине, часть твоей силы, силы рационалиста. Серьёзный дизайнерский недочёт человеческого мышления заключается в том, что это ощущение лишь тихо шуршит на самой границе восприятия, вместо того, чтобы под вой сирен вешать яркую неоновую надпись «ЛИБО ТВОЯ МОДЕЛЬ НЕВЕРНА, ЛИБО ЭТА ИСТОРИЯ ЛОЖНА».

Перевод: 
BT
Номер в книге "Рациональность: от ИИ до зомби": 
26
Оцените качество перевода: 
Средняя оценка: 3.6 (86 votes)

Отсутствие свидетельств — свидетельство отсутствия

Элиезер Юдковский

Запоздалое впихивание свидетельств в гипотезу сыграло свою роль в самой горестной главе истории Соединенных Штатов, интернировании японцев в начале Второй Мировой. 21 февраля 1942 года Эрл Варрен, губернатор Калифорнии, в ответ на замечание об отсутствии случаев саботажа, шпионажа или иной подрывной деятельности живущих в Америке японцев, сказал:

«Я придерживаюсь мнения, что это отсутствие является самым зловещим во всей этой ситуации. Больше чем что-либо ещё, это убеждает меня в том, что будущие саботажи, будущие действия Пятой Колонны будут назначены на определённое время, точно так же, как на определённое время был назначен Перл Харбор… Я считаю, что нам внушают лживое ощущение безопасности» — Робин Дэйвс, «Rational Choice in an Uncertain World».

Рассмотрим утверждение Варрена через призму теоремы Байеса. Когда мы видим свидетельство, распределение вероятностей между гипотезами меняется: вероятность гипотез, которые считали появление такого свидетельства более правдоподобным, увеличивается за счёт вероятности гипотез, которые считали появление такого свидетельства менее правдоподобным. На исход влияют лишь относительные отношения правдоподобия и вероятности: можно приписать свидетельству очень большое правдоподобие, но всё равно потерять вероятностную меру из-за того, что какая-то другая гипотеза приписала этому свидетельству ещё большее правдоподобие.

Варрен, похоже, утверждает, что отсутствие саботажа закрепляет его убеждение о существовании Пятой Колонны. Да, возможно, Пятая Колонна совершит саботаж потом. Но вероятность того, что отсутствие саботажа совершила существующая Пятая Колонна ниже вероятности того, что отсутствие саботажа совершила несуществующая Пятая Колонна.

Пусть E — наблюдение отсутствия саботажа, H1 — гипотеза о американо-японской Пятой Колонне и H2 — гипотеза о том, что её не существует. Чему бы ни равнялась вероятность того, что Пятая Колонна не совершит саботажа (величина P(E|H1)), она не может быть больше вероятности того, что отсутствие Пятой Колонны не совершит саботажа (величины P(E|H2)). Поэтому наблюдение отсутствия саботажа увеличивает вероятность того, что Пятой Колонны не существует.

Отсутствие саботажа не доказывает, что Пятой Колонны не существует. Отсутствие доказательства — не доказательство отсутствия. В логике A->B, «из А следует B» не эквивалентно ~A->~B, «из не-А следует не-B».

Но в теории вероятности отсутствие свидетельствасвидетельство отсутствия. Если E — бинарное событие и P(H|E) больше P(H), «наблюдение E увеличивает вероятность H», то P(H|~E) меньше P(H), «неудачное наблюдение E уменьшает вероятность H». P(H) — это взвешенное среднее P(H|E) и P(H|~E), и поэтому она обязательно лежит между ними.

В большинстве случаев, которые встречаются в реальном мире, явление не обязано постоянно создавать свидетельства своего существования, но ждать этих свидетельств от отсутствия этого явления ещё более безнадёжно. Отсутствие наблюдений может быть как сильным свидетельством отсутствия, так и очень слабым свидетельством отсутствия — зависит от вероятности, с которой явление создаёт эти наблюдения. Отсутствие довольно слабо разрешённого события (пусть даже альтернативная гипотеза не разрешает его вообще) — довольно слабое свидетельство отсутствия (но всё же свидетельство). В этом заключается ошибка креационистов, ссылающихся на «пробелы в летописи окаменелостей»: окаменелости формируются редко, и поэтому бессмысленно праздновать отсутствие наблюдения, слабо разрешённого теорией, достоверность которой уже установлена множеством сильных положительных наблюдений. Однако, если не зафиксировано вообще ни одного положительного наблюдения — время беспокоиться; отсюда и парадокс Ферми.

Cила рационалиста состоит в способности быть озадаченным вымыслом больше, чем реальностью. Если ты одинаково хорошо объясняешь любой исход, то знаний у тебя — ноль. Сила модели измеряется не тем, что она может объяснить, а тем, что она объяснить не может — только запреты могут упорядочить ожидания будущего. Если ты не замечаешь, вероятность каких наблюдений твоя модель уменьшает, то ты с тем же успехом можешь выбросить эту модель, и с тем же успехом можешь жить без этих наблюдений; без мозга и без глаз.

Перевод: 
BT
Номер в книге "Рациональность: от ИИ до зомби": 
27
Оцените качество перевода: 
Средняя оценка: 4 (19 votes)

Закон сохранения ожидаемых свидетельств

Элиезер Юдковский

Фридрих Шпее фон Лангенфельд, духовник приговорённых к смерти ведьм, в 1631 году написал книгу «Cautio Criminalis» («Предосторожность касательно преступлений»), в которой он язвительно описал древо принятия решения о приговоре обвинённой в колдовстве: если ведьма вела злую и грешную жизнь, то это говорило о её вине; если она вела добрую и благочестивую жизнь, то это тоже было доказательством, поскольку ведьмы, скрываясь, пытаются притвориться образцами добродетели. После того, как женщину привели в тюрьму: если она была испугана, то она была виновной; если она не была испугана, то это подтверждало её вину, поскольку ведьмы, стараясь казаться невинными, натягивают храбрую мину. Услышав обвинение в колдовстве, женщина может попытаться спастись бегством: если она убегает, то она виновна; если она остаётся на месте, то её ноги сковал дьявол.

Шпее давал последние напутствия более двумстам осуждённым ведьмам. У него имелась возможность посмотреть на каждую ветвь дерева обвинений и увидеть, что абсолютно любые слова или действия обвинённой лишь укрепляли уверенность инквизиторов в её вине. Однако в каждом отдельном случае люди видели только одну ветвь дилеммы. Именно поэтому учёные формулируют свои экспериментальные предсказания заранее.

Но нельзя получить и то, и другое. «Нельзя» в смысле «математически невозможно», а не просто «нечестно». Правило «отсутствие свидетельства есть свидетельство отсутствия» — частный случай более общего утверждения, которое я называю законом сохранения ожидаемых свидетельств: ожидаемая апостериорная вероятность с учётом будущего свидетельства должна равняться априорной вероятности.

P(H) = P(H)
P(H) = P(H,E) + P(H,~E)
P(H) = P(H|E) ∙ P(E) + P(H|~E) ∙ P(~E)

Или, перенеся P(H) на другую сторону: (P(H|E) − P(H)) ∙ P(E) + (P(H|~E) - P(H)) ∙ P(~E) = 0, ожидаемое изменение вероятности — ноль.
Следовательно, для каждого ожидаемого свидетельства в пользу, существует равное и противоположно направленное ожидание свидетельства против.

Если имеется высокая вероятность получения слабого свидетельства в одну сторону, то она компенсируется низкой вероятностью получения сильного свидетельства в другую сторону. Если ты очень уверен в своей теории, и поэтому ожидаешь увидеть предсказанный результат, то исполнение предсказания лишь самую чуточку усиливает убеждённость в этой теории (эта убеждённость и без того близка к 1), однако неожиданная неудача нанесёт уверенности сильный удар, как и должно быть. В среднем, твоя убеждённость остаётся совершенно неизменной. Аналогично, одно лишь ожидание встретить свидетельство — до того, как ты увидел, в чём именно оно заключается — не должно сдвигать твоей априорной убеждённости.

Поэтому, заявляя, что отсутствие саботажа подтверждает существование японско-американской Пятой Колонны, человек должен подразумевать, что наличие саботажа опровергает существование Пятой Колонны. Если доброта и благочестие — свидетельство того, что женщина является ведьмой, то злоба и грех должны быть свидетельством её невиновности. Если Господь, проверяя нашу веру, отказывается явить Себя нашим глазам, то описанные в Библии чудеса должны разубеждать нас в существовании Бога.

Звучит как-то неправильно, ведь так? Прислушивайся к ощущению натянутости, внимательно ищи это тихое напряжение на границе восприятия. Это важно.

Истинный байесианец не может искать свидетельства в пользу теории. Не может существовать ни чёткого плана, ни умной стратегии, ни хитрого приёма, с помощью которых можно проводить эксперименты, систематически убеждающие всех в каком-либо утверждении. Нельзя поставить эксперимент, который подтвердит теорию; эксперименты могут лишь испытывать теорию.

Осознав это, можно ощутить: дышится намного легче. Не надо мучиться, пытаясь интерпретировать каждый возможный исход эксперимента так, чтобы он подтверждал твою теорию. Не надо обдумывать, как заставить каждую йоту свидетельств подтверждать твою теорию, ведь для каждого ожидания свидетельства в пользу, существует равное и противоположно направленное ожидание свидетельства против. Можно ослабить силу укуса возможного «аномального» наблюдения, лишь ослабив поддержку от «нормального» наблюдения; сила среднего укуса всегда в точности равна силе средней поддержки. Это игра с нулевой суммой. Как бы ты ни спорил, как бы ты ни сотрудничал с Тёмной Стороной, какие бы будущие стратегии ты ни вырабатывал, — ты не можешь рассчитывать, что будущее сдвинет твои взгляды в определённую сторону.

С тем же успехом ты можешь просто сесть, расслабиться, и ждать, пока твои свидетельства сами не придут к тебе.

…временами я ужасаюсь извращённости человеческой психики.

Перевод: 
BT
Номер в книге "Рациональность: от ИИ до зомби": 
28
Оцените качество перевода: 
Средняя оценка: 4 (26 votes)

Знание задним числом обесценивает науку

Элиезер Юдковский

Этот отрывок(English) из книги Дэвида Майерса «Изучаем социальную психологию»1, стоит того, чтобы прочитать его полностью. Каллен Мерфи, издатель журнала «Atlantic», заметил, что социальные науки не открывают ничего, что нельзя было бы «найти в цитатниках заранее… День за днем ученые-социологи выходят в мир. И день за днем они открывают, что поведение людей очень похоже на то, что ты и так мог предсказать и ожидал увидеть».

Конечно же, всё это «ожидание увидеть» вытекает из эффекта знания задним числом (эффект знания задним числом: знающие ответ на вопрос люди считают его более очевидным, чем люди, пытающиеся угадать ответ, не зная его заранее; «разумеется, я додумался бы до этого!»).

Историк Артур Шлезингер-младший называл социологические исследования американских солдат времен второй мировой войны «нудной демонстрацией» здравого смысла. Например:

  1. У солдат с более высоким уровнем образования возникало больше проблем с адаптацией, чем у менее образованных (интеллектуалы были менее готовы к стрессам войны, чем люди, выросшие на улицах).
  2. Южане легче, чем северяне, переносили жару островов Южного моря (южане более привычны к жаркому климату).
  3. Белые рядовые сильнее, чем чернокожие, стремились к продвижению по службе (годы угнетения посеяли в чернокожих желание «не высовываться»).
  4. Чернокожие южане предпочитали белых офицеров с Юга офицерам с Севера (так как первые обладали большим опытом общения с чернокожими).
  5. Когда война окончилась, солдаты скучали по дому не так сильно, как во время боевых действий (во время битвы солдаты знали, что находятся в смертельной опасности и могут больше не увидеть родных).

Сколько из этих наблюдений ты мог бы вывести заранее? 3 из 5? 4 из 5? Есть ли случаи, касательно которых ты предсказал бы противоположное; случаи, наносящие твоей модели мира удар? Прежде чем продолжить чтение, хорошо подумай над этим.

Все утверждения из этого списка (которые Мейерс взял из книги Пола Лазарсфельда2) прямо противоположны тому, что было обнаружено в действительности. Сколько раз твоя модель мира была испытана на прочность? Сколько раз ты признал, что ты бы ошибся? Теперь можно сделать вывод о том, насколько хороша твоя модель на самом деле: сила рационалиста состоит в способности удивляться вымыслу больше, чем реальности.

А ещё я мог перевернуть результаты ещё раз — тогда удары остаются ударами, а удачные предсказания удачными предсказаниями. Что скажешь?

Теперь ты действительно не знаешь ответа. Замечаешь ли ты, что процессы, идущие в твоей голове сейчас, чем-то отличаются от тех процессов, которые происходили там ранее? Чувствуешь ли ты, что поиск ответа ощущается по-другому, не так, как рационализация обеих сторон «известного» ответа?

Дафна Барац разделила студентов на две группы и сообщала одной результат социологического исследования (например, «Во время подъема экономики люди тра­тят большую часть своего дохода, чем во время спада» или «Люди, регулярно посещающие церковь, стремятся иметь больше детей, чем те, кто редко ходит в церковь»), а другой — перевёрнутый результат того же социологического исследования.3 Обе группы утверждали, что данный им результат они смогли бы предсказать заранее. Отличный пример эффекта знания задним числом.

Что приводит людей к мысли, что им не нужна наука, ведь всё «и так ясно».

(довольно очевидный вывод, не так ли?)

Знание задним числом заставляет нас систематически недооценивать неожиданность научных открытий, особенно тех открытий, которые мы можем понять; тех открытий, которые нам близки, и которые мы можем постфактум уместить в свою модель мира. Регулярно читающий новости человек, разбирающийся в неврологии или физике, скорее всего тоже недооценивает неожиданность открытий в этих дисциплинах. Этот эффект несправедливо обесценивает вклад исследователей, и, что ещё хуже, не даёт тебе заметить свидетельства, которые отличаются от того, что бы ты предсказал на самом деле.

Без сознательного усилия невозможно почувствовать должный уровень шока.

  • 1. David G. Meyers, «Exploring Social Psychology» (New York: McGraw-Hill, 1994), 15–19.
  • 2. Paul F. Lazarsfeld, «The American Solidier—An Expository Review», Public Opinion Quarterly 13, no. 3 (1949): 377–404.
  • 3. Daphna Baratz, «How Justified Is the “Obvious” Reaction?» (Stanford University, 1983).
Перевод: 
BT
Номер в книге "Рациональность: от ИИ до зомби": 
29
Оцените качество перевода: 
Средняя оценка: 4.7 (25 votes)

Загадочные ответы

«Таинственные ответы» ставит вопрос: способна ли наука решить эти проблемы для нас. Ученые основывают свои модели на повторяемых экспериментах, а не на спекуляциях и слухах. Наука, так же, имеет великолепную репутацию в сравнении с историями, религией и… Да в сравнении с чем угодно имеет. Стоит ли волноваться по поводу «ложных» убеждений, ошибки подтверждения, ошибки знания задним числом и им подобных, если мы работаем с сообществом людей, желающих объяснять феномены, а не рассказывать трогательные истории?

Автор: 
Элиезер Юдковский

Лжеобъяснения

Элиезер Юдковский

Давным-давно жила-была в одном городе учительница физики. В один прекрасный день она пригласила в класс своих студентов и показала им широкую квадратную металлическую пластину рядом с обогревателем. Студенты прикладывали ладони к пластине и ощущали, что сторона пластины рядом с обогревателем кажется холодной, а дальняя от обогревателя сторона кажется тёплой. «В чём дело, как вы думаете?» — спросила учительница. Некоторые заговорили о конвекции воздушных потоков, остальные предположили наличие странных примесей в пластине. Студенты предложили много изобретательных объяснений, никто не снизошёл до фраз «Я не знаю» или «По-моему, это просто невозможно».

А разгадка была в том, что учительница развернула пластину ненагретой стороной к обогревателю перед тем, как студенты вошли в комнату.1 2

Посмотрим на студента, растерянно бормочущего «Э… Ну, может быть, это из-за теплопроводности и всего такого?». Являются ли его слова полноценным убеждением? Слова достаточно легко произнести громким, убедительным голосом. Но контролируют ли они ожидание?

Подумаем о маленьком невинном предлоге «из-за», который стоит перед словом «теплопроводности». Подумаем о других вещах, которые он может предварять: например, можно сказать «из-за флогистона» или «из-за волшебства».

«Магия — не научное объяснение!» — можете закричать вы. Действительно, легко заметить, что эти две фразы — «из-за теплопроводности» и «из-за волшебства» — принадлежат различным литературным жанрам. Слово «теплопроводность» можно найти в лексиконе Спока из «Звёздного пути», а про «волшебство» может рассуждать Руперт Джайлз из «Баффи — истребительницы вампиров».

Но, будучи байесианцами, мы игнорируем жанры литературы. В наших глазах модель определяется через её воздействие на ожидания будущих событий. Ты сказал «теплопроводность»; на какие будущие переживания ты рассчитываешь, исходя из этой модели? В нормальных условиях эта модель подскажет тебе, что более тёплой на ощупь должна быть сторона пластины, которая ближе к обогревателю. Если фраза «из-за теплопроводности» может объяснить ещё и то, что ближняя к обогревателю сторона ощущается холоднее, то эта фраза может объяснить вообще всё что угодно.

Ну и, как все ужея надеюсьпоняли к этому моменту: если ты одинаково хорошо объясняешь любой исход, то знаний у тебя — ноль.

Если постоянно злоупотреблять фразой «из-за теплопроводности», то эта модель превратится в замаскированную гипотезу максимальной энтропии. В плане предсказаний такое предположение изоморфно фразе «это магия». Выглядит как объяснение, но им не является.

Представим, что мы измеряем температуру металлической пластины в различных точках и в разное время, вместо того, чтобы высказывать догадки вслух. Видя металлическую пластину рядом с нагревателем, обычно мы ожидаем увидеть, что температуры точек удовлетворяют равновесию диффузионного уравнения с учётом граничных условий, наложенных окружающей средой. У тебя может не получиться правильно определить температуру в первой точке измерения, но после измерения нескольких первых точек — я не настолько физик, чтобы знать, сколько именно точек потребуется — уже можно с отличной точностью вычислить температуру остальных.

Истинный гуру искусства Ограничения Ожиданий По Поводу Материальных Явлений С Помощью Чисел — один из тех людей, которых мы называем «физиками» — сделает измерения и скажет «Эта пластина находилась в равновесии с окружением две с половиной минуты назад, потом её повернули на 180 градусов, и сейчас она снова к нему приближается»

Ошибка студентов не просто в том, что они не сумели ограничить свои ожидания. Их менее явная, и более глубокая ошибка заключалась в том, что они думали, что занимаются физикой. Они сказали «потому что», дополненное чем-то похожим на изречения Спока в «Звёздном пути», и решили, что тем самым они приобщились к магистерию науки.

Это не так. Они просто переместили магию из одного жанра литературы в другой.

  • 1. История позаимствована из коллекции шуток Иохима Верхагена (Joachim Verhagen). Упомянутую историю можно найти поиском по словосочетанию «heat conduction».
  • 2. Замечу, что в оригинале история гораздо короче, чем в изложении Юдковского. — Прим.перев.
Перевод: 
BT
Номер в книге "Рациональность: от ИИ до зомби": 
30
Оцените качество перевода: 
Средняя оценка: 4.1 (16 votes)

Угадай слово, задуманное учителем

Элиезер Юдковский

В юности я читал популярные книги по физике, например «КЭД — странная теория света и вещества» Ричарда Фейнмана. Я знал: свет — это волны, звук — это волны, материя — это волны. Мне было девять лет и я гордился своей научной грамотностью.

Намного позже, когда я начал читать фейнмановские лекции по физике, я наткнулся на жемчужину под названием «волновое уравнение». Я мог проследить за его выводом, но у меня не выходило охватить это доказательство одним взглядом(English). В течении трёх дней, от случая к случаю, я думал об этом уравнении, и, наконец, понял, что оно до смешного очевидно. И после этого я осознал, что всё то время, когда я верил в честные заверения физиков о том, что свет — это волны, звук — это волны, материя — это волны, я не имел ни малейшего понятия о том, какой именно смысл вкладывают физики в слово «волна».

Вполне естественно думать, что если учёный говорит: «Свет — это волны», и учитель спрашивает, что такое свет, на что студент отвечает: «это волны», то студент произнёс истинное утверждение. По-другому ведь нечестно, правда? Если мы считаем фразу «свет — это волны» верной в устах физика, то она же должна быть верна и в устах студента? В самом деле, утверждение «свет — это волны» либо истинно, либо ложно, не так ли?

И это — ещё одна плохая привычка, которой нас учат в школе. У слов нет встроенных значений. Когда я слышу слоги «бо-бёр», в моём мозгу возникает образ большого грызуна; но это факт о состоянии моего разума, а не о слогах «бо-бёр». Последовательность слогов «это волны» (или «из-за теплопроводности») — это не гипотеза. Это набор колебаний воздуха, либо форма, принятая чернилами на бумаге. Внутри разума может быть связь между этой фразой и какой-нибудь гипотезой, но эта фраза, сама по себе, не является ни истинной, ни ложной.

Однако, если сказать школьному учителю «это волны», то ты получишь пятёрку с плюсом: учитель считает ответ «это волны» правильным, поскольку он наблюдал, как физик создаёт эти же колебания воздуха. А раз пятёрки с плюсом раздают за определённые фразы (написанные либо произнесённые), то студенты начинают думать, что у фраз есть истинностное значение. В конце концов, свет либо волны, либо не волны, так?

И это ведёт к ещё более ужасной привычке. Представим, что учитель ставит перед тобой странную задачу: ближняя сторона металлической пластины, лежащей рядом с обогревателем, ощущается менее тёплой, чем дальняя. Учитель спрашивает, в чём дело. Ответить «я не знаю» нельзя: тогда ты не только не получишь пятёрку с плюсом, но даже не будешь считаться участвовавшим в уроке. Но в течение этого семестра учитель использовал фразы «из-за теплопроводности», «из-за конвекции» и «из-за теплового излучения». Видимо, одну из них учитель и желает услышать в ответ. Поэтому ты тянешь: «Нууу… может быть, из-за теплопроводности?».

Это не гипотеза о металлической пластине. Это даже не полноценное убеждение. Это попытка подобрать пароль.

Даже вспомнить уравнение диффузии (математическое описание процесса теплопроводности) — не то же самое, что и сформировать гипотезу о металлической пластине. Это не школа, и никто не собирается проверять, способен ли ты написать уравнение диффузии по памяти. Это байесоткачество, и мы начисляем очки за ожидания будущих переживаний. Если ты используешь уравнение диффузии — измерив температуру нескольких точек термометром, а затем пытаясь предсказать результат следующего измерения — то тогда это определённо привязано к переживаниям реального мира. Даже если студент просто представляет себе движение тепла, и поэтому подносит спичку к холодной стороне для того, чтобы измерить, куда течёт тепло, то этот мысленный образ движения привязан к переживаниям и контролирует ожидание будущего.

Если ты не используешь уравнение диффузии: не подставляешь в него числа и не получаешь данные, влияющие на твои ожидания определённых переживаний, то тогда твоя когнитивная карта полностью отрезана от местности. То, что осталось, даже нельзя назвать убеждением — это просто речевое поведение.

Школьная система построена вокруг речевого поведения, выражается ли оно через колебания воздуха, или через узор чернил на бумаге. От речевого поведения зависит, получишь ли ты пятёрку с плюсом, или двойку вместе с вызовом родителей. Осознавать различие между объяснением и паролем — первый шаг на пути избавления от этой вредной привычки.

Не слишком ли это жестоко? Ведь, когда человек пытается разрешить загадку металлической пластины, мысль «теплопроводность?» может быть первым шагом к нахождению ответа, верно? Может быть, но только в том случае, если этот человек старается разрешить загадку, а не подобрать пароль. Если нет учителя, готового указать на ошибку, то ловушка становится ещё страшнее. Тогда можно считать фразу «Свет — это вакаликс» хорошим объяснением, можно думать, что слово «вакаликс» — правильный пароль. Когда мне было 9 лет, это случилось и со мной: не потому что я был глуп, а потому, что это то, что случается обычно, по умолчанию. Это привычный для людей образ мыслей, и чтобы его избежать, нужно приучить себя не попадать в эту ловушку. Человечество падало в такие ямы и сидело в них тысячелетиями.

Возможно, если вдолбить студентам, что слова не считаются, а имеют значение лишь контроллеры ожиданий, то никто больше не застрянет в западне алгоритма «Теплопроводность? Нет? Тогда конвекция? Тоже не то?». Возможно, тогда мысль «может быть, теплопроводность?» будет началом действительно полезного пути, например:

«Теплопроводность?»

Но это только фраза — что она означает?

Уравнение диффузии?

Но это только символы — как мне их применить?

Будь уравнение диффузии справедливым, чего бы я рассчитывал почувствовать?

Я определенно не рассчитывал бы обнаружить, что отдаленная от обогревателя часть металлической пластины будет теплей.

Я замечаю своё замешательство. Возможно ближняя сторона всего лишь ощущается более холодной? Скажем, она изготовлена из какого-нибудь плохо проводящего тепло материала и поэтому передает меньше теплоты моей руке? Я попробую измерить температуру…

Ладно, не сработало. А если проверить, приложимо ли вообще уравнение диффузии к этой металлической пластине? Тепло течет как обычно, или происходит нечто иное?

Можно поднести спичку к пластине и попробовать измерить, как тепло распространяется с течением времени…

Если не выкорчевать сорняк лжеобъяснения «Эмм, может быть из-за теплопроводности?», то студент, скорее всего, застрянет на стадии паролей и вакаликса. Это происходит по умолчанию, это происходило со всем человечеством на протяжении тысяч лет.

Перевод: 
BT
Номер в книге "Рациональность: от ИИ до зомби": 
31
Оцените качество перевода: 
Средняя оценка: 4.5 (37 votes)

Наука как одеяние

Элиезер Юдковский

В трейлере фильма «Люди Икс» голос за кадром говорит: «В каждом человеке… есть генетический код… вызывающий мутации». Несложно заметить, что, мутировав, можно приобрести разнообразные полезные способности. К примеру, мутант Шторм без труда метает молнии.

Прошу тебя, дорогой читатель, подумать о биологических приспособлениях, необходимых для производства электричества; о природной защите от собственного электричества; а также о структурах внутри мозга, отвечающих за тонкое управление разрядом молнии. Если мы бы и в самом деле обнаружили бы организм, который приобрёл такие способности за одно поколение, в результате мутации, то это бы одним махом вдребезги бы опровергло неодарвинистскую модель естественного отбора: это было бы ещё убийственнее, чем скелет кролика, датируемый докембрием. Если теорию эволюции действительно можно растянуть так, чтобы она не возражала против истории Шторм, то она станет способной объяснить всё, что угодно, и нам прекрасно известно, к чему такое ведёт.

Серия комиксов «Люди X» использует термины вроде «эволюция», «мутация» и «генетический код» лишь для создания атмосферы наукообразия; лишь для того, чтобы она относилась к науке как литературному жанру. И меня пугает то, как часто я встречаю людей (особенно в СМИ), воспринимающих науку исключительно как жанр литературы.

Я сталкиваюсь с людьми, которые твёрдо верят в эволюцию и не воспринимают всерьёз креационистов. При этом они вообще не имеют представления о том, что запрещено законами эволюционной биологии, а что нет. Они могут говорить про «следующий шаг эволюции человечества», как если бы естественный отбор имел бы свой план действий. Хуже того, они могут говорить о явлениях, вообще находящихся за пределами эволюционной биологии, вроде улучшений дизайна компьютерных чипов, дробления корпораций, или загрузке человека в компьютер, называя всё это «эволюцией». Если это относится к эволюционной биологии, то эволюционная биология относится ко всем явлениям на Земле.

Вероятно, большинство людей, которые верят в эволюцию, используют фразу «вследствие эволюции», потому что они хотят чувствовать себя причастными к научной тусовке. Выражение становится деталью имиджа, символическим одеянием, навроде лабораторного халата. Если бы научная тусовка вместо «вследствие эволюции» говорила бы «вследствие разумного замысла», то такие люди вдохновенно повторяли бы и это — это совершенно бы не затронуло их ожидания того, что можно встретить в мире. Для них нет разницы, говорить ли «из-за эволюции» или «из-за разумного замысла». По их мнению, эволюция никоим образом не запрещает существования Шторм, а научные словечки имеют единственное назначение — возможность отождествлять себя с племенем.

Я регулярно встречаю людей, которые с распростёртыми объятиями ждут создания «более-глупого-чем-человек» искусственного интеллекта, или даже «чуточку-более-умного-чем-человек» ИИ. Стоит начать рассказывать им о разработке ИИ, намного превосходящего человеческие возможности, как они сразу относят это к «псевдонауке» (English). При этом, разумеется, ни у кого из них нет теории интеллекта (пусть и сомнительной), позволявшей бы рассчитывать верхний и нижний пределы мощи процессов оптимизации. Скорее, они просто ассоциируют сверхчеловеческий ИИ с литературным жанром беллетристики о конце света; а, услышав историю о небольшой компании под управлением ИИ, они вспоминают забавные заметки из «Компьюленты». Их утверждения не опираются на какую-либо модель разума. Они не понимают, что им нужна модель, чтобы делать такие утверждения. Они даже не понимают, что наука основана на моделях. Их уничижительная критика целиком построена на сравнениях с апокалиптическими сюжетами, а не на, скажем, существующих принципах, которые делают невозможными подобное развитие событий. Наука для них — лишь литературный жанр, или группа «своих», к которой стоит быть причастным. Одеяние, которое они носят, не похоже на лабораторный халат; а учёные не похожи на футбольную команду, за которую они болеют.

Есть ли что-то такое в науке, верой во что вы гордитесь, но до сих пор не применяете вашу веру на практике? Вам лучше спросить себя сейчас, какие возможные варианты будущего ваша вера запрещает. Эта проверка покажет, что вы усвоили на самом деле, что вы сделали частью своей личности. Всё остальное — скорее всего, лишь пароли или одеяния.

Перевод: 
santacloud, MacDelph, BT, kostyazen, 7yukari7, Dmitry Antonyuk
Номер в книге "Рациональность: от ИИ до зомби": 
32
Оцените качество перевода: 
Средняя оценка: 4.1 (28 votes)

Лжепричинность

Элиезер Юдковский

Флогистон — это ответ Европы XVIII века на первоэлемент огня, введённый греческими алхимиками. Зажги древесину и позволь ей сгореть. Что представляет из себя эта яркая оранжевая штука? Почему древесина превратилась в пепел? На оба эти вопроса химики XVIII века отвечали — «флогистон».

…и больше ничего. Это всё, в этом и заключался их ответ: «флогистон».

Флогистон покидал горящие вещества как видимое пламя. В результате горящие вещества теряли свой флогистон и становились пеплом, своим «истинным материалом». Огонь, помещённый в герметичный сосуд, быстро гас потому, что воздух насыщался флогистоном и больше не мог его вместить. Уголь почти не оставлял никакого пепла, потому что он почти полностью состоял из флогистона.

Разумеется, никто не использовал теорию флогистона для того, чтобы предсказать результат химического превращения. Алхимик сначала смотрел на результат, а затем при помощи флогистона объяснял его. Не было и намёка на то, чтобы флогистонщики предсказали прекращение горения в замкнутом сосуде; они, скорее, зажгли огонь в сосуде, увидели его угасание и затем сказали: «Должно быть, воздух насытился флогистоном». Теорию флогистона нельзя применить для того, чтобы выяснить, чего ты точно не сможешь увидеть. Она может объяснить всё.

Наука ещё только начинала выходить на сцену. Очень долго никто не осознавал, что в этой теории что-то не так.

Встретив лжеобъяснение, очень легко не ощутить его фальшивость: потому они и опасны.

Современные специалисты предполагают, что люди думают о причино-следственных связях, используя нечто вроде направленных ациклических графов или байесовских сетей. Поскольку шел дождь, тротуар мокрый; поскольку тротуар мокрый, он скользкий:

[Дождь] -> [Тротуар мокрый] -> [Тротуар скользкий]

Из этого можно вывести (а, имея байесовскую сеть, можно даже точно вычислить эту вероятность), что, если тротуар скользкий, то, вероятно, шёл дождь. Однако, если уже известно о мокрости тротуара, то сообщение о его скользкости не несёт в себе никакой новой информации о дожде.

Почему огонь горячий и яркий?

[«Флогистон»] -> [Огонь горячий и яркий]

Это выглядит как объяснение. И в мозгу эта информация хранится в том же формате и под тем же расширением, что и «настоящие» объяснения. Но человеческий разум неспособен автоматически определить, что стрелка, соединяющая гипотезу с её возможными следствиями, никак не ограничивает пути, которыми могут проявляться эти следствия. Эффект знания задним числом делает ситуацию ещё хуже: люди могут считать, что гипотеза действительно ограничивает происходящее, хотя на самом деле гипотеза подогнана под происходящее постфактум.

Современная трактовка вероятностных рассуждений о причинности может точно описать, в чём именно состояла ошибка флогистонщиков. Байесовские сети были разработаны для того, чтобы, кроме всего прочего, не учитывать свидетельства дважды в том случае, когда логический вывод между причиной и следствием возможен в обе стороны. Например, я добыл кусочек ненадёжной информации о том, что тротуар мокрый. Это заставляет меня подумать: «возможно, идёт дождь». Но если идёт дождь, то утверждение «тротуар мокрый» стало более правдоподобным, так? То же самое ведь касается и скользкости тротуара, верно? Но если тротуар скользкий, то он, скорее всего, мокрый — и тогда нужно опять повысить вероятность того, что идёт дождь.

Джуда Перл приводит в качестве метафоры алгоритм подсчёта солдат в колонне. Представьте, что вы стоите в колонне и видите рядом только двух солдат: одного спереди и одного сзади. Всего трое солдат. Вы спрашиваете своего соседа: «А сколько солдат видишь ты?» Он вертит головой и говорит: «Троих». Получается, всего солдат шесть. Очевидно, что так решать эту задачу совершенно не стоит.

Умнее будет спросить у стоящего впереди солдата: «Сколько солдат перед тобой?», и у стоящего позади: «Сколько солдат за тобой?». Сообщение с вопросом «сколько солдат перед тобой?» можно передать дальше без особых затруднений. Если я стою первым, то я передам назад «1 солдат впереди». Человек, стоящий прямо за мной, получит сообщение «1 солдат впереди» и скажет второму своему соседу «2 солдата впереди». В это же время кто-то получает сообщение «N солдат позади» и передаёт стоящему впереди солдату сообщение «N+1 солдат позади». Сколько же всего солдат? Сложите оба полученных числа и добавьте единицу для себя — это и есть общее число солдат в линии.

Ключевая идея состоит в том, что каждый солдат должен отдельно отслеживать эти два сообщения, прямое и обратное, и сложить их вместе только в конце. Нельзя добавлять солдат из обратного сообщения, которое ты получил, в прямое сообщение, которое ты передашь дальше. Разумеется, сообщение с общим числом солдат никогда не появляется в этой цепочке: никто не произносит этого числа вслух.

Аналогичный принцип применяется в строгих вероятностных рассуждениях о причинности. Получение из не связанного с мокрым тротуаром источника каких-либо свидетельств о дожде создаст прямое сообщение от узла [дождь] к узлу [мокрый тротуар], и тем самым усилит ожидание увидеть мокрый тротуар. Наблюдение мокрого тротуара создаст обратное сообщение, идущее к убеждению о дожде, а затем это сообщение распространится от узла [дождь] до всех его соседей, кроме узла [мокрый тротуар]. Каждый кусочек свидетельства учитывается ровно единожды; корректировки никогда не застревают между узлами, скача туда и обратно. Точный алгоритм можно найти в классической книге «Probabilistic Reasoning in Intelligent Systems: Networks of Plausible Inference» Джуды Перла.

Так что же было неправильно в теории флогистона? Когда мы наблюдаем, что огонь горячий, узел [огонь] посылает обратное сообщение со свидетельством узлу [флогистон], вынуждая нас обновить убеждения о флогистоне. Но тогда мы не можем считать это успешным предсказанием теории флогистона. Сообщение должно идти в единственном направлении, не отражаясь назад.

Увы, для обновления сетей убеждений люди используют не строгий алгоритм, а его грубое приближение. Мы изучаем родительские узлы, наблюдая за дочерними узлами, и предсказываем поведение дочерних узлов, используя убеждения о родительских узлах. Но ящик с документацией по прямым сообщениям не отделён от ящика с документацией по обратным сообщениям толстой непроницаемой стеной. Мы просто помним: «флогистон горячий, и из-за этого огонь тоже горячий». Всё это выглядит так, будто теория флогистона предсказывает «горячесть» огня. Или, что ещё хуже, нам кажется: «флогистон делает огонь горячим».

Лишь после того, как кто-нибудь заметит полное отсутствие предсказаний заранее, не ограничивающий ожиданий причинно-следственный узел получит ярлык «фальшивка». До этого момента он не будет отличаться от остальных узлов в сети убеждений. Утверждение «флогистон делает огонь горячим» ощущается фактом точно так же, как и все остальные известные тебе факты.

Правильно спроектированный ИИ заметит проблему мгновенно. Для этого не понадобится какой-нибудь особенной заплатки, нужен всего лишь правильный учёт происходящего в сети убеждений (к сожалению, в отличие от правильно спроектированных ИИ, люди не способны переписывать свой исходный код, чтобы исправить найденные ошибки)

Рассуждения об «эффекте знания задним числом» — это просто способ не привлекая технических терминов рассказать о том, что люди не разделяют прямые и обратные сообщения, из-за чего прямые сообщения могут загрязняться обратными.

Люди, пошедшие по пути флогистона, не намеревались стать дураками. Ни один учёный не желает застрять в тупике. Не скрываются ли лжеобъяснения в недрах твоего разума? Если они там есть, то к ним определённо не приклеен ярлык «лжеобъяснение», и поэтому поиска по ключевому слову «фальшивка» явно недостаточно для того, чтобы их обнаружить.

Проверить, насколько хорошо теория «предсказывает» уже известные тебе факты, также недостаточно: эффект знания задним числом обесценит все усилия. Предсказывать нужно на завтра, а не на вчера. Лишь так можно быть уверенным в том, что захламлённый человеческий разум действительно посылает чистое прямое сообщение.

Перевод: 
BT
Номер в книге "Рациональность: от ИИ до зомби": 
33
Оцените качество перевода: 
Средняя оценка: 4.1 (29 votes)

Семантические стоп-сигналы

Элиезер Юдковский

И ребёнок спросил:

— Откуда взялся этот булыжник?

— Я отломил его от большого камня в центре деревни.

— Откуда взялся этот камень?

— Наверное, он скатился с большой горы, что возвышается над нашей деревней.

— Откуда взялась эта гора?

— Оттуда же, откуда и все камни. Это кости Имира, изначального великана.

— Откуда появился изначальный великан Имир?

— Из мировой бездны по имени Гинунгагап.

— Откуда появилась мировая бездна Гинунгагап?

— Никогда этого не спрашивай.

Рассмотрим кажущийся парадокс первопричины. Наука отследила цепочку событий до Большого Взрыва, но отчего случился сам Большой Взрыв? Можно и нужно сказать, что Большой Взрыв произошёл в «ноль часов ноль минут», и поэтому нельзя говорить о времени «до Большого Взрыва», поскольку к этому понятию неприменима обычная концепция времени. Но, говоря так, мы используем существующие физические законы, которые звучат довольно структурировано, что тоже требует объяснений. Откуда появились физические законы? Можно ответить, что вся Вселенная является компьютерной симуляцией, но тогда это симуляция должна быть запущена в каком-то другом мире, подчиняющемся другим законам физики — а откуда взялись они?

После того, как вопросы доходят до этой стадии, некоторые люди отвечают «Бог!».

Почему кто-либо, даже очень религиозный человек, может думать, что это может хоть как-нибудь помочь ответить на вопрос о первопричине? Почему вопрос «откуда взялся Бог?» автоматически не всплывает в разуме? Утверждения «Бог не может иметь причины» или «Бог создал Себя Сам» приводят нас в то же состояние, что и «время началось вместе с Большим Взрывом». Далее следует спросить, почему существует вся эта метасистема, или почему какие-то явления могут иметь причину, а какие-то не могут.

Я ставлю цель не обсудить мнимый парадокс первопричины, а задаться вопросом, почему кто-то считает, что восклицание «Бог!» может разрешить парадокс. Восклицание «Бог!» говорит о принадлежности племени, и поэтому у людей возникает соблазн делать это как можно чаще — иногда это утверждение можно услышать даже в ответ на «почему ураган обрушился на Новый Орлеан?». Но всё же… Совершенно очевидно, что в этой конкретной головоломке «Бог» ничем не помогает. Бог не смог бы сделать парадокс менее парадоксальным, даже если бы существовал. Как можно этого не замечать?

Джонатан Уоллес предположил, что «Бог!» работает, как семантический стоп-сигнал: это не столько сознательное утверждение, сколько дорожный знак на трассе для мыслей, говорящий «дальше не думай, проезд закрыт». Восклицание «Бог!» не разрешает парадокс, а, скорее, устанавливает в нужном месте дорожный знак, чтобы остановить цепочку естественных вопросов и ответов.

Но ты — хороший и правильный атеист, и, разумеется, ни за что не попадёшься в ловушку. Но семантические стоп-сигналы не исчерпываются восклицанием «Бог!», это лишь наглядный пример.

Трансгуманистические технологии — молекулярная нанотехнология, продвинутые биотехнологии, генетическая инженерия, искусственный интеллект, и так далее — ставят нас перед лицом нелёгких политических вопросов. В какой степени правительство должно вмешиваться в выбор генов будущего ребёнка его родителями, или оно не должно вмешиваться вообще? Если родители желают дать ребёнку ген шизофрении, то следует ли им это позволить? Если улучшение интеллекта — крайне дорогостоящая процедура, то должно ли государство её обеспечивать, чтобы не допустить возникновения когнитивной элиты? Эти задачи могут выполнять различные общественные институты — например, частные благотворительные фонды, оказывающие финансовую помощь в усилении интеллекта — но в ответ на каждое такое предложение неизбежно возникает очевидный дальнейший вопрос: «Справится ли этот институт со своей задачей?». Изготовление опасных нанотехнологий может караться судебными исками, но сработает ли такая схема?

Один из моих знакомых знает ответ на любой из этих вопросов: «Либеральная демократия!». Это всё. В этом и заключается его ответ. Если же попытаться спросить: «А насколько хорошо в мировой истории либеральные демократии справлялись с такими сложными задачами?» или «А если либеральная демократия совершит глупость?», тогда вы станете автократом, либертариа́нцем, или просто очень, очень нехорошим человеком. Никто не имеет права сомневаться в демократии.

Как-то я назвал такие размышления «божественной привилегией демократии», но точнее будет сказать, что «Демократия!» была его семантическим стоп-сигналом. Если бы кто-нибудь заявил: «Пусть всё это решает «Газпром»!», то он бы начал задавать очевидные вопросы: «Почему? Что «Газпром» может тут сделать? Почему ему можно доверять в таких вопросах? Что насчёт его прошлого опыта в решении похожих по сложности задач?»

Или, представим, что кто-нибудь заявляет: «поляки строят заговор, чтобы убрать кислород из атмосферы Земли». Ты наверняка задашься вопросом зачем им это надо, чем они будут дышать и способны ли они вообще тайно преследовать единые цели. Если ты не задаёшь дальнейших вопросов после утверждения «Корпорации планируют убрать кислород из атмосферы Земли», то слово «корпорации» сработало тут как семантический стоп-сигнал.

Не забывай, что понятие «семантический стоп-сигнал» нельзя превращать в универсальный контраргумент против вещей, которые тебе не по душе («Да ну, это просто бессмыслица, приправленная семантическими стоп-сигналами!»). Слово не может быть стоп-сигналом само по себе; вопрос заключается в том, производит ли оно этот эффект на конкретного человека. Сильные эмоции по отношению к чему-то — недостаточное основание для того, чтобы назвать это стоп-сигналом. Я не одобряю террористов и не испытываю страха перед частной собственностью, но это не означает, что слова «террористы» или «капитализм» выполняют функции дорожных знаков в моём мышлении (когда-то такой эффект имело слово «интеллект», но это уже в прошлом). Семантический стоп-сигнал отличает неспособность представить следующий очевидный вопрос.

Перевод: 
BT
Номер в книге "Рациональность: от ИИ до зомби": 
34
Оцените качество перевода: 
Средняя оценка: 4.2 (33 votes)

Таинственные ответы на таинственные вопросы

Элиезер Юдковский

Представь, что ты глядишь на свою руку, ничего не зная ни о клетках, ни о биохимии, ни о ДНК. У тебя есть некоторые познания в анатомии, полученные путём препарирования, поэтому тебе ясно, что в ладони есть мышцы. Однако, ты не знаешь, почему они движутся, вместо того, чтобы неподвижно лежать, как кусок глины. Твоя рука — просто кусок… эмм… вещества, и почему-то этот предмет исполняет твои мысленные приказы. Разве это не волшебство?

«Животное тело не ведёт себя, как термодинамическая система… сознание говорит каждому человеку, что он является, в какой-то степени, предметом своей воли. Это проявляется в том, что живые существа могут мгновенно прикладывать к определённым движущимся частицам материи внутри своих тел силы, направляющие движение этих частиц, для того, чтобы создавать наблюдаемые механические эффекты… Вопрос о влиянии животной или растительной жизни на материю беспредельно далёк от любых научных изысканий, начатых до настоящего времени. Сила управлять движением материи, ежедневно проявляемая в чуде свободы воли человека, и в поколениях растений, выросших из единого зерна, безгранично непохожа на любой возможный результат движения атомов, каким бы удачным он ни оказался… Современным биологам придётся запомнить ещё один принцип, и на этот раз — жизненно важный». (Лорд Кельвин)

В этом состоит теория витализма: загадочные различия между живой и неживой материей могут быть объяснены посредством «жизненной силы» («elan vital» или «vis vitalis»). «Жизненная сила» внедряется в живую материю и подчиняет её приказам сознания. Жизненная сила участвует в химических реакциях, из-за чего неживая материя не может проявлять часть свойств живой материи. В частности, без помощи жизненной силы невозможно получить живую материю из неживой; поэтому проведённый Фридрихом Вёлером химический синтез мочевины нанёс сильный удар по теории витализма, показав, что обыкновенной химии по силам получить биологический продукт.

Называть «жизненную силу» объяснением или даже лжеобъяснением вроде флогистона — значит, переоценивать эту теорию. «Жизненная сила», в первую очередь, работает, как затычка для любопытства. Ты спрашиваешь «почему?», слышишь ответ «жизненная сила!», и на этом разговор окончен.

Когда ты говоришь «жизненная сила!», тебе кажется, будто ты знаешь, почему двигается твоя рука. В твоей голове есть маленькая причинно-следственная диаграмма, которая говорит: [«жизненная сила!»] -> [рука двигается].

Однако на самом деле ты не знаешь ничего, что не знал раньше. Например, ты не сможешь сказать, будут ли твои руки отдавать или поглощать тепло, пока не пронаблюдаешь это в действительности. Ты не сможешь предсказать этого заранее. Твоё любопытство удовлетворено, но оно удовлетворено пустышкой. Раз любому наблюдению ты можешь сказать «Почему? Жизненная сила!», то витализм одинаково хорошо объясняет все исходы, не способен противоречить вообще хоть каким-нибудь фактам, является замаскированной гипотезой максимальной энтропии, и так далее.

Но главный урок нужно извлечь из благоговения виталистов пред жизненной силой, из их старания провозгласить её тайной, стоящей выше всей науки. Встретив великого дракона по имени Неизвестность, виталисты не обнажили клинков, чтобы попытаться пронзить его сердце, но мирно склонили головы в знак подчинения. Они превратили биологию в священную тайну и гордились своим невежеством, потому и не желая отказаться от незнания, когда на сцене появились свидетельства.

Великий Секрет Живого был бесконечно далёк от науки! Не просто слегка вдали, заметьте, но бесконечно далёк! Лорд Кельвин явно получал колоссальное наслаждение от незнания.

Но невежество — это то, что рисуется на карте, а не то, что можно обнаружить, гуляя по местности. Если я не имею ни малейшего представления о неком явлении, то это факт о состоянии моего разума, а не о самом явлении. Явление может быть таинственным в глазах некого определённого человека. Не существует явлений, таинственных самих по себе. Поклоняться явлению, потому что оно выглядит столь потрясающе таинственно, — означает поклоняться собственному невежеству.

Витализм, как и флогистон, заключил загадку в отдельную субстанцию. Огонь был загадкой, и теория флогистона заключила загадку в таинственную субстанцию под названием «флогистон». Жизнь была священной тайной, и витализм заключил священную тайну в таинственную субстанцию под названием «жизненная сила». Ни один из ответов не попытался сконцентрировать плотность вероятности модели, сделать какие-то результаты более ожидаемыми, чем другие. Эти «объяснения» просто закутали вопрос в твёрдый непрозрачный чёрный шарик.

В одной из комедий Мольера доктор объясняет действие снотворного тем, что в нём содержится «фактор усыпления». Тот же самый принцип. Это универсальный недочёт человеческой психики: столкнувшись с таинственным явлением, нам легче объяснить его через таинственную субстанцию с внутренне присущими ей свойствами, чем через лежащие в основе сложные процессы.

Но ещё более страшная ошибка — допущение того, что ответ может быть таинственным. Если явление кажется таинственным, то это факт о наших знаниях, а не факт о самом явлении. Виталисты увидели таинственный пробел в своих знаниях и постулировали таинственную штуку, заполняющую этот пробел. Тем самым они перемешали карту и местность. Всё недоумение и замешательство находятся в карте, а не внутри отдельных субстанций.

Именно поэтому раз за разом на протяжении всей человеческой истории люди поражаются тому, что невероятно таинственный вопрос имеет приземлённый не-таинственный ответ. Окутанными тайной могут быть только вопросы, но не ответы.

Поэтому я называю теории вроде витализма «таинственными ответами на таинственные вопросы».

Признаки таинственных ответов на таинственные вопросы:

Во-первых, объяснение работает не контроллером ожиданий, а затычкой для любопытства.

Во-вторых, в гипотезе нет движущихся частей: модель является не определённым сложным механизмом, а, скорее, просто сплошной субстанцией или силой. В гипотезе говорится, что таинственная субстанция или таинственная сила находятся вот здесь и вызывают вот это, но причина, по которой таинственная сила ведёт себя именно таким образом, инкапсулирована в пустую тавтологию.

В-третьих, люди, предлагающие это объяснение, дорожат собственным незнанием. Они с гордостью говорят о том, что обычная наука терпит поражение от этого явления, и о том, как это явление непохоже на все остальные обыденные явления.

В-четвёртых, несмотря на этот ответ, явление по-прежнему остаётся тайной, сохранив ту же степень завораживающей необъяснимости, что и вначале.

Перевод: 
BT
Номер в книге "Рациональность: от ИИ до зомби": 
35
Оцените качество перевода: 
Средняя оценка: 4 (56 votes)

Тщетность эмерджентности

Элиезер Юдковский

Провалы флогистона и витализма — примеры исторического знания задним числом. Посмею ли я выступить и назвать какую-то современную теорию, которую считаю настолько же ошибочной?

Я называю эмерджентность или эмерджентный феномен — обычно определяемый как исследование систем, поведение которых на высоких уровнях являются следствием или «появляется» (to emerge) из взаимодействия многих низкоуровневых элементов. (Википедия: «То, как сложные системы и паттерны появляются из множества относительно простых взаимодействий»). Если воспринимать буквально, это утверждение подходит для любого феномена нашей вселенной, выше уровня отдельных кварков, что и является частью проблемы. Представьте, что можно показать на обвал рынка и сказать «Это не кварк!». Похоже на объяснение? Нет? Ну так и «Это эмерджентный феномен!» тоже не подходит.

Я протестую против прилагательного «эмерджентный», а не против глагола «появляется». Нет ничего плохого во том, чтобы сказать «X появляется на основе Y», где Y — специфическая, детальная модель с динамическими частями. «Возникает на» — еще один способ выразить ту же мысль: гравитация возникает из кривизны пространства-времени, в соответствии со специфической математической моделью Общей Теории Относительности. Химия возникает на основе взаимодействия атомов, в соответствии со специфической моделью квантовой электродинамики.

Представим, я буду говорить, что гравитация объясняется «возникновением», или что химия — «возникающий феномен», и утверждать, что это мое объяснение.

Слово «появляется» приемлемо, так же, как и «появляется из» или «вызвано» приемлемы, если ссылаются на специфическую модель, которую можно оценить саму по себе.

Однако, «эмерджентность» обычно используется иначе. Оно используется как объяснение само по себе.

Я уже потерял счет случаям, когда я слышал «Интеллект — эмерджентный феномен!», как объяснение интеллекта. Это применение подходит всем пунктам, характеризующим мистический ответ на мистический вопрос. Что ты узнал, сказав, что интеллект «эмерджентный»? Ты не можешь сделать новых предсказаний. Ты не знаешь ничего о поведении реальных разумов, о чем бы не знал ранее. Это воспринимается как новый факт, но ты не ожидаешь других результатов. Твое любопытство вроде бы удовлетворено, но не накормлено. Гипотеза не имеет динамических частей, нет детализированной внутренней модели для манипуляций. Те, кто предлагают гипотезу «эмерджентности», признаются в своем незнании внутреннего устройства и гордятся этим; они противопоставляют «эмерджентные» науки и «обычные».

И даже после того, как ответ «Как? Эмерджентность!» дан, феномен не перестаёт быть таинственным и обладать той же непроницаемостью, что и прежде.

Интересное упражнение: удалить прилагательное «эмерджентный» из предложения и посмотреть, изменилось ли что:

• До: человеческий интеллект — это эмерджентный результат нейронных вспышек.

• После: человеческий интеллект — это результат нейронных вспышек.

• До: поведение колонии муравьем — это эмерджентный результат взаимодействия многих отдельных муравьев.

• После: поведение колонии муравьев — это результат взаимодействия многих отдельных муравьев.

• Даже лучше: Колония состоит из муравьев. Мы можем успешно предсказывать поведение колонии, используя модели, включающие только индивидуальных муравьев, без общих переменных колонии, показывая, что мы понимаем, как поведение колонии возникает на основе поведения муравьев.

Еще одно интересное упражнение: заменять слово «эмерджентный» на старое, то, которым люди пользовались до эмерджентности:

• До: жизнь — эмерджентный феномен.

• После: жизнь — магический феномен.

• До: человеческий интеллект — это эмерджентный результат нейронных вспышек.

• После: человеческий интеллект — это магический результат нейронных вспышек.

Не правда ли, что каждое утверждение дает одинаковый объем информации о поведение феномена? Что каждая гипотеза подходит под одинаковый набор результатов?

«Эмерджентность» очень популярна, как раньше «магия» была популярна. «Эмерджентность» имеет глубокую притягательность для человеческой психологии по тем же причинам. «Эмерджентность» — такое замечательное простое объяснение, и это приятно произносить; это дает тебе священную тайну для поклонения. «Эмерджентность» популярна, потому что это «доширак» для любопытства. Можно объяснить что угодно, используя эмерджентность, так что люди это и делают; ведь это так замечательно — объяснять что-то. Люди остаются людьми, даже если посещали пару научных курсов в колледже. Найдя способ избежать оков обычной науки, они возвращаются к тем же проделкам, что и их предки, одеваясь в одежды науки, но сохраняя ту же самую видовую психологию.

Перевод: 
Muyyd
Номер в книге "Рациональность: от ИИ до зомби": 
36
Оцените качество перевода: 
Средняя оценка: 4.2 (23 votes)

Скажи нет «сложности»

Элиезер Юдковский

Однажды…

Эта история произошла во времена, когда я впервые встретил Марчелло, с которым, позже, я буду год работать над теорией ИИ, но на тот момент я еще не принял его в свои ученики. Я знал, что он участвовал в соревнованиях по математике и информатике на национальном уровне, и этого было достаточно, чтобы я захотел присмотреться к нему. Но я еще не знал, сможет ли он научиться думать об ИИ.

Я спросил Марчелло, как, по его мнению, ИИ может разработать способ решения кубика Рубика. Не в смысле написания программы, что довольно тривиально, а открытия законов вселенной Рубика и построения рассуждений об их использовании. Как ИИ изобретёт для себя концепты «оператора» или «макро», которые являются ключами для сборки кубика Рубика?

И, в процессе дискуссии, Марчелло сказал: «Ну, ИИ понадобится сложность для того, чтобы сделать Х, и для того, чтобы сделать У…»

И я сказал: «Не говори „сложность“».

Марчелло спросил: «Почему?»

Я ответил: «Сложность не должна быть самоцелью. Возможно, тебе понадобится определённый алгоритм, добавив который, ты увеличишь сложность, но сложность ради сложности лишь усложняет задачу». (Сказав это, я вспомнил о всех тех людях, что рассказывают про Интернет, который, став «достаточно сложным», «проснется» в качестве ИИ).

И Марчелло сказал: «Но должно же быть какое-то количество сложности, которое бы позволяло сделать это».

Я прикрыл глаза и попытался облечь мою мысль в слова. Для меня, говорить «сложность» - делать неправильный пируэт в танце ИИ. Никто не может думать достаточно быстро, осознанно, используя слова для выражения потока сознания, это потребует бесконечной рекурсии. Мы думаем словами, но поток нашего сознания протекает ниже уровня слов, посредством выученных остатков озарений прошлого и горького опыта…

Я спросил:

— Ты читал «Техническое объяснение технического объяснения»(English)?

— Да, — ответил Марчелло.

— Ладно! — сказал я. — Говоря «сложность», ты не помогаешь себе сконцентрировать массу вероятности.

— Ох, — сказал Марчелло, — это как с «эмерджентностью», хмх. Так… Теперь мне надо подумать, как Х может случиться по-настоящему.

Тогда я и подумал: «Возможно, это парень обучаем».

Сложность — далеко не бесполезный концепт. Она может быть определена математически, например в виде Колмогоровской сложности или размерности Вапника-Червоненкиса. Даже на интуитивном уровне, сложность стоит того, чтобы рассуждать о ней — тебе надо оценить сложность гипотезы и решить что она «слишком сложна» для имеющегося объема свидетельств или посмотреть на строение и попытаться упростить.

Но концепты сами по себе не становятся полезными или бесполезными. Использование может быть корректным или нет. Движение, которое пытался вплести в танец Марчелло, — попытка объяснения «за просто так», получить что-то в обмен на ничего. Это часто повторяемая ошибка, в моей сфере, как минимум. Можно вступить в дискуссию об Искусственном Интеллекте и наблюдать, как люди наступают на те же грабли тут и там, вновь и вновь, постоянно игнорируя собственное непонимание.

Ты и моргнуть не успеешь, а это уже произошло: проталкивание неконтролируемого причинного узла для чего-то таинственного — причинного узла, который воспринимается как объяснение, но не является таковым. Эта ошибка случается ниже уровня слов. Она не требует какого-то особенного изъяна личности: так устроено человеческое мышление по-умолчанию, так люди рассуждали с древних времен.

Чего тебе следует избегать, так это игнорирования таинственного: ты должен задержаться рядом с тайной и столкнуться с ней напрямую. Есть множество слов, способных «пронести» тайну, и некоторые из них можно вполне обоснованно применять в других контекстах, например, сложность. Но главная ошибка — игнорировать контрабанду таинственного, не замечая причинный узел, скрытый за ней. Контрабанда не является мыслью, а микромыслью. Нужно уделять пристальное внимание, чтобы заметить это. И, натренировав себя в избегании этого, можно превратить умение в инстинкт, ниже уровня слов. Нужно ощущать какие области карты пока пусты и, главное, уделять внимание этому чувству.

Подозреваю, что в научных кругах присутствует сильное давление, способствующее сокрытию таких проблем, ведь нужно выдавать бумаги с шлейфом законченности. Тебя будут больше почитать, если в твоей будто бы полной работе будет присутствовать «эмерджентный феномен», а не за незаконченную, в которой есть ярлыки «понятия не имею, как это работает» или «а тут происходит чудо». Журнал может даже не принять такую работу, ведь, кто знает, вдруг необъяснённые явления и являются моментами, где всё самое интересное происходит? И да, случается так, что все немагические части оказываются также и неважными. Это цена, которую приходится порой платить за вход в неизведанное и попытки решить проблемы мелкими шажками. Но это так же означает, что понимание того, что ты еще не закончил чрезвычайно важно. Часто люди даже не осмеливаются исследовать неизведанное, ужасно боясь потратить время впустую.

И, если ты работаешь над революционным ИИ-стартапом, то давление ещё выше, а желание спрятать проблемы ещё больше, иначе придётся признать, что ты не знаешь пока как создать ИИ, и твоя жизнь обратится в прах. Но, возможно, я слишком усложняю, ведь контрабанда происходит неосознанно. Далеко ходить за примерами не нужно: просто послушай, как люди обсуждают философию, религию или науки, в которых у них нет профессиональной подготовки.

Марчелло и я пришли к соглашению о работе с ИИ: если мы сталкиваемся с чем-то, чего мы не понимаем, а это случалось довольно часто, мы будем говорить об этом как о «магии». Например, «Х магически делает У», чтобы напоминать себе о том, что тут присутствует нерешённая проблема, провал в понимании. Гораздо полезней говорить «магия», чем «сложность», ведь последнее слово создает иллюзию понимания. Мудрей говорить «магия» и оставлять себе напоминание о работе, которую надо сделать позже.

Перевод: 
Muyyd
Номер в книге "Рациональность: от ИИ до зомби": 
37
Оцените качество перевода: 
Средняя оценка: 4.5 (18 votes)

Подтверждающее искажение: взгляд во тьму

Элиезер Юдковский

Однажды, проводя занятие у студентов, я выписал на доске три числа: 2-4-6. «Я загадал правило, — сказал я, — которому подчиняются последовательности трех чисел. Данная последовательность подходит под это правило. Каждый из вас найдет у себя на парте листы бумаги. Выпишите последовательность трех чисел на карточке, а я в зависимости от того, подходит ли последовательность под правило, напишу на листе «да» или «нет». Это можно повторять до тех пор, пока вы не будете уверены, что угадали правило. После этого вы должны записать правило внизу листа».

Вот последовательность предположений одного из студентов:

4, 6, 2 — нет,

4, 6, 8 — да,

10, 12, 14 — да.

После этого студент записал свою версию правила. Как вы думаете, что он написал? А вы бы тоже остановились здесь или хотели бы протестировать еще триплет чисел? Если да, то какой? Остановитесь здесь и немного подумайте, прежде чем продолжить чтение.

Вышеприведенное упражнение основано на классическом эксперименте Питера Уэйсона, задаче «2-4-6». Несмотря на то, что испытуемые, которым дают эту задачу, выражают большую уверенность в своих предположениях, только 21% из них успешно угадывают задуманное экспериментатором правило, и все повторные опыты продолжали показывать тот же уровень успехов в районе 20%.

Исследование называлось «On the failure to eliminate hypotheses in a conceptual task» (Еженедельник экспериментальной психологии 12: 129-140, 1960). Испытуемые, которым предлагалась задача «2-4-6», обычно пытались придумать положительные примеры, а не негативные — они применяли гипотетическое правило для создания триплета, а потом смотрели, будет ли он отмечен как «да».

Таким образом, кто-то, кто формирует гипотезу «числа, каждое из которых больше предыдущего на два», тестирует триплет 8-10-12, видит, что подходит и уверенно объявляет свое правило. Кто-то, кто формирует гипотезу Х-2Х-3Х, тестирует триплет 3-6-9, обнаруживает, что триплет подходит, и тоже объявит правило.

В каждом из этих случаев настоящее правило одно и то же: три любых числа в порядке возрастания.

Однако, чтобы додуматься до этого, вы должны придумывать триплеты, которые не должны быть правильными, такие как 20-23-26 и проверять, будут ли они отмечены как «нет». Что люди обычно не склонны делать в этом эксперименте. В некоторых случаях испытуемые изобретают, «тестируют» и объявляют правила куда более сложные, чем настоящий ответ.

Данное когнитивное явление часто валят в одну кучу с предвзятостью подтверждения. Однако, на мой взгляд, явление склонности тестирования положительных примеров, а не отрицательных, следует отделять от явления стремления защитить изначальное убеждение. «Положительное искажение» иногда используется как синоним для предвзятости подтверждения и может описывать данный недостаток куда лучше.

Раньше казалось, что теория флогистона может объяснить прекращение огня в закрытой коробке (воздух переполнился флогистоном и больше не может вместить), но теория флогистона точно так же могла бы объяснить и тот вариант, если бы огонь продолжал гореть. Чтобы заметить это, вы должны искать негативные примеры вместо положительных, смотреть на ноль, а не на единицу; что, как показал эксперимент, идет вразрез с человеческим инстинктом.

Следуя инстинкту, мы живем в половинчатом мире.

Можно днями читать про положительное искажение и всё еще не увидеть его в момент срабатывания. Положительное искажение работает не на уровне логики или даже эмоциональной привязанности. Задача 2-4-6 «холодная», логичная, не эмоционально «горячая». Ошибка находится ниже уровня слов, на уровне образов, инстинктивных реакций. Поскольку проблема появляется не из-за следования осознанному правилу, которое говорит «Думай только о положительных примерах», ее нельзя решить, сказав вслух «Мы должны думать как о положительных, так и о негативных примерах». Какие примеры автоматически всплывают в вашей голове? Вы должны уметь еще до вербального формулирования думать о негативном примере, а не о положительном. Вы должны научиться поворачиваться лицом к нулю, а не убегать от него.

Некоторое время назад я писал, что сила гипотезы определяется тем, что она не может объяснить, а не тем что может — если вы одинаково легко объясняете любой исход, то у вас ноль знаний. Так, чтобы указать на то, что объяснение не является полезным, недостаточно задуматься над тем, что оно может объяснить хорошо — вам нужно также искать результаты, которые нельзя объяснить, это и будет истинной силой теории.

Теперь, после всего сказанного напомню, что вчера я бросил вызов «сложности» как понятию. Один комментатор привел сверхпроводимость и ферромагнетизм как примеры сложности. Я ответил, что несверхпроводимость и неферромагнетизм тоже примеры сложности, в чём и состоит проблема. Но я не имел в виду критиковать комментатора! Несмотря на то, что я много читал про предвзятость подтверждения, я не воскликнул «Эврика!», когда в первый раз прочитал про задачу «2-4-6». Это невербальная реакция, работающая очень быстро, и которую надо тренировать заново. Сам я всё еще работаю над этим.

Так что большая часть навыка рационалиста находится за пределами уровня слов. Это делает трудной работу по попыткам передать Искусство через посты в блоге. Люди согласятся с вами, а уже в следующем предложении сделают нечто, ведущее совершенно в другом направлении. Не подумайте, что я жалуюсь! Основная причина, по которой я пишу здесь — наблюдать, какие из моих слов не передаются.

Прямо сейчас вы ищете положительные примеры положительного искажения или ищете, что вы могли не увидеть из-за положительного искажения? Вы смотрите во свет или во тьму?

Перевод: 
Remlin
Номер в книге "Рациональность: от ИИ до зомби": 
38
Оцените качество перевода: 
Средняя оценка: 3.9 (55 votes)

Закономерная неопределённость

Элиезер Юдковский

В «Рациональном выборе в неопределенном мире» Робина Доуза описан эксперимент, проведенный Тверским1 2:

Множество психологических экспериментов были проведены в конце 50-х начале 60-х, в которых испытуемых просили предсказать результат события, имевшего элемент случайности и, при этом, еще и предсказуемую базовую ставку. Например, испытуемых просили предсказать цвет следующей карты (синяя/красная) при условии, что 70% карты были синими, но последовательность красных и синих карт была совершенно случайной.

В такой ситуации, стратегия, обеспечивающая большее количество успеха - ставить на наиболее часто встречающиеся события. Например, если 70% карт были синими, то предсказывая появление синей карты дает 70% успеха в каждом случае.

Испытуемые же предпочитали сопоставлять вероятности - предсказывать наиболее вероятный вариант с учетом относительной частоты появления. Например, испытуемые предсказывали появление синей карты 70% раз и красной 30% раз. Такая стратегия давала 58% долю успеха, потому что испытуемые были правы 70% времени, когда выпадала синяя карта (что происходило с вероятностью .70) и 30% времени, когда выпадала красная карта (что происходило с вероятностью 0,3); (0,7 × 0,7) + (0,3 × 0,3) = 0,58.

Даже больше, испытуемые предсказывали наиболее часто повторяющееся событие с большей вероятностью, чем оно происходило, но и близко не подходили к частоте в 100%, даже если им платили за точность их предсказаний… Например, испытуемые, которым платили по пятаку за каждое сбывшееся предсказание из тысячи,.. предсказывали [наиболее часто встречающееся событие] 76%.

Не стоит думать, что этот эксперимент про небольшие изъяны в игорных стратегиях. Он кратко рассказывает о наиболее важной идее всей рациональности.

Испытуемые продолжали выбирать красный, как будто они полагали, что способны предугадать случайную последовательность. Доуз пишет про это: «Несмотря на получение фидбека от тысячи случаев, испытуемые не могли поверить, что в это ситуации они не в состоянии предугадать».

Но ошибка должна иметь более глубокие последствия. Даже если испытуемые сформулировали какую-то гипотезу, им совершенно необязательно делать ставки на ее основе. Они могут говорить: «Если гипотеза верна - следующая карта будет красной», и ставить на синюю. Они могут выбирать синюю каждый раз, собирая как можно больше пятаков, отмечая мысленно как можно больше паттернов, которые они замечают. Если их предсказания сбываются, они могут легко переключится на новую стратегию.

Я бы не стал упрекать испытуемых за постоянное изобретение новых гипотез - откуда им знать, что цепочка за границами их способностей предсказывать? Но я буду упрекать за ставки на догадки, когда в этом не было необходимости для сбора информации, и буквально сотни предыдущих догадок были опровергнуты.

Неужто люди настолько самонадеянны?

Я бы предположил, что дело обстоит проще - стратегия «всегда-на-синее» просто не приходила испытуемым в голову.

Люди видят кучу синих карт вперемешку с несколькими красными, и полагают, что выигрышная стратегия - больше на синий, но иногда на красный.

Идея оптимальной стратегии, с учетом неполной информации, не предполагающая ставку на типичную последовательность карт - контринтуитивна.

Идея оптимальной стратегии, предполагающая законное поведение, даже если среда содержит элементы случайности - контринтуитивна.

Кажется, что твое поведение, следом за окружающей средой, должно быть непредсказуемым, но нет! Случайный ключ не отпирает случайный замок просто потому что они «оба случайные».

Ты не гасишь огонь огнем, ты гасишь огонь водой. Но эта мысль подразумевает лишний шаг, новый концепт, не активируемый напрямую формулировкой задачи. Поэтому, не приходящий в голову первым.

В дилемме красных и синих карт наше неполное знание говорит нам ставить в каждом раунде на синюю. Совет, даваемый нам нашим неполным знанием, одинаков от раунда к раунду. Если 30% времени мы будем идти против нашего неполного знания и ставить на красную карту, мы будем проваливаться, ведь теперь, мы нарочно тупим, ставя на, как нам прекрасно известно, менее вероятный исход.

Если ты будешь ставить на красную карту в каждом раунде, то ты будешь проигрывать так, как это максимально возможно; ты будешь на 100% тупым. Если ты ставишь на красную карту 30% времени, то ты оказываешься на 30% тупым.

Если твое знание неполно, если реальность, как кажется, содержит элемент случайности - случайное поведение не решит проблему. Делая свое поведение случайным, напротив, ты уводишь себя от цели, а не приближаешь. Если реальность туманна, выбрасывание интеллекта лишь ухудшает ситуацию.

Это контринтуитивно - думать, что оптимальная стратегия предполагает законное поведение, даже в условиях неопределенности.

Поэтому не так уж много вокруг рационалистов; для большинства, восприятие хаотичного мира предполагает хаотичные стратегии борьбы с ним. Тебе надо сделать лишний шаг, подумать мысль, не приходящую в голову первой, для того, чтобы сообразить что-то иное для борьбы с огнем, чем огонь.

Ты слышал, как непросвещенные говорят: «Рациональность работает лишь при взаимодействии с рациональными людьми, но мир нерационален». Но выбрасывание собственной рациональности, при встрече с нерациональным оппонентом, не поможет. Есть законные формы мышления, которые все же генерируют наилучший ответ, даже при встрече с оппонентом, нарушающим законы. Теория принятия решений не сгорает синим пламенем при встрече с оппонентом, не подчиняющимся этой теории.

Это настолько же не очевидно, как и всегда ставить на синюю карту, при встрече с совокупностью синих и красных карт. Но каждая ставка на красную - ожидаемый проигрыш, как и каждое отступление от принципов Пути, когда рассуждаешь.

Как много эпизодов Звездного Пути опровергается? Как много теорий ИИ?

  • 1. Dawes, Rational Choice in An Uncertain World; Yaacov Schul and Ruth Mayo, “Searching for Certainty in an Uncertain World: The Difficulty of Giving Up the Experiential for the Rational Mode of Thinking,” Journal of Behavioral Decision Making 16, no. 2 (2003): 93–106, doi:10.1002/bdm.434.
  • 2. Amos Tversky and Ward Edwards, “Information versus Reward in Binary Choices,” Journal of Experimental Psychology 71, no. 5 (1966): 680–683, doi:10.1037/h0023123.
Перевод: 
Muyyd
Номер в книге "Рациональность: от ИИ до зомби": 
39
Оцените качество перевода: 
Средняя оценка: 4.2 (12 votes)

Моя дикая и безбашенная юность

Элиезер Юдковский

Говорят, что всё, что родители запрещают делать детям, они делали сами — так они знают, почему это делать не стоит.

Давным-давно, в непостижимо далеком прошлом, я был преданным Традиционным Рационалистом, думая о себе, как о квалифицированном, в соответствии с ее стандартами, тем не менее, я не знал тогда Пути Байеса. Когда юный Элиезер столкнулся с кажется-мистическим вопросом, принципы Традиционной Рациональности не остановили его от производства Таинственного Ответа. Это, на данный момент, моя самая постыдная ошибка, думать о которой мне до сих пор больно.

Что за таинственный ответ на таинственный вопрос? Этого я говорить не стану, так как это длинная и запутанная история. Я был молод, был всего лишь Традиционным Рационалистом, который не ведал учения Тверски и Канемана. Я знал про Бритву Оккама, но не про конъюнктивное заблуждение. Я полагал, что мне удастся думать сложные мысли самому, в том же стиле, который я наблюдал в научных книгах, но не осознавал, что единственно верная сложность — та, где каждый шаг описан с безжалостной точностью. Сегодня, одним из главных советов, который я даю начинающим рационалистам: «Не пытайтесь строить сложных цепочек рассуждений и планов».

Нет нужды говорить больше: даже после того, как я придумал мой «ответ», феномен не потерял свою таинственность и непрошибаемость, которые имел с самого начала.

Не стоит думать, что юный Элиезер был глуп. Все те ошибки, в совершении которых виновен Элиезер, совершаются солидными учеными и в солидных изданиях и сегодня. Ему потребовался более утонченный навык, чем то, что могла дать Традиционная Рациональность.

Действительно, юный Элиезер старательно и кропотливо следовал предписаниям Традиционной Рациональности, сбиваясь при этом с пути.

Как Традиционный Рационалист, юный Элиезер внимательно следил, чтобы его Таинственный Ответ делал прямое предсказание будущего опыта. Конкретно, я ожидал, что будущие неврологи откроют использование квантовой гравитации нейронами, а ля Сэр Роджер Пенроуз. Подразумевалось, что нейроны будут обладать некоторой мерой квантовой запутанности, а это можно обнаружить наблюдениями, или не обнаружить. Ты либо будешь наблюдать это, либо нет, так ведь?

Но моя гипотеза не делала ретроспективных предсказаний. Ведь согласно Традиционной Науке, они не считаются — так что зачем тратить силы на их производство? Но для Байесовского Агента, если гипотеза сегодня не имеет предпочтительных отношений правдоподобия над «я не знаю», возникает вопрос: почему ты веришь сегодня во что-то более сложное чем «я не знаю». Но я не ведал Пути Байеса, так что я не думал про отношения правдоподобия или фокусирование плотности вероятности. Я ведь Сделал Фальсифицируемое Предсказание; разве не таков Закон?

Как Традиционный Рационалист, юный Элиезер был внимателен и не верил в магию, мистицизм, углеродный шовинизм и все такое прочее. Я гордо провозглашал, что мой Мистический Ответ «Такая же физика, как и вся остальная физика!». Как будто можно спасти магию от когнитивной изоморфности магии, называя ее квантовой гравитацией. Но я не ведал Пути Байеса и не видел уровень, на котором моя идея становилась изоморфна магии. Я отдавал мою верность физике, но это не спасло меня; какое дело теории вероятностей до чьей-либо верности? Я избегал всего, что запрещала мне Традиционная Рациональность, но даже то, что оставалось — было магией.

Не сомневайтесь, моя верность Традиционной Рациональности помогла мне выбраться из ямы, что я выкопал для себя. Если бы я не был Традиционным Рационалистом, со мной было бы покончено. Но Традиционной рациональности было все еще недостаточно. Это уводило меня от запрещенных ошибок, но приводило к другим.

Когда я думаю о себе, осторожно следовавшем правилам Традиционной Рациональности, получая при этом неверный ответ, то начинаю понимать, почему люди, называющие себя «рационалистами» не правят миром. Нужно дофига и больше рациональности, прежде чем она приведет хоть куда-то, а не к новым интересным ошибкам.

Традиционная Рациональность преподается как искусство, а не как наука. Читаешь биографии известных физиков, описывающие уроки, которым их научила жизнь, и стараешься делать то же, что и они. Но ты не прожил их жизнь, и половина того, что они описывают — инстинкт, который выработался у них.

Традиционная Рациональность сделана так, что было бы приемлемым для меня провести следующие тридцать лет, исследуя мою глупую идею, пока я был бы способен фальсифицировать её, со временем, и был бы честен сам с собой по поводу предсказаний, которые делает моя гипотеза, столкнувшись с опровержением, и прочее и прочее. Этого достаточно для того, чтобы Колесо Науки двигалось вперед, но немного жестоко по отношению к тем, кто тратит по тридцать лет своей жизни. Традиционная рациональность — прогулка, не танец. Она предназначена привести тебя к ответу, в итоге, но позволяет слишком тратить время на любование цветами по пути.

Традиционные Рационалисты могут согласиться не соглашаться. Традиционная Рациональность не имеет идеи о том, что рассуждения — точно искусство, предполагающее, что есть лишь одна корректная вероятность, с учетом имеющихся свидетельств. В Традиционной Рациональности тебе позволено гадать и тестировать свои догадки. Но опыт подсказывает мне, что если ты гадаешь, не зная, то получаешь неправильный ответ.

Путь Байеса тоже неточное искусство, насколько я его освоил пока. Эти статьи все еще пытаются передать словами то, что лучше будет постигнуто опытом. Но, по крайней мере, тут в основе лежит математика, плюс, экспериментальные свидетельства когнитивной психологии по поводу того, как люди действительно думают. Может быть, этого будет достаточно, чтобы преодолеть стратосферических размеров порог дисциплины, позволяющей получать правильные ответы, вместо новых интересных ошибок.

Перевод: 
Muyyd
Номер в книге "Рациональность: от ИИ до зомби": 
40
Оцените качество перевода: 
Средняя оценка: 3.4 (49 votes)

Неспособность учиться у истории

Элиезер Юдковский

Однажды, во времена моей дикой и безбашенной юности, когда я еще не ведал Пути Байеса, я дал Таинственный Ответ на вроде бы таинственный вопрос. Это стало следствием цепочки из множества ошибок, однако, среди них была одна критическая: юная версия меня не осознавала, что получение ответа должно делать вопрос менее запутанным. Я пытался объяснить Таинственный Феномен, что означало — дать причину для него, вписывающуюся в интегрированную модель реальности. Почему же это должно было сделать феномен менее Таинственным, когда такова его природа? Я ведь пытался объяснить Таинственный Феномен, а не превратить (посредством какой-то неведомой алхимии) в обыденный, такой, который вообще не потребует странного объяснения.

Будучи Традиционным Рационалистом, я был в курсе историй о астрологии и астрономии, алхимии и химии, витализма и биологии. Но Таинственный Феномен был не таким. Он был чем-то новым, чем-то странным, чем-то более сложным, чем-то, чему обычная наука не могла дать объяснение на протяжении веков…

…как будто звезды и материя, и жизнь не были таинственными в течении сотен и тысяч лет, с самого восхода человеческой мысли, до того момента, как наука взяла и решила их…

Мы узнаём про астрономию и химию, и биологию в школе; нам кажется, что эти знания всегда были частью сферы научного знания, что они никогда не были таинственными. Когда наука бросает вызов новой Великой Загадке, дети этого поколения проявляют скепсис, ведь они не видели, что наука способна объяснить что-то, кажущееся мистическим для них. Наука годится лишь для объяснения научных субъектов, вроде звёзд или материи с жизнью.

Я думал, что урок истории в том, что астрологи с виталистами и алхимиками имели особый изъян характера, тенденцию в пользу таинственности, что и приводило их к таинственным объяснениями совершенно не-таинственных вопросов. Но правда ведь, если феномен странный — объяснение тоже должно быть странным?

Лишь позже, когда я начал видеть обыденную структуру внутри тайны, я начал понимать, на месте кого же я оказался. Лишь позже я понял, насколько разумным казался витализм в то время, насколько неожиданным и смущающим оказался ответ вселенной: «Жизнь обыденная, ей не требуется странных объяснений».

Мы изучаем историю, но мы не живём ей, не переживаем этот опыт. Если бы только я постулировал астрологическую тайну и позже открыл бы механику Ньютона, постулировал алхимическую тайну и открыл бы химию, постулировал бы витализм и открыл бы биологию. Я бы посмотрел на мой Таинственный Ответ и решил бы: ни за что на свете, я не попадусь на это снова.

Перевод: 
Muyyd
Номер в книге "Рациональность: от ИИ до зомби": 
41
Оцените качество перевода: 
Средняя оценка: 4.6 (16 votes)

Делая историю доступной

Элиезер Юдковский

Есть такая привычка мышления, которую я называю ошибкой обобщения на основе вымышленного свидетельства. Журналисты, которые, например, ведут речь про Терминатора, в обсуждении про ИИ, не относятся к этому сюжету, как к пророчеству или свершившейся правде. Но фильм приходит на ум, он доступен, как будто-то бы он является иллюстрированным историческим случаем. Как если бы журналисты видели, как это случилось на какой-то другой планете, и может произойти и на нашей. Подробней про это в секции 7 в «Когнитивных искажениях, влияющих на оценку глобальных рисков»1.

Есть и обратная обобщению на основе вымышленного свидетельства ошибка: неспособность в достаточной степени учитывать исторические свидетельства. Проблема с обобщением на основе вымышленного свидетельства в том, что оно вымышленное — оно никогда не происходило. Оно не начертано на основе того же распределения, что и наша вселенная; художественная литература систематически отличается от реальности. Но история же действительна случилась и должна быть доступной.

В мире наших предков не было фильмов; всё, что ты видел своими глазами, было правдой. Стоит ли удивляться, что вымысел, который мы видим в реалистичном кино, оказывает на нас такое серьезное влияние? Наоборот, то, что реально происходило, доступно нам на бумаге; оно произошло, но мы не видели этого. Мы не помним, что это происходило с нами.

Обратная ошибка — обращаться с историей как с обычными рассказами, анализировать той же частью разума, что используется при чтении новелл. Ты можешь произнести, что это «правда», а не «вымысел», но это не означает, что ты воспринимаешь это так серьезно, как следовало бы. Множество искажений являются следствием недостаточно серьезного восприятия сухой, абстрактной информации.

Однажды, я дал Таинственный Ответ на таинственный вопрос, не осознавая, что совершаю ту же самую ошибку, что и астрологи, придумавшие мистическое объяснение звёздам, или алхимики, придумавшие мистические свойства материи, или виталисты, постулировавшие мутное «élan vital» для объяснения биологии.

Когда я посмотрел на ситуацию с другой стороны, то ощутил шок неожиданной связи с прошлым. Я осознал, что изобретение и уничтожение витализма, о котором я читал лишь в книгах, действительно происходило с реальными людьми, которые переживали опыт так же, как и я переживал изобретение и разрушение моего собственного таинственного ответа. И я понял, что если бы я действительно переживал опыт прошлого, если бы я жил во времена научных революций, а не читал о них в книгах, я, вероятно, не совершил бы эту же ошибку снова. Я бы не стал изобретать очередной таинственный ответ; достаточно было бы тысяч предыдущих.

Итак, — подумал я, — для того, чтобы действительно ощутить силу истории, я должен думать, как Элиезер, живший в прошлом, должен думать о событиях так, как если бы они случились со мной (с соответствующей переоценкой искажения доступности исторической литературы — мне следует помнить себя тысячей крестьян на одного лорда). Я должен погрузить себя в историю, вообразить жизнь сквозь эры, которые я наблюдал лишь посредством чернил на бумаге.

Почему мне следует помнить полёт Братьев Райт? Меня там не было. Но, как рационалист, посмею ли я не помнить, если событие действительно произошло? Действительно ли есть большая разница, видеть событие собственными глазами — что, по сути, представляет собой воздействие отраженных фотонов и даже не прямой контакт, — и наблюдать событие через книгу по истории? Фотоны и книги по истории спускаются по цепочке причин и следствий от самого события.

Мне нужно было пересилить ложную амнезию, вызванную рождением в конкретную эпоху. Я должен был вспомнить, сделать доступными все воспоминания, а не только те, что чисто случайно принадлежат мне и моему времени.

Земля внезапно стала старше.

С точки зрения моих старых воспоминаний, Соединённые Штаты существовали всегда — не было времени, когда не было бы Соединённых Штатов. Я не помнил, до того момента, как поднялась Римская Империя, принесла мир и порядок, и, просуществовав множество веков, что я даже забыл, что могло быть иначе, пала, и варвары захватили мой город, и знание, которым я обладал, было потеряно. Современный мир стал более хрупким для меня, ведь я перестал воспринимать его, как мой первый современный мир.

Так много ошибок, снова и снова, ведь я не помнил, что совершал их в каждой эре, в которой я никогда не жил.

Только представьте, люди ещё удивляются, почему преодоление искажений важно.

Неужели ты не помнишь, как много раз ошибки убивали тебя? Я заметил, что внезапная амнезия часто следует сразу за роковой ошибкой. Но поверьте мне, это случалось. Я помню, хоть меня там и не было.

Так что в следующий раз, когда ты усомнишься в странности будущего, вспомни, как ты был рожден в племени охотников-собирателей тысячи лет назад, когда еще никто не знал о Науке. Вспомни, как ты был шокирован до глубины души, когда Наука объяснила великие и ужасные священные тайны, которые ты так восхвалял. Вспомни, как ты думал, что сможешь летать, если съешь нужный гриб, как ты разочарованно усвоил, что никогда не сможешь полететь, а потом полетел. Вспомни, как ты всегда думал, что рабство — это правильно и хорошо, а потом передумал. Не надо воображать, как бы ты мог предсказать перемены — ведь это проявление амнезии. Вспомни, что на самом деле ты не угадал. Вспомни, как век за веком мир менялся так, как ты и представить не мог.

Может так ты будешь менее шокирован тем, что будет дальше.

  • 1. Eliezer Yudkowsky, “Cognitive Biases Potentially Affecting Judgment of Global Risks,” in Global Catastrophic Risks, ed. Nick Bostrom and Milan M. Ćirković (New York: Oxford University Press, 2008), 91–119.
Перевод: 
Muyyd, Abel
Номер в книге "Рациональность: от ИИ до зомби": 
42
Оцените качество перевода: 
Средняя оценка: 4.6 (19 votes)

Объяснить, поклониться, пренебречь

Элиезер Юдковский

Наше племя бродит по лугам в поисках съедобных растений и добычи, а с неба время от времени льётся вода.

— Почему с неба иногда падает вода? — спрашиваю я у бородатого мудреца нашего племени.

Старик погружается в размышления, — он никогда не задавался этим вопросом раньше — и через некоторое время отвечает:

— Иногда небесные духи сражаются, и во время этих битв с небес капает их кровь.

— Откуда появились небесные духи? — спрашиваю я.

Голос старца превращается в шепот:

— Они зародились в далёком прошлом в таких далях и безднах, что нам и не снились.

Твоё незнание причин, по которым идёт дождь, предоставляет тебе несколько вариантов действий. Во-первых, ты можешь просто не спрашивать «почему?» — не уделять вопросу никакого внимания или просто вообще никогда не задаваться этой мыслью. Это — команда «пренебречь», и именно её изначально выбрал старец. Во-вторых, ты можешь попытаться придумать какое-нибудь объяснение, то есть выбрать команду «объяснить», как сделал старец в ответ на первый вопрос. В-третьих, ты можешь смаковать тайну, выбрав команду «поклониться».

Прочитав эту историю, трудно не заметить то, что каждый выбор «объяснить», если всё получится, предоставляет тебе объяснение (например, «небесные духи»). Но это объяснение вновь возвращает нас к трилемме: объяснить, поклониться, пренебречь? После каждого нажатия «объяснить» наука трещит своими шестернями, возвращает ответ, и затем всплывает новое диалоговое окно. Рационалисты считают своим долгом постоянно нажимать «объяснить», но это выглядит, как дорога без конца.

Нажмёшь «объяснить» для жизни — получишь химию. Нажмёшь «объяснить» для химии — получишь атомы. Нажмёшь «объяснить» для атомов — получишь электроны и нуклоны. Нажмёшь «объяснить» для нуклонов — получишь хромодинамику и кварки. Нажмёшь «объяснить» для того, чтобы узнать, откуда взялись кварки — вернёшься во времена Большого Взрыва…

Если нажать «объяснить» для Большого Взрыва, то придётся некоторое время подождать, пока наука, треща своими шестернями, будет искать ответ. И, возможно, она когда-нибудь вернёт замечательное объяснение — но это повлечёт за собой ещё одно диалоговое окно. И, если мы продолжим достаточно долго, то мы должны увидеть особенное диалоговое окно с Объяснением, Не Требующим Объяснения, и оно закончит эту цепочку. Возможно, это будет самое важное объяснение из числа всех объяснений: как уже известных, так и тех, что ещё станут известными человеку.

Погодите! Я только что нажал «поклониться».

Не забывайте, что поклоняться можно по-разному. Поклоняться можно и не зажигая свечей вокруг алтаря.

Если бы я сказал «Хм, это какой-то парадокс. Интересно, как он разрешается?», то это означало бы, что я нажал «объяснить» и теперь терпеливо жду ответа.

И если весь вопрос кажется тебе неважным, или неуместным, или тебе кажется, что лучше подумать о нём потом — значит, ты нажал «пренебречь».

Выбирай кнопку с умом.

Перевод: 
BT
Номер в книге "Рациональность: от ИИ до зомби": 
43
Оцените качество перевода: 
Средняя оценка: 4.4 (29 votes)

«Наука» — затычка для любопытства

Элиезер Юдковский

Представьте что я, находясь под наблюдением телекамер, поднял руки, произнёс «абракадабра!» — и произвёл слепящий сгусток света, висящий в воздухе невдалеке от моих вытянутых рук. Представьте, что я осуществил этот акт отъявленной, безошибочно-подлинной магии под прямым наблюдением со стороны Джеймса Рэнди (известный скептик и разоблачитель псевдонаучных теорий — прим. перев.) и всех армий скептиков. Думаю, большинству людей будет довольно любопытно, что же здесь происходит.

А теперь представим, что я не отправлялся на телевидение. Я не хочу делиться ни своей силой, ни правдой, которая за ней стоит. Я хочу, чтобы моя магия оставалась в секрете. Но при этом я хочу также иметь возможность пользоваться ею где и когда захочу. Я хочу призывать сгусток света, чтобы почитать книгу в поезде — но так, чтобы при этом никому не становилось любопытно. Есть ли заклинание, останавливающее любопытство?

Конечно есть! Когда кто-то спрашивает: «Как ты это сделал?», я просто отвечаю: «Наука!»

Это не столько настоящее объяснение, сколько эдакая «затычка для любопытства». Оно не говорит, станет ли свет ярче или потускнеет, изменятся ли тон или насыщенность его цвета, и, конечно же, не объясняет, как сделать такой огонёк самому. Вопрошающий не получил новых знаний, которых у него не было до того, как я произнёс магическое слово. Но он отворачивается, удовлетворившись тем, что не происходит ничего необычного.

Что ещё лучше, тот же трюк работает с обыкновенным выключателем света. Клацаем выключателем — и загорается лампочка. Почему? В школе нам объясняют, что паролем для лампочки накаливания является «Электричество!». Надеюсь, что к данному моменту вы уже не столь склонны к тому, чтобы обозначать лампочку «понятой» на основе такого «объяснения». Позволяет ли фраза «Электричество!» проводить расчёты, контролирующие ожидания? Нужно ещё много чего изучить! (Физики должны проигнорировать этот абзац и заменить его проблемой в эволюционной теории, где суть теории, опять-таки, заключается в вычислениях, которые умеет проводить очень малое количество людей).

Если бы вы считали, что лампочка накаливания научно необъяснима, она захватила бы всё ваше внимание. Вы бы бросили все дела и занялись бы исключительно лампочкой.

Но что значит фраза «научно объяснима»? Она значит, что кто-то ещё знает, как работает лампочка. Когда вам говорят, что лампочка «научно объяснима», вы не узнаёте ничего нового; вы не знаете, загорится ли лампочка ярче или потускнеет. Но раз кто-то уже знает о лампочке, эти знания в ваших глазах уже не так ценны. Вы становитесь менее любопытным.

Кто-то обязан сказать: «Если лампочка была неизвестна науке, вы можете получить славу и богатство исследуя её». Но я не говорю о жадности. Я не говорю о карьерных амбициях. Я говорю о чистом любопытстве—чувстве интереса. Почему ваше любопытство должно уменьшаться, если кто-то, не вы, знает как работает лампочка? Это ли не призрак? Для вас недостаточно знать; другие люди должны также быть невежественны, иначе вы не будете счастливы?

Хорошо, что знание может служить не только любопытству, как например социальная полезность технологий. Для таких инструментальных благ это важно как некоторая сущность в локальном пространстве знаний. Но что это должно значить для моего любопытства?

Кроме того, учитывайте последствия если вы допускаете «кто-то ещё знает ответ» в качестве семантического стоп-сигнала.

Однажды вы заходите в вашу комнату и видите гигантского зелёного слона, по видимому парящего в воздухе, окружённого аурой серебристого света. Вы скажете: «Какого чёрта?»

И голос, исходящий сверху от слона, говорит:

Кто-то уже знает почему слон здесь.

«О, — говорите вы. — Тогда это не важно». И идёте в кухню, как ни в чём не бывало.

Я не знаю великую единую теорию для законов физики этой вселенной. Я также многого не знаю о человеческой анатомии за исключением мозга. Я не могу указать где у меня почки, и я не могу тотчас вспомнить что делает моя печень.(Я не горжусь этим. Увы, со всей той математикой, которую мне нужно исследовать, я, скорее всего не научусь анатомии в какое-либо ближайшее время)

Следует мне, столь далеко как простирается любопытство, быть более заинтересованным моим незнанием элементарных законов физики, когда факт, что я немногое знаю о том, что происходит внутри моего тела?

Если бы я поднял свои руки и сотворил заклинание света, вы бы заинтересовались. Следует ли вам быть менее заинтересованным самим фактом поднятия моих рук? Когда вы поднимаете руку и водите ладонью вокруг, этот акт воли координируется (превыше других зон мозга) вашим мозжечком. Я поспорю, вы не знаете как работает мозжечок. Я знаю немного поверхностных деталей, недостаточных для проведения вычислений… но что с того? Какое имеет значение, если вы не знаете? Почему должен быть двойной стандарт любопытства для колдовства и движений рук?

Осмотрите себя в зеркале. Знаете ли вы, на что смотрите? Знаете ли вы, что выглядывает из-за ваших глаз? Знаете, чем вы являетесь? На некоторые из этих вопросов Наука знает ответы, на некоторые нет. Но почему это различие значимо для вашего любопытства, если вы не знаете?

Вы знаете, как работают ваши колени? Вы знаете, как сделана ваша обувь? Вы знаете, как светится ваш монитор? Вы знаете, почему вода мокрая?

Мир вокруг вас полон загадок. Расставляйте приоритеты, если вам нужно. Но не жалуйтесь, что жестокая наука опустошила мир тайн. С таким рассуждением я могу заставить вас проглядеть слона в вашей комнате.

Перевод: 
Muyyd
Номер в книге "Рациональность: от ИИ до зомби": 
44
Оцените качество перевода: 
Средняя оценка: 4.6 (18 votes)

Поистине часть тебя

Элиезер Юдковский

Классическая работа Дрю МакДермота «Искусственный интеллект и естественная глупость» критикует ИИ программы, которые пытаются представить понятия вроде счастья, как состояние ума, при помощи семантической сети: 1


СОСТОЯНИЕ-УМА
^
| ЭТО
|
СЧАСТЬЕ

И разумеется, внутри узла СЧАСТЬЕ ничего нет. Это просто токен языка Лисп с говорящим английским названием.

И вот, МакДермот пишет: «Для дисциплинированного программиста хорошим тестом будет использование gensyms в ключевых местах, чтобы посмотреть будет ли все ещё восхищаться системой, если СОСТОЯНИЕ-УМА будет переименовано в G1073…» мы получим ЭТО (СЧАСТЬЕ, G1073) «что выглядит далеко не столь впечатляюще».

Или если я слегка перефразирую идею: если ты заменишь случайными символами вообще все осмысленные английские слова, то ты никак не поймёшь, что означает G1071 (G1072, 1073). Эта программа ИИ описывает гамбургеры? Яблоки? Счастье? Кто знает? Если ты удалишь все говорящие английские названия, обратно они не вернутся.

Представим, что физик сказал тебе: «Свет — это волны», и ты ему полностью поверил. У тебя в голове появилась маленькая сеть:

ЭТО(СВЕТ, ВОЛНЫ)

И если кто-то вдруг спросит «Из чего состоит свет?», ты сможешь ответить «Из волн!»

Как пишет МакДермот: «Проблема в том, чтобы слушатель заметил, что именно ему сказали. Не „понял“, а всего лишь „заметил“». Предположим, что физик сказал бы тебе «Свет сделан из мелких изгибающихся штучек» (на всякий случай: это неправда). Заметишь ли ты разницу в ожидаемом опыте?

Как можно понять, что не стоит доверять кажущемуся знанию «Свет — это волны»? Один из способов: «смогу ли я восстановить это знание, если оно почему-то исчезнет из моей головы?»

Это похоже по духу на замену говорящих английских названий из программы ИИ с целью узнать, сможет ли кто-то понять к чему они должны «относиться». Также здесь можно привести пример Искусственного Вычислителя, который запрограммирован записывать и выдавать ПРИБАВИТЬ(СЕМЬ, ШЕСТЬ) = ТРИНАДЦАТЬ. Он, разумеется, не сможет восстановить это знание, если стереть его из его памяти (если только ещё кто-нибудь не запишет его обратно). Аналогично и знание «Свет — это волны» будет потеряно безвозвратно, если только ты не спросишь о нём заново у физика. Ты не сможешь создать это знание сам, как может это сделать физик.

Тот же опыт, что заставляет нас формулировать убеждения, соединяет их с другими нашими знаниями, чувственными входными данными или моторными выходными данными. Если ты увидишь, как бобёр грызёт дерево, то ты будешь знать как «зверь-что-грызёт-дерево» выглядит и сможешь опознать его в будущем, независимо от того, будут ли его называть «бобром» или как-то иначе. Но если ты приобрёл свои убеждения о «бобрах», когда кто-то другой рассказал тебе факты о «бобрах», то, возможно, ты не опознаешь бобра, если его увидишь.

Знание, которое ИИ не сможет восстановить сам — чудовищная опасность. Это так же опасно, как говорить кому-то факты о физике, которые он не в состоянии проверить сам. Ведь физики, говоря «волны», имеют ввиду не «мелкие кривые штучки», а чисто математическое понятие.

Как заметил Давидсон, если ты веришь, что «бобры» живут в пустынях, что они белого цвета, а взрослые особи весят полтора центнера, то у тебя вообще отсутствуют какие-либо убеждения о бобрах: ложные или правдивые. Твои убеждения о бобрах недостаточно верны, чтобы быть ложными2. Если у тебя нет достаточно опыта, чтобы восстановить убеждение в случае, если оно будет удалено, то есть ли у тебя опыт, позволяющий соединить это убеждение с чем-нибудь вообще? Витгенштейн: «Колесо, которое можно свободно вращать, не задевая других частей, не является частью механизма».

Почти сразу же, когда я начал читать про ИИ, даже до того, как я начал читать МакДермота, я понял, что постоянно спрашивать себя «Как бы я смог восстановить это знание, если оно будет удалено из моего разума?» — это очень хорошая идея.

Чем обширней удаление, тем строже тест. Если удалить доказательства теоремы Пифагора, смогу ли я доказать её заново? Думаю, да. Если удалить всё знание о теореме Пифагора, догадаюсь ли я о ней, чтобы доказать её заново? Сложно поставить такой эксперимент. Однако, если бы мне дали прямоугольный треугольник с длиной сторон 3 и 4 и сказали бы, что гипотенуза вычисляема, думаю, я бы смог её вычислить, при условии, что другие мои знания математики остались при мне.

Как насчёт самого понятия математического доказательства? Если бы мне никто не говорил о нём, мог бы я изобрести его, основываясь на моих оставшихся убеждениях? Ведь когда-то люди не знали о такой идее. Кто-то же её придумал. Что же он заметил? Замечу ли я, если увижу что-то такое же новое и не менее важное? Окажусь ли я способен на оригинальное мышление?

Как много своих знаний ты смог бы восстановить самостоятельно после удаления? Какова допустимая глубина удаления? Это не просто проверка для отсеивания недостаточно связанных убеждений. Такие размышления позволяют впитать целый фонтан знаний, а не всего лишь один факт.

Пастух строит систему учёта овец, которая работает, если добавлять камень в корзину каждый раз, когда овца покидает амбар, и изымать - когда овца возвращается. Если ты, ученик, не понимаешь работу этой системы, если для тебя это магия, которая непонятно почему работает, то ты не поймёшь, что делать, если случайно бросишь лишний камень в корзину. Если ты не можешь что-то создать сам, ты не сможешь это и восстановить, если ситуация этого потребует. Ты не сможешь вернуться к истокам, подправить параметры и заново получить результат, если у тебя нет истоков. Предположим, тебе известен факт «два плюс четыре равно шести». Один из элементов изменяется на «пять». Как ты узнаешь, что «два плюс пять равно семи», когда всё, что ты знаешь: «два плюс четыре равно шести»?

Если ты видишь, как маленькое растение роняет семя каждый раз, когда мимо пролетает птица, ты не догадаешься, что ты можешь использовать эту отчасти автоматизированную систему для подсчёта овец. Если бы первый изобретатель узнал об этом факте, он мог бы его использовать для улучшения своей системы. Однако, ты не в состоянии вернуться к истокам и переизобрести новую систему подсчёта.

Если источник мысли находится внутри тебя самого, то с приобретением новых знаний и навыков мысль может меняться. Она поистине становится частью тебя и растёт вместе с тобой.

Старайся стать источником для всех своих мыслей, стоящих обдумывания. Если мысль первоначально пришла извне, убедись, что она также исходит изнутри. Постоянно спрашивай себя: «Как я восстановлю это знание, если оно исчезнет?» Найдя ответ на этот вопрос, представь, что знание, которое помогло бы тебе в таком случае, тоже удалено. И когда увидишь фонтан - посмотри, что ещё из него течёт.

  • 1. Drew McDermott, «Artificial Intelligence Meets Natural Stupidity», SIGART Newsletter, no. 57 (1976): 4–9, doi:10.1145/1045339.1045340.
  • 2. Richard Rorty, «Out of the Matrix: How the Late Philosopher Donald Davidson Showed That Reality Can’t Be an Illusion», The Boston Globe (October 2003).
Перевод: 
Muyyd, Alaric
Номер в книге "Рациональность: от ИИ до зомби": 
45
Оцените качество перевода: 
Средняя оценка: 4.4 (16 votes)

Простая истина

Элиезер Юдковский

Как-то я писала сочинение об экзистенциализме. Учительница тогда вернула мне его с двойкой. Она подчеркнула слова „истина“ и „истинный“ и поставила знаки вопроса на полях напротив каждого, всего раз 20. Она хотела знать, что я имела в виду под истиной.

Даниэла Эган (журналист)

Предисловие автора

Это эссе написано, чтобы восстановить наивное представление об истине.

Допустим, кто-то говорит вам: «Моё чудодейственное лекарство избавит вас от рака лёгких всего за три недели». Вы говорите: «Но я знаю о результатах клинических исследований: ваше утверждение не соответствует истине». А этот кто-то отвечает: «Понятие „истина“ довольно неточно, что вы называете истиной?»

Многие люди, поставленные перед этим вопросом, не будут знать, как ответить с достаточной точностью. Тем не менее, будет крайне неразумным отказаться от концепции истины. Было время, когда никто не знал точную формулу тяготения – и всё же, шагнув с обрыва, вы бы разбились и тогда.

Часто, особенно в интернет-дискуссиях, я встречал чьи-то заявления «X истинно», а дальше спор поворачивал в сторону поиска определения истины. Это эссе ни в коем случае не является энциклопедическим ответом на этот вопрос. Скорее я надеюсь, что спорщики прочтут это эссе, а затем вернутся к изначальному вопросу.

В этом эссе я ставлю вопросы. Если вы находите на них кажущийся очевидным ответ – скорее всего, это он и есть. Очевидный выбор не всегда наилучший, но всё-таки иногда, черт возьми, он таков. Я не перестаю искать, когда встречаю очевидный ответ, но если по мере повышения моей информированности ответ всё ещё выглядит очевидным, то я не вижу вины в том, чтобы использовать его. Ну да, разумеется, все считают, что 2 + 2 = 4, говорят, что 2 + 2 = 4, и в повседневной жизни все ведут себя, как если бы 2 + 2 равнялось 4. И всё же чему 2 + 2 равно в высшем, абсолютном смысле? Насколько я могу судить, всё еще 4. Ответ будет равен четырём, даже если я задам этот вопрос официальным, строгим, напыщенным тоном. Слишком просто, говорите? Может быть, в этом случае мир и не должен быть сложен. Для разнообразия.

Если вы — один из тех счастливчиков, которым этот вопрос кажется тривиальным с самого начала, я надеюсь, что он окажется тривиальным и на самом деле. И если перед вами вдруг встанет глубокий и важный вопрос, вспомните, что если вы в точности знаете, как работает система, вы сами можете построить её аналог из ведёрок и камней, и это не должно быть для вас какой-то тайной.

Если вы теряетесь, пытаясь понять метафору «ведёрок и камней» как метафору, попробуйте понять её буквально.

Простая истина

Представьте, что мы оказались в доисторических временах, когда ещё не были изобретены начала математики. Здесь я пастух, и у меня есть проблема. Мне трудно уследить за своими овцами. Они спят в загоне и защищены от волков высоким забором. Каждое утро я выпускаю овец на пастбище. Каждый вечер я должен загнать их всех назад. Если я оставлю овцу снаружи, наутро меня встретит обглоданная волками тушка. Но меня так раздражает часами бродить по полям, когда я почти уверен, что все овцы уже в загоне. Иногда я бросаю поиски рано, и обычно наутро всё в порядке. Но примерно в одном случае из десяти утром я нахожу мёртвую овцу.

Вот если бы был какой-нибудь хитрый способ точно определить, остались ли снаружи ещё овцы! Я пробовал несколько методов: я бросал гадальные палочки, я развивал силу духа, чтобы видеть стадо внутренним взором, я старался найти убедительные основания полагать, что все овцы нашлись. Бесполезно. Примерно один раз из десяти, когда я ложился спать рано, наутро я обнаруживал тушку овечки. Может быть, я осознаю, что мои методы не работают, и, возможно, я тщательно подберу уважительную причину каждой своей неудачи. Но дилемма прежняя: либо я в течение часа обшариваю все закоулки и расщелины, хотя в большинстве случаев все овцы уже в загоне, либо я иду спать рано и теряю в среднем одну десятую овцы.

Однажды поздним вечером я еле стою на ногах. Заговорённые палочки утверждают, что все овцы вернулись домой. Я рисую в воображении каждый уголок, каждую расщелину на полях, и гадание говорит, что овец там нет. Но я ещё не уверен, поэтому я захожу в загон. Овец много, очень много, и я уверен, что следил за стадом тщательно и не отвлекался ни на минуту. Всё это развеивает мои тревоги, и я отправляюсь спать. На следующее утро я нахожу двух мёртвых овец. Что-то внутри меня ломается, и я начинаю творчески обдумывать проблему.

Громкие звуки молотка слышатся из загона в тот день.

На следующее утро я только слегка приоткрываю калитку загона. Каждый раз, когда выходит овца, я бросаю камешек в приколоченное рядом с калиткой ведро. Тем же вечером, когда я загоняю овцу обратно, я вынимаю один камушек из ведра. Когда ведро опустеет, я прекращаю поиски и иду спать. Это превосходное изобретение. Я уверен, оно совершит революцию в пастушестве.

Такова была теория. На практике потребовалось существенно доработать систему до надёжности. Несколько раз получалось так, что я не находил отсутствующих овец после нескольких часов поисков, однако на следующий день трупов не было. Каждый случай требовал глубокого обдумывания, в чем же моя ведёрная система не срабатывает. Однажды после очередного бесплодного поиска я остановился и прокрутил в голове весь день. Оказалось, что в тот день в ведре уже были камни, когда я выпускал первую овцу. Это была плохая идея. В другой раз, чтобы скоротать время с утра до обеда, я забавлялся игрой с камушками, от скуки кидая их в ведро. Это тоже было плохой идеей, что я понял после нескольких часов поиска. Я практиковался в своем ведёрно-каменном ремесле и со временем стал довольно компетентным овцесчетоводом.

Однажды на дороге, ведущей к моим пастбищам, появляется человек, одетый в дорогие белые одежды бизнес-покроя, сандалии, с лавровым венком на голове.

— Чем я могу вам помочь? — спрашиваю я.

Человек достаёт из-под одежд и с щелчком открывает значок, без тени сомнения подтверждающий, что этот человек является Маркосом Замысловатусом Максимусом, представителем Сената Рума. (Интересно, а что если бы кто-то другой украл этот значок? Но сила подобных знаков настолько велика, что этот кто-то, укради он знак, мгновенно превратился бы в Маркоса.)

— Зовите меня просто Марк, — говорит он. — Я здесь, чтобы конфисковать магические камни от имени Сената. Таким могущественным артефактам не должно находиться в столь невежественных руках.

— Ох уж это чёртов подмастерье! — ворчу я себе под нос. — Опять он трепался с деревенскими о чём не следует. Я смотрю в строгое лицо Марка и вздыхаю. — Послушайте, это никакие не магические камни. Самые обычные камни, которые я набрал с земли.

Тень недоумения пробегает по лицу Марка, но затем оно снова проясняется. — Я здесь, чтобы конфисковать магическое ведро! — заявляет он.

— Да не магическое это ведро, — устало говорю я. — Раньше я в нём хранил грязные носки.

Вот теперь видно, что Марк действительно озадачен.

— Тогда в чём же магия? — требовательно спрашивает он.

А ведь интересный вопрос.

— Это непросто объяснить, — начинаю я.

Привлечённый нашим разговором, мой подмастерье Отри подходит и предлагает своё объяснение:

— Магия в уровне камешков в ведре. Есть определённый магический уровень камешков в ведре, и они должны быть точно на нужной отметке, иначе магия не сработает. Если положить в ведро больше камней или достать несколько, они не будут на магическом уровне. Вот сейчас магический уровень, — Отри заглядывает в ведёрко — это примерно полное на треть.

— Понятно! — восклицает Марк. Он достаёт из вещевого мешка своё ведро и кучу камешков. Затем он берет несколько горстей камней и кладёт их в ведро. Он смотрит в ведро, примечая, сколько там камней. — Ну вот, — говорит он, — магический уровень этого ведра — это наполовину полное. Так это работает?

— Нет! — резко отвечает Отри. — Наполовину полное — это не магический уровень. Магический уровень — это полное примерно на треть. Наполовину полное — это совершенно не волшебно. Кроме того, у вас неправильное ведро.

Озадаченный Марк обращается ко мне:

— Кажется вы говорили, что ведро не магическое?

— Оно не магическое, — отвечаю я. Из загона выходит овца и я бросаю ещё один камешек в ведро. — И вообще, я смотрю за овцами. Поговорите с Отри.

Марк с сомнением провожает глазами брошенный камешек, но решает на время отложить свой вопрос. Он поворачивается в Отри и надменно выпрямляется. — Это свободная страна, — говорит он, — под благословенной диктатурой Сената, конечно. Я могу бросать любые камешки в какое мне угодно ведро.

Отри обдумывает это заявление. — Нет, не можете, — наконец отвечает он, — тогда совсем не будет волшебства.

— Послушайте, — терпеливо продолжает Марк. — Я внимательно наблюдал за вами. Вы посмотрели в своё ведро, проверили уровень камешков и сказали, что это и есть магический уровень. Я сделал всё точно так же.

— Это не так работает, — говорит Отри.

— О, понятно, — говорит Марк, — Магический уровень камешков не в моем ведре, а уровень камешков в вашем ведре. Так вы утверждаете? И чем же это ваше ведро гораздо лучше моего, а?

— Ну, если бы мы освободили ваше ведро, а затем наполнили его камешками из моего, то в вашем ведре был бы волшебный уровень. Также существует способ проверить, есть ли у вашего ведра волшебный уровень, если известно, что у моего ведра он имеется. Мы называем это операцией сравнения вёдер. — сказал Отри.

Выходит другая овца, и я кидаю в ведро ещё один камешек.

— Он только что закинул ещё один камешек! — говорит Марк. — И я полагаю, вы утверждаете, что новый уровень тоже является волшебным? Я мог бы бросать камешки в ваше ведро, пока уровень не стал бы таким же, как в моём, и тогда наши ведра сравнялись бы. Вы просто сравниваете мое ведро со своим, чтобы определить, считаете ли вы уровень «магическим» или нет. Так вот, я считаю, что ваше ведро не волшебное, потому что в нём не такой же уровень камешков, как в моём. Вот так!

— Погодите, вы не понимаете, — говорит Отри.

— Под «волшебным уровнем» вы всего лишь понимаете уровень камешков в вашем собственном ведре. А когда я говорю «волшебный уровень», я имею в виду уровень камешков в моем ведре. Итак, вы смотрите на мое ведро и говорите, что оно «не волшебное», но «волшебство» имеет разное значение для разных людей. Необходимо указать, чья это магия. Вы должны сказать, что у моего ведра нет «волшебного уровня Отри», и я скажу, что у вашего ведра нет «волшебного уровня Марка». Таким образом, кажущееся противоречие исчезает.

— Но… — беспомощно говорит Отри.

— Разные люди могут иметь разные вёдра с разным уровнем камешков, что доказывает, что всё это дело с «магией» совершенно произвольно и субъективно.

— Марк, — говорю я, — а кто-нибудь сказал вам, что эти камешки делают?

— Делают? — говорит Марк. — Я думал, они просто волшебные и всё.

— Если бы камешки ничего не делали, — говорит Отри, — то наш аудитор эффективности бизнес-процессов на соответствие стандарту ISO 9000 исключил бы эту процедуру из повседневной работы.

— Как зовут вашего аудитора?

— Дарвин, — сказал Отри.

— Хм, — говорит Марк, — Чарльз имеет репутацию строгого ревизора. Так что, камушки благословляют стада и ведут к приумножению овец?

— Нет, — говорю я. — Сила камушков заключается в следующем: если мы смотрим в ведро и видим, что камушков в нём нет, то мы знаем, что на пастбищах также овец не осталось. Если мы не используем ведро, мы должны искать и искать до темноты оставшихся овец. Или, если мы останавливаем наши поиски раньше, то иногда на следующее утро мы находим мертвых овец, павших жертвами волков. Если мы смотрим в ведро, мы знаем, когда все овцы в загоне, и можем спать спокойно.

Марк задумался.

— Звучит слишком неправдоподобно, — заключил он. — Как насчёт использования заговоренных палочек? Заговоренные палочки не лгут. По крайней мере всякий, кто скажет, что они лгут, будет сожжён заживо. Это очень болезненная смерть; следовательно, гадальные палочки не лгут.

— Вы можете пользоваться гадальными палочками, если вам нравится, — сказал я.

— О, небеса, разумеется нет, — говорит Марк. — Они работают безошибочно, с абсолютной точностью в любой ситуации, при надлежащем использовании благословенных инструментов; но что если на следующее утро будут мёртвые овцы? Я использую гадальные палочки только когда исключена возможность неверного предсказания. Иначе я буду сожжён заживо. Так как работает ваше магическое ведро?

Как работает ведро?.. Пожалуй, я начну с простейшей возможной ситуации.

— Ну, — говорю я, — предположим, что пастбища пусты, а ведро — не пустое. Тогда мы тратим часы в поисках овец, которых нет. А если на пастбищах остались овцы, но ведро пустое, то Отри и я вернёмся слишком рано, и на следующее утро мы найдём мёртвых овец. Так что пустое ведро является магическим, только если пастбища пусты…

— Постой, — говорит Отри. — Звучит как бессмысленная тавтология. Разве не очевидно, что пустое ведро и пустые пастбища — одно и то же?

— Она не бессмысленная, — говорю я. — Здесь аналогия: логик Альфред Тарски однажды сказал, что утверждение «снег белый» истинно тогда и только тогда, когда снег белый. Если ты можешь понять это, то ты должен видеть, почему пустое ведро волшебно тогда и только тогда, когда пастбища свободны от овец.

— Постойте, — говорит Марк. — Это ведра. Они никак не связаны с овцами. Ведра и овцы, очевидно, совершенно разные вещи. Никакого способа взаимодействия овец с ведром просто нет.

— Тогда откуда же появляется магия, как вы думаете? — поинтересовался Отри.

Марк задумался.

— Вы сказали, что сравнили два ведра, чтобы проверить, что они заполнены на одном уровне… Я могу понять, как вёдра взаимодействуют с вёдрами. Может быть, когда вы соберете много вёдер, и они будут иметь одинаковый уровень — это то, что сгенерирует магию. Я бы назвал это когерентистскойтеориеймагических_вёдер.

— Интересно, — сказал Отри. — Мне известно, что мой хозяин работал над системой с несколькими ведрами — он говорил, что должно работать лучше, потому что «избыточность» и «коррекция ошибок». На мой взгляд, это похоже на когерентизм.

— Это не совсем то же самое… — начал я.

— Давайте проверим когерентистскую теорию магии, — сказал Отри. — Я вижу, у вас с собой еще пять вёдер. Я дам вам ведро, которым мы пользуемся, а затем вы наполните свои вёдра до того же уровня…

Марк в ужасе отпрянул:

— Стойте! Эти вёдра передаются в моей семье через поколения, и у них всегда одинаковый уровень! Если я приму ваше ведро, моя коллекция вёдер станет менее связной, и магия уйдет!

— Но в данный момент ваши вёдра вообще никак не связаны с овцами! — протестует Отри.

Марк рассердился.

— Смотри, я уже объяснял ранее, очевидно, что овцы никак не могут взаимодействовать с вёдрами. Вёдра могут взаимодействовать только с другими ведрами.

— Я бросаю камешек, когда проходит овца, — заметил я.

— Когда овца проходит, вы бросаете камешек? — сказал Марк. — Как это связано с остальным?

— Это взаимодействие между овцой и камешками, — ответил я.

— Нет, это взаимодействие между вами и камешками, — сказал Марк. — Магия не происходит от овцы, она происходит от вас. Овца как таковая — не магическая по определению. Магия приходит откуда-то по пути в ведро.

Я указал на деревянную конструкцию на воротах. — Видите кусок ткани, свисающий из этой деревянной штуковины? Мы постоянно возимся с ней — она ненадежна — но когда овца проходит, она задевает тряпку. Когда ткань отклоняется, камешек из резервуара падает в ведро. Так нам с Отри не обязательно самим бросать камешки.

Марк нахмурился.

— Я не успеваю за вами… Это магическая ткань?

Я пожал плечами.

— Я заказал это онлайн в компании «Естественный Отбор». Ткань называется «Сенсорная Модальность», — увидев скептический взгляд Марка и Отри, я сделал паузу. — Допускаю, что названия звучат несколько эзотерично. Суть в том, что проход овцы начинает цепочку событий и в конце мы имеем камешек в ведре. Теперь вы можете сравнивать ведро с другими вёдрами, и так далее.

— Я так и не понял, — сказал Марк. — Вы не можете поместить овцу в ведро. В ведро попадают только камешки, и, очевидно, эти камешки могут взаимодействовать только с остальными камешками.

— Овца взаимодействует с вещами, которые взаимодействуют с камешками… — я ищу аналогию. — Допустим, вы опустили взгляд на шнурки. Фотон покидает Солнце, затем путешествует сквозь атмосферу Земли, затем отскакивает от ваших шнурков, затем проходит через зрачок в вашем глазе, затем сталкивается с сетчаткой, затем поглощается палочкой или колбочкой. Энергия фотона возбуждает нейрон, который возбуждает другие нейроны. Схема активации нейронов в вашей зрительной коре может взаимодействовать с вашими убеждениями о ваших шнурках, если такие убеждения уже есть в вашей нервной ткани. Если вы можете понять это, то вы должны понять, как проход овцы вызывает попадание камешка в ведро.

— И всё-таки, в какой момент процесса камешек становится магическим? — спрашивает Марк.

— Так… Это… — теперь уже я начинал смущаться. Я встряхнул головой, прогоняя путаницу. Всё казалось таким простым, когда я проснулся этим утром, и система ведра и камней избавила меня от проблем. — Будет гораздо проще понять это, если вы вспомните, что цель данной системы — упрощение наблюдения за овцами.

Марк погрустнел.

— Никогда бы не подумал… Похоже, вы сами не знаете. Может, все камешки изначально волшебны, даже до попадания в ведро. Можно назвать эту позицию панкаменизм.

— Ха! — воскликнул Отри, подражая его манере. — Какая натянутая мысль! Камешки не созданы одинаковыми. Те камешки, что в вашем ведре, не волшебны. Это простые кусочки камня!

Лицо Марка стало суровым.

— Теперь, — крикнул он, — теперь ты видишь, на какую опасную дорожку ступил! Как только ты заявил, что чьи-то камешки волшебны, а чьи-то — нет, твоя гордыня тебя поглотит! Ты возомнишь себя выше всех остальных и так падёшь! История знает множество людей, которые убивали и истязали, потому что считали, что именно их камешки выше прочих! — Голос Марка обрел тень снисходительности. — Поклонение уровню камешков как «магическому» подразумевает существование абсолютного уровня камешков в Великом Ведре. Никто не верит в Великое Ведро в наше время.

— Во-первых, — сказал я, — овцы не являются абсолютными камешками. Во-вторых, я не считаю, что моё ведро на самом деле содержит овец. В-третьих, я не поклоняюсь уровню моего ведра как идеальному — я его поправляю иногда, и делаю это, потому что забочусь об овцах.

— Кроме того, — говорит Отри, — если кто-то считает, что обладанием абсолютными камешками развязывает руки для убийств и пыток, он совершает ошибку, которая никак не связана с вёдрами. Вы решаете не ту проблему.

Марк успокаивается, говоря:

— Это же всего лишь пастухи, чего более мне от них ожидать… Вы, наверное, и в то, что снег белый, тоже верите?

— Эмм… ну да, а что? — отвечает Отри.

— А вас не беспокоит, что Иосиф Сталин тоже верил в то, что снег белый?

— Ну… нет.

Марк некоторое время с недоверием пристально смотрит на Отри, затем пожимает плечами.

— Ладно, предположим, — исключительно для дискуссии, — что ваши камешки магические, а мои нет. Скажите, в чём разница?

— Мои камешки представляют овец! — торжественно заявляет Отри. — У ваших камешков нет свойства представительности, поэтому они не работают. Они лишены смыслового наполнения. И просто посмотрите на них — никакой ауры содержательного наполнения, просто камни. Вам нужно ведро с особыми причинно-следственными силами.

— Ага, — говорит Марк. — Значит теперь это уже «особые причинно-следственные силы», а не магия.

— Вот именно, — продолжает Отри. — Я не суеверен. Постулировать существование магии в наши времена неприемлемо для международного пастушеского сообщества. Мы обнаружили, что постулирование магии просто не работает в качестве объяснения феноменов скотоводства. Так что теперь, если я вижу что-то мне непонятное и хочу объяснить это с использованием модели без детализации, которая не позволяет делать предсказания, даже в ретроспективе, я постулирую существование особых причинно-следственных сил. Если это не работает, то я буду называть это стихийным феноменом.

— А какими особыми силами наделено это ведро? — спрашивает Марк.

— Хм, — задумывается Отри. — Возможно, оно заряжено сродством с пастбищами. Это бы объяснило, почему оно работает — когда ведро пусто, значит и на пастбищах тоже пусто.

— Где вы нашли это ведро? — продолжает задавать вопросы Марк. — И как вы поняли, что ему свойственно сродство с пастбищами?

— Да это обычное ведро, — говорю я. — Я с него на деревья залезал. Не заморачивайтесь на этом вопросе.

— Я всё-таки говорю с Отри, — не даёт себя сбить Марк.

— Нужно привязать ведро к пастбищам и камешки к овцам с помощью магического ритуала… то есть, простите, стихийного процесса с особыми причинно-следственными силами, который открыл мой хозяин, — объясняет Отри.

Отри пытается описать процесс ритуала, при этом Марк с умным понимающим видом кивает.

— Нужно бросать камешек в ведро каждый раз, как овца выходит из загона, — уточняет Марк. — И доставать камешек каждый раз, когда овца возвращается?

Отри кивает.

— Это, должно быть, очень тяжёлое занятие, — сочувствующе говорит Марк.

Отри оживляется, поглощая сочувствие Марка, как сухая земля — дождь.

— Вот именно! — восклицает он. — Это такое давление на чувства. Когда в ведре некоторое время держится один уровень, вы… в некотором роде начинаете ощущать свою связь с этим уровнем.

В это время из загона выходит овца. Отри замечает это, замолкает, берет камешек и поднимает его вверх на вытянутой руке.

— Узрите! — провозглашает он. — Вышла овца! И сейчас я должен бросить камешек в это ведро, моё дорогое ведро, и разрушить этот дорогой мне уровень, который продержался так долго… — Из загона выходит другая овца, Отри, поглощённый своей драматической игрой, не замечает её, поэтому я кидаю камешек в ведро. А он продолжает: — ибо таково высшее испытание пастуха, бросить камень в ведро, несмотря на муки, страдания, несмотря на всю любовь к старому уровню. Воистину, только лучшие из пастухов способны соответствовать столь жестокому требованию…

— Отри, — прерываю я его, — если ты хочешь когда-нибудь стать великим пастухом, научись затыкаться и бросать камешки в ведро. Без суеты. Без драматизма. Просто бросай.

— А этот ритуал, — возвращается к разговору Марк, — он привязывает камешки к овцам силой магических законов Взаимодействия и Распространения, как кукла вуду.

Отри вздрагивает и оглядывается по сторонам.

— Прошу вас! Не называйте это Взаимодействием и Распространением. Мы, пастухи, антисуеверный народ. Пользуйтесь словом «преднамеренность» или чем-то подобным.

— Можно взглянуть на камешек? — спрашивает Марк.

— Да, конечно, — отвечаю я. Достаю один камешек из ведра и бросаю его Марку. Затем наклоняюсь к земле, подбираю другой и кидаю его в ведро.

Отри озадаченно смотрит на меня: — Разве вы сейчас не нарушили весь процесс?

Я пожимаю плечами:

— Вряд ли. Если нарушил — мы узнаем об этом, когда найдём убитую овцу на утро, или если несколько часов проведём в бесплодных поисках.

— Но… — сомневается Отри.

— Я научил тебя всему, что ты знаешь, но не всему, что знаю я, — говорю я ему.

Марк изучает камешек, тщательно рассматривая его. Он проводит над ним рукой, бормочет какие-то слова и наконец отрицательно качает головой.

— Я не чувствую никакой магии, то есть, прошу прощения, не чувствую никакой преднамеренности.

— Камешек обладает преднамеренностью только внутри маг… стихийного ведра, — заявляет Отри. — Иначе это просто камешек.

— Да не проблема, — отвечаю я. Достаю камешек из ведра и выкидываю его. Затем подхожу к Марку, касаюсь его руки с камешком и говорю: — Я объявляю эту руку частью магического ведра! Затем я возвращаюсь на свой пост у ворот загона.

— Вы сейчас просто беспричинно вредничаете, — смеется Отри.

Я киваю, ибо это действительно так.

— Однако это действительно сработает? — спрашивает Отри.

Я снова киваю, надеясь на свою правоту. Я проделывал это раньше с двумя ведрами, и, в принципе, не должно быть разницы между ведром и рукой Марка. Даже если рука Марка насыщена силой жизни, которая отличает живую материю от мёртвой, этот фокус должен сработать, даже если бы Марк был мраморной статуей.

Марк немного обеспокоенно смотрит на свою руку.

— Так что… сейчас камешек снова обладает преднамеренностью?

— Да, — подтверждаю я. — Не берите больше камней в руку и не выбрасывайте тот, который в ней держите, а то вы нарушите ритуал.

Марк торжественно кивает и возвращается к изучению камня.

— Теперь я понимаю, как ваши стада плодятся так хорошо. С силой этого ведра можно просто кидать в него камешки, и овцы бы возвращались с полей. Можно начать с нескольких овец, подождать, пока они уйдут на пастбища, а потом до краёв наполнить ведро. А если следить за такими большими стадами станет утомительно, можно просто выпустить их всех и выкинуть почти все камешки из ведра — так что вернутся только несколько овец. А когда придёт время стрижки, можно снова увеличить стада… святые небеса! Да вы хоть представляете всю чистую силу этого ритуала, который открыли? Я могу только представить последствия, возможно, это прыжок вперёд на десять — нет, на сто лет для всего человечества!

— Так не заработает, — сказал я. — Если вы забросите камешек без вышедшей овцы, либо достанете камешек, хотя овца не вернулась, это нарушит ритуал. Сила не содержится в камешках, но пропадает мгновенно, как мыльный пузырь.

Марк страшно разочаровался.

— Вы уверены?

Я кивнул.

— Я пробовал, и это не сработало.

Марк тяжело вздыхает.

— До этого момента вся эта… математика казалась такой… сильной и полезной. А был бы такой большой прогресс для человечества. А, ладно.

— Марк, это отличная идея, — одобряюще сказал Отри. — Это не моё открытие, оно носится в воздухе… можно сэкономить громадные средства… это должно спасти ваш план! Мы можем попробовать разные вёдра, найдя то, что сохраняет изначальную магическую сил… преднамеренность камней даже без ритуала. Или попробовать другие камешки. Может, наши камешки имеют не те свойства, присущие преднамеренности. Что если мы попробуем использовать камни, вырезанные в форме маленьких овечек? Или просто напишем «овца» на камешках, этого вполне достаточно.

— Не поможет, — сухо предсказал я.

Отри продолжил.

— Может, нам нужны органические камешки вместо кремниевых… или даже драгоценные камни. Цена драгоценных камней удваивается каждые восемнадцать месяцев, так что вы можете купить пригоршню недорогих драгоценных камней, подождать, и через двадцать лет они станут очень дорогими.

— Вы пробовали добавить камешков в ведро, чтобы сотворить ещё овец, и это не сработало? — спрашивает меня Марк. — А что конкретно вы делали?

— Я взял пачку долларовых банкнот. Затем я спрятал их, одну за другой, под складкой одеяла. Каждый раз, когда я прятал банкноту, я доставал из коробки скрепку и складывал их кучкой. Я тщательно старался не считать в уме, так что в итоге я знал только то, что у меня есть «много» долларовых банкнот и «много» скрепок. Когда все банкноты были спрятаны под одеялом, я добавил одну скрепку в кучку скрепок, что равносильно забросу одного лишнего камешка в ведро. После этого я начал доставать банкноты из-под одеяла, одновременно складывая скрепки назад в коробку. Когда я закончил, осталась одна лишняя скрепка.

— И что это значит? — спрашивает Отри.

— Значит, что уловка не удалась. Как только я нарушил ритуал всего лишь одной ошибкой, сила мгновенно исчезла. Кучка скрепок и кучка долларов теперь не уменьшались одновременно.

— Вы и правда это пробовали? — поинтересовался Марк.

— Да, — отвечаю я. — Я действительно провёл этот эксперимент, чтобы удостовериться в том, что результат согласуется с моим теоретическим предсказанием. У меня есть сентиментальная склонность к научной методике, даже когда дело похоже на бред. Кроме того, а что если бы я ошибся?

— Если бы это сработало, — говорит Марк, — вы были бы повинны в фальшивомонетничестве! Представьте, если бы все этим занялись — экономика бы рухнула! У всех были бы миллиарды долларов наличности, на которые ничего нельзя было бы купить!

— Вовсе нет, — уверил я. — По этой же логике, если добавление скрепки в кучку создавало бы ещё один доллар, то этот доллар при создании создавал бы дополнительно товаров и услуг на доллар.

Марк осуждающе качает головой:

— Все равно, подделка денег — это преступление. Вам не следовало пытаться это осуществить.

— Я был обоснованно уверен, что у меня не получится.

— Ага! — восклицает Марк. — Вы ожидали, что у вас не получится! Вы не верили, что у вас может получиться!

— Действительно. Вы с ошеломляющей точностью угадали мои ожидания, — признаю я.

— Ну вот, в этом и проблема, — оживлённо заявляет Марк. — Магия стимулируется верой и силой воли. Если не верить, что сможешь, то и не сможешь. Вам нужно изменить свою веру в результат опыта, это изменит и сам результат.

— Забавно, — с ностальгией вспоминаю я. — То же самое сказал Отри, когда я рассказал ему о методе ведра и камешков. Что это слишком смехотворная чушь, чтобы он в неё поверил, поэтому этот метод у него не сработает.

— И как же вы его убедили? — спрашивает Марк.

— Я сказал ему заткнуться и следовать инструкциям — а когда метод сработал, Отри начал в него верить.

Марк озадаченно хмурится:

— Но это же бессмыслица. Это не решает основную дилемму курицы и яйца.

— Отнюдь, решает. Метод ведра работает, верите вы в него или нет.

— Что за ерунда! — пробормотал Марк. — Не верю я в магию, которая работает независимо от того, веришь ты в нее или нет!

— Я говорил то же самое, — вмешался Отри. — Очевидно, я ошибался.

На лице Марка видны следы напряжённого размышления.

— Но… если вы не верили в магию, которая работает, веришь в неё или нет, то почему метод с ведром сработал, когда вы в него не верили? Вы что, верили в магию, которая работает, веришь в неё или нет, независимо от того, веришь ли в магию, которая работает, веришь ты в неё или нет?

— Я… по-моему… нет, — с сомнением отвечает Отри.

— Тогда, если вы не верили в магию, которая работает, веришь в неё или нет, то… сейчас, это надо решить в письменном виде. — Марк лихорадочно строчит на листе, затем скептически смотрит на получивший результат, переворачивает листок вверх ногами и наконец сдается. — Неважно, — резюмирует он. — Даже просто магию мне трудно понять, а уж метамагия вообще вне моего понимания.

— Марк, по-моему вы не понимаете, в чём мастерство ведёрного ремесла, — говорю я. — Дело не в том, чтобы с помощью камешков управлять овцами. Дело в том, чтобы овцы управляли камешками. В этом творческом деле не обязательно начинать с веры в то, что оно сработает. Скорее, сначала человек становится свидетелем того, что ведёрное ремесло действительно работает, а затем приходит к вере, что оно работает.

— Или вы верите, что это так, — парирует Марк.

— Да, я верю, что это так, потому что это имеет место быть как факт, — соглашаюсь я. — Соответствие между реальностью и моими верованиями происходит от того, что моя вера обусловлена реальностью, а не наоборот.

Мимо проходит ещё одна овца, поэтому я кидаю в ведро ещё один камешек.

— Ага! Вот мы и добрались до корня проблемы, — восклицает Марк. — Что вообще за дело с этой так называемой «реальностью»? Я понимаю, что значит, когда гипотеза изящна или фальсифицируема, или согласуется с фактами. По мне так называть веру «истинной», или «настоящей», или «действительной» в отличие от просто веры — это то же самое, что сказать «я верю в то-то» и сказать «я очень-очень верю в то-то».

Я немного медлю с ответом.

— В общем… — задумчиво говорю я, — откровенно говоря, я и сам не до конца уверен, откуда взялась вся эта муть с «реальностью». Я не могу создать собственную реальность в лаборатории, поэтому понять, что это такое, пока нельзя. Но время от времени я сильно верю, что что-то случится, а вместо этого затем происходит что-то другое. Мне нужно как-то называть это «что бы то ни было», которое определяет мои опытные результаты, поэтому я называю это «реальностью». Эта «реальность» как-то совсем не совпадает иногда даже с моими лучшими гипотезами. В некоторых случаях, когда у меня есть простая гипотеза, которая прекрасно согласуется со всеми известными мне данными, даже и тогда случаются сюрпризы. Так что мне нужно по-разному называть те штуки, которые определяют предсказанные мной результаты, и ту штуку, которая определяет опытный результат. Первое я называю «верой», а второе — «реальностью».

Марк фыркает.

— Даже не знаю, почему я вообще слушаю всю эту очевидную чепуху. Всё, что вы говорите об этой так называемой «реальности» — это всего лишь ещё одна вера. Даже ваше мнение о том, что реальность существует прежде ваших убеждений — это тоже вера. Из этого с логической неизбежностью следует, что реальность не существует, существуют только убеждения.

— Погодите, — говорит Отри, — вы не могли бы повторить последнюю часть? Вы где-то в середине фразы круто повернули, и я потерял нить.

— Что бы вы ни говорили о реальности, всё это только ещё одна вера, — объясняет Марк. — Из этого с ошеломляющей неотвратимостью следует, что реальности нет, есть только убеждения.

— Понятно, — говорю я. — Значит, аналогично, не важно, что человек ест, он должен есть ртом. Из этого следует, что еды не существует, есть только рты.

— Именно, — отвечает Марк. — Всё, что вы едите, должно быть во рту. Может ли еда существовать вне рта? Это бессмысленно, что доказывает, что «еда» — это несвязная идея. Поэтому мы все голодаем до смерти, еды не существует.

Отри смотрит на свой живот.

— Но я же не голодаю.

— Ага! — ликующе восклицает Марк. — А как вы произнесли это самое возражение? Ртом, друг мой! Своим ртом! Чем ещё лучше можно было наглядно доказать, что еды не существует?

— Кто тут говорит о голоде? — требовательно вопрошает резкий, скрипучий голос у нас за спиной. Мы с Отри спокойно стоим, мы слышали его и раньше. Марк же прямо подпрыгивает от неожиданности, сильно испуганный.

Инспектор Дарвин сухо улыбается, довольный получившимся сюрпризом, и делает небольшую пометку в своем блокноте.

— Это просто метафора! — быстро заявляет Марк. — Не надо забирать у меня рот или что-то ещё…

— А зачем вам рот, если еды не существует? — сердито настаивает Дарвин. — Неважно. У меня нет времени на эту ерунду. Я здесь, чтобы проинспектировать овец.

— Стада плодятся, господин, — сообщаю я. — Ни одной задранной овцы с января.

— Прекрасно. Награждаю вас 0,12 единицами приспособляемости. Так, а что этот человек тут делает? Является ли он необходимой частью производственной деятельности?

— Насколько я понимаю, он принёс бы больше пользы человеческому виду, если его повесить как балласт на корзине воздушного шара, — отвечаю я.

Отри тихо ойкает.

— Мне нет дела до человеческого вида. Пусть сам скажет.

Марк надменно выпрямляется.

— Вот этот всего лишь пастух, — он показывает на меня, — заявляет, что существует такое явление, как реальность. Это оскорбляет меня, так как я с глубокой и стойкой уверенностью знаю, что правды не существует. Идея «истины» — это всего лишь уловка людей, чтобы навязывать другим людям свои убеждения. У разных культур разные «истины», ни одна из которых не превосходит другую. То, что я сказал, имеет силу в любом месте и в любое время, и я настаиваю, чтобы вы согласились с этим.

— Секундочку! — вступает Отри. — Если ничто не истинно, почему я должен верить вам, когда вы говорите, что ничто не истинно?

— Я не говорил, что ничто не является истинным, — отвечает Марк.

— Нет, сказали, я слышал, — не соглашается Отри.

— Я сказал, что «истина» — это оправдание, которое используется какой-либо культурой, чтобы навязать другим свои убеждения. Так что, когда вы заявляете, что что-то «истинно», вы имеете в виду только то, что верование в это принесёт вашей собственной социальной группе какие-то преимущества.

— А вот это, что вы сейчас изложили — это истинно? — говорю я.

— Абсолютно, несомненно истинно! — выразительно отвечает Марк. — Люди сами творят свои реальности.

— Погодите, — озадаченно говорит Отри, — рассуждая логически, заявлять о том, что люди создают собственные реальности — это совершенно отдельный вопрос от заявления о том, что истины не существует — такое положение дел я даже внятно представить не могу, может быть, потому что вы так и не пояснили, как это вообще должно работать…

— Ну вот опять, — раздражается Марк, — опять вы пытаетесь применить свои западные идеи логики, рациональности, причинности, обоснованности и непротиворечивости.

— Ну отлично, — бурчит Отри, — теперь мне надо добавить третий индекс, чтобы следить за этим особым и отдельным утверждением…

— Оно не отдельное, — отвечает Марк. — Слушайте, вы неверно относитесь к моим утверждениям, принимая их за гипотезы и тщательно выводя их следствия. Вам следует считать их полностью универсальными оправданиями, которые я применяю, когда кто-то заявляет что-то, что мне не нравится. Это не столько модель того, как работает вселенная, сколько карточка «Освобождение из тюрьмы». Ключевой момент — применять эти оправдания избирательно. Когда я говорю, что истины не существует, это применимо только к вашему заявлению о том, что магическое ведро работает независимо от того, верю я в него или нет. Это не применимо к моему заявлению о том, что истины не существует.

— Эмм… а почему нет? — вопрошает Отри.

Марк терпеливо вздыхает.

— Отри, неужели вы думаете, что вы первый, кто задался этим вопросом? Спросил, как может наша собственная вера быть наполнена глубочайшим смыслом, если все верования бессмысленны? Многие студенты задают такой же вопрос, когда они сталкиваются с этой философией, у которой, я вас уверяю, множество сторонников, и по которой написана обширная литература.

— Ну и каков ответ? — говорит Отри.

— Мы назвали его «проблемой рефлексивности», — объясняет Марк.

— Да, но какой же ответ? — продолжает настаивать Отри.

Марк снисходительно улыбается.

— Поверьте, Отри, вы не первый, кто придумал такой простой вопрос. Не нужно преподносить его так триумфальное опровержение.

— Да нет, в самом деле, какой ответ?

— А теперь я бы хотел перейти к проблеме того, как логика убивает таких милых детёнышей тюленей…

— Вы напрасно теряете время! — резко обрывает Инспектор Дарвин.

— Это кроме того, что вы далеко ушли от слежения за овцами, — говорю я, закидывая ещё один камешек.

Инспектор Дарвин бросает взгляд на спорщиков, каждый из которых, очевидно, не хочет сдавать свою позицию.

— Послушайте, — говорит он уже более мягким тоном, — У меня есть простое решение вашего диспута. Вот вы утверждаете, — обращается Дарвин к Марку, — что вера человека преобразовывает его личную реальность. А вы, — он поворачивается и указывает на Отри, — истово верите в то, что вера Марка не может изменить реальность. Так пусть Марк очень сильно поверит в то, что он умеет летать, и спрыгнет с обрыва. Марк увидит, что полетел, как птица, а Отри увидит его стремительное падение и затем услышит шлепок приземления, и вы оба будете счастливы.

Мы все нерешительно застываем, обдумывая это.

— Звучит разумно… — наконец говорит Марк.

— А вот там как раз есть обрыв — осматривается Инспектор Дарвин.

Отри выглядит глубоко задумавшимся. Наконец он восклицает:

— Погодите! Если бы это было правдой, то мы все давно бы оказались в собственных отдельных вселенных, и тогда все люди здесь были бы всего лишь плодом вашего воображения — нет смысла пытаться что-то доказать нам…

С ближайшего обрыва слышится длинный затихающий крик, за которым следует глухой одиночный шлепок. Инспектор Дарвин открывает свой планшет на странице, на которой показан современный генетический пул и карандашом вписывает в показатель распространённости аллелей Марка несколько более низкое значение.

Отри несколько бледнеет.

— Неужели это было действительно необходимо?

— Необходимо? — озадаченно переспрашивает его Инспектор Дарвин, — Это просто произошло… Я не вполне понимаю ваш вопрос.

Мы с Отри возвращаемся к нашему ведру. Пора загонять овец. Не стоит забывать об этом. Иначе к чему бы всё это было?

Перевод: 
BT
Оцените качество перевода: 
Средняя оценка: 3.4 (475 votes)